PEDIATRICS

Ace your homework & exams now with Quizwiz!

the most common etiologic agent for bacterial tracheitis? A. Hemophilus Influenza B. Staphylococcus aureus C. Streptococcus pyogenes D. Streptococcus pneumoniae E. Moraxella catarrhalis

311 B. Nelson 18th ed page 1766

Rectal prolapse in an infant associated with failure to thrive is highly suggestive of? A. Crohn's disease B. Functional constipation C. Ehler-Danlos syndrome D. Intussusception of the colon E. Cystic Fibrosis

634 E. Cystic fibrosis in children with rectal prolapse and failure to thrive is common.

An 8 month old child is expected to have what number of teeth/tooth? A. 1 B. 2 C. 3 D. 4 E. None

644 B. Age in months - 6 months = expected number of teeth

Live attenuated vaccines except? A. MMR B. BCG C. Yellow fever D. Polio Salk E. Influenza

665 D. Salk polio vaccine is inactivated whole virus, while Sabin polio vaccine is live attenuated.

Karyotyping result of a patient with Klinefelter syndrome will show: A. 45 XO B. 47 XXY C. 46 XY D. None of the above

433 B. SIMILAR TO PREVIOUS BOARD EXAM CONCEPT/PRINCIPLE

Precocious puberty is defined as onset of puberty before age ___ years in girls and age 9 years in boys: A. 7 B. 8 C. 9 D. 10 E. 11

453 B. Precocious puberty is the onset of secondary sexual characteristics before 8 years old in girls and 9 years old in boys.

A 10 month old infant will have how many teeth? A. 4 B. 2 C. 1 D. 3 E. 5

477 A. the formula is age in month-6, so 10-6=4.

In diphtheria, the most common cause of death of patient is usually: A. pneumonia B. myocarditis C. sepsis D. meningitis E. Renal failure

162 B.

The classic triad of infectious mononucleosis include the following, except: A. fever B. fatigue C. pharyngitis D. generalized lymphadenopathy E. None of the above

512 A. Nelson's 19th ed, p1111

A 7 year-old child developed high-grade fever and cough. Chest x-ray was done and revealed pneumatoceles. What is the most common pathogenic organism responsible for the above findings: A. Haemophilus influenzae B. Streptococcus pneumoniae C. Staphylococcus aureus D. Parainfluenza virus E. Respiratory syncitial virus

447 C. Staphylococcus aureus is one of the causes of overwhelming hospital-acquired pneumonia. In children, Xray findings in children are rapidly developing lobar/multilobar consolidation, pleural effusion and pneumatocoele.

A neonate was born with a heart rate over 100/minute, slow irregular breathing, some flexion of the extremities, grimacing, pink body with blue extremities. APGAR score at 1 minute is: A. 5 B. 6 C. 7 D. 8 E. 9

451 B. Heart rate over 100/minute=2, slow irregular breathing=1, some flexion of the extremities=1, grimacing=1, pink body with blue extremities=1. 2+1+1+1+1=6

A 15 year-old consulted for leg swelling and pain. Xray showed lytic lesion with periosteal reaction of the tibia. The most likely diagnosis is: A. Osteosarcoma B. Ewing's sarcoma C. Rhabdomyosarcoma D. Chondroblastoma E. Enchondroma

454 B. Ewing's sarcoma is a rare bone cancer that primarily affects children and adolescents. On conventional radiographs, the most common osseous presentation is a lytic lesion with periosteal reaction. The classic description of lamellated or "onion skin" type periosteal reaction is often associated with this lesion.

A child complains of anal pruritus causing sleep disruption. You do a scotch tape test to check for the parasite. Upon identification, what is the drug of choice that you will prescribed for the patient? A. Ivermectin B. Mebendazole C. Pyrantel pamoate D. Diethylcarbamazine E. Thiabendazole

455 C. Enterobiasis is a pinworm infection, a human parasitic disease and one of the most common parasitic worm infections in the developed world. It is caused by infestation of the parasitic roundworm Enterobius vermicularis. The drug of choice for pinworm is Pyrantel pamoate which causes depolarization- induced paralysis in woms. Ivermectin is the drug of choice for strongyloides, mebendazole is for trichuris, diethylcarbamazine is for wuchereria and brugia and thiabendazole is for trichinella.

A G3P2 mother delivered a live term baby Boy with good cry, good activity, peripheral cyanosis, respiratory rate of 60cpm, heart rate of 110bpm. What is the APGAR score? A. 10 B. 9 C. 8 D. 7 E. 6

463 B. The APGAR score is 9 as computed: good cry=2, good activity = 2, peripheral cyanosis = 1, RR of 60cpm = 2, HR = 110bpm.

A 2 year old child presents with non-tender abdominal mass crossing the midline. On abdominal CT scan there is calcification and hemorrhage seen. What is the diagnosis? A. Wilms tumor B. Neuroblastoma C. Metastatic tumor D. Intestinal obstruction E. medulloblastoma

479 B. In neuroblastoma, abdominal mass crosses the midline; while in wilms tumor, it does not cross the midline.

Andy, a 3-day old infant was noted to have progressively deepening cyanosis since birth. Which of the following congenital cardiac malformations is most likely responsible for her cyanosis? A. Transposition of great arteries B. Tetralogy of Fallot C. Tricuspid atresia D. Total anomalous pulmonary venous return

486 A. TGA is the most common cause of early cyanosis in the newborns (unlike TOF which presents with cyanosis after a fews days of life).

Which among the following is not a prominent feature of Wiskott-Aldrich syndrome? A. X-linked recessive inheritance B. Atopic dermatitis C. Thrombocytopenia D. Recurrent infections with encapsulated bacteria E. Leukopenia

5 E. Note: The prominent immunologic impairment in Wiskott Aldrich Syndrome is against polysaccharides of encapsulated organisms Source: Nelson's Textbook of Pediatrics 19th edition p734

A 1 year old baby girl was rushed to the ER because of poor feeding for 3 days now. Condition started 3 days ago when patient developed fever, sore throat and lesions in the posterior pharynx. Physical exam revealed discrete 1-2mm vesicles and ulcers that enlarge and are surrounded by erythematous rings on the anterior tonsillar pillars, soft palate, uvula, tonsils and posterior pharyngeal wall. What is your primary consideration? A. German measles B. Measles C. Herpangina D. Chickenpox E. Streptococcal pharyngitis

514 C. Nelson's 19th ed, p1090

A 4 year old boy is seen with pink, maculopapular rash on the buttocks and legs with vague, intermittent abdominal pain. His mother reported that patient has recently recovered from cough and colds a week ago. Urinalysis revealed microscopic hematuria. the rest of the labs were unremarkable. What is your primary consideration? A. Henoch-Schonlein Purpura B. Kawasaki Disease C. IgA Nephropathy D. Post-Streptococcus Glomerulonephritis E. None of the above

515 A. IgA nephropathy is the most common chronic glomerular disease worldwide. It most commonly presents with gross hematuria within 1-2 days of onset of an upper respiratory or gastrointestinal infection. Nelson's 19th ed., 1781-82

What is the pathognomonic microscopic finding in the rash and exanthem of measles? A. Koplik spots B. Warthin-Finkeldey giant cells C. Forchheimer spots D. All of the above E. None of the above

517 Forchheimer spots- petechial hemorrhages in the soft palate in patients with German mealsesNelson-1076; Koplik spots represent the enanthem in measles and is the pathognomonic sign of measles. Nelson-1070

A 15 year old boy is brought to the physician because of headaches for 3 months. The headaches occur most often in the morning and are associated with vomiting. Over the past month, he also has had increasing visual difficulty. During this period, he has had a 4.5 kg weight gain, and his school performance has declined. He is also concerned that his pubertal development has been slower that his friends. Fundoscopic examination shows mild papilledema. Which of the following is the most likely diagnosis? A) Adrenal insufficiency B) Craniopharyngioma C) Medulloblastoma D) Migraines

524 B.

A previously healthy 10 month old boy is brought to the ER by his parents because he has vomited twice during the past 2 hours. During this period, he has had episodes of inconsolable crying and has passed one bloody stool. On arrival, he appears lethargic and is difficult to arouse. His temperature is 38C, pulse is 110 bpm, respirations are 24 cpm, and blood pressure is 90/65 mmHg. The abdomen is soft with no masses or organomegaly. Rectal examination shows bloody mucus in the rectal vault. Which o the following is the most appropriate next step in diagnosis? A) Contrast enema B) Upper GI series with small bowel follow-through C) CT scan of the head D) Colonoscopy

526 A.

Major criteria for diagnosis of acute rheumatic fever includes the following, except A. Polyarthritis B. Erythema nodosum C. Subcutaneous nodules D. Carditis E. NOTA

53 B. B should be erythema marginatum.

This is the most common cause of hematuria second only to Berger’s disease: A. PSGN B. FSGS C. Cresentic Glomerulonephritis D. Lupus nephritis

572 A. PSGN is the second most common cause of hematuria in children following igA Nephropathy

The systolic blood pressure of a 4 year old boy with no known morbidities will be considered low if it falls below: A: 59 mmHg B. 67 mmHg C. 78 mmHg D. 90 mmHg

576 C. BP= Age x 2 + 90 (upper SBP range) BP= Age x 2 + 70 (lower SBP range) Therefore, 4 x 2 + 70 and 4 x 2+90 = 78 â€" 98 mmHg is the normal SBP range of a 4 year old child. Below 78mmHg is already considered low.

Factors that are more characteristic of childhood- onset Asthma than adult-onset asthma? A. A ratio of 2:1 females to males B. An asscociation with history of eczema C. Normal levels of IgE D. Lack of response to provocation with inhaled antigens E. Lack of seasonal variation in severity of symptoms

655 B.

A 3 month old infant has had tachypnea and tachycardia for 10 days; during this period he has been feeding poorly. A grade 3/6 holosystolic murmur and a grade 2/6, apical mid-diastolic murmur are heard. An x-ray of the chest shows cardiomegaly with increased pulmonary vascular markings. Echocardiography shows a large ventricular septal defect. Which of the following is the most likely cause of these symptoms? A) Excessive pulmonary blood flow B) Mitral valve obstruction C) Reduced left ventricular contractility D) Reduced right ventricular preload

528. A.

The most common cause of cardiac arrest in the pediatric population is due to a primary failure of what organ system? A. Cardiovascular B. Respiratory C. Metabolic D. Neurologic

580 B. In contrast to adults in which cardiac causes account for majority of Sudden cardiac arrests, the pediatric population most commonly suffer arrests due to Respiratory causes.

A mother is concerned because her 6 year old son developed low grade fevers, painful ulcers in the mouth, and rashes on his hands and feet. What could be the likely causative agent? A. Adenovirus B. Coxsackievirus A16 C. Cytomegalovirus D. Echovirus 22 E. Epstein Barr virus

65 B. Coxsackie virus A16 is the most common cause of hand, foot and mouth disease which is characteristic of the case.

A 16 year old male was brought to ER due to severe abdominal pain and tea-colored urine. History revealed Viral pneumonia a week PTC. PE showed macular rashes on lower extremities. What is your impression? A. Henoch Schonlein Purpura B. Post-strep Glomerulonephritis C. IgA nephropathy D. Berger's disease E. Buerger's disease

651 A.

A 6 month old infant came in for complaints of stridor. Upon workup, a barium esophagram shows a posterior indentation of the esophagus. Which of the following is the most likely diagnosis? A. laryngomalacia B. vascular ring C. posterior laryngeal cleft D. congenital subglottic hemangioma E. congenital subglottic stenosis

100 B. Vascular ring results from abnormal development of aortic arch complex. The double aortic arch is the most common complete vascular ring encircling both trachea and esophagus compressing both. Nelson's Textbook of Pediatrics 19th edition

Males and females are equally affected except in: A. Autosomal dominant disorder B. Autosomal recessive disorder C. X-linked disorder D. Y-linked disorder E. Mitochondrial inheritance

101 C. As the X chromosome is one of the sex chromosomes (the other being the Y chromosome), X-linked inheritance is determined by the sex of the parent carrying a specific gene and can often seem complex. This is due to the fact that, typically, females have two copies of the X- chromosome, while males have only one copy. The difference between dominant and recessive inheritance patterns also plays a role in determining the chances of a child inheriting an X-linked disorder from their parentage. Males are normally hemizygous for the X chromosome, having only one copy. As a result, X- linked dominant disorders usually show higher expressivity in males than females Page 34 of Topnotch Handout

You are given a case of 2 year-old female suffering from loose watery diarrhea of 5-6 episodes per day. Condition started just a day prior to consult and no other symptoms like fever or vomiting noted. PE showed weak-looking child, eager to drink, and skin draws back slowly. According to IMCI, what is the best mangement? A. Give probiotics B. Give ORS C. Start IV fluids at 75ml/kg D. Observe E. None of the above

657 B.

According to IMCI, every measles patient or suspected measles patient should be given A. Vitamin A B. Pyridoxine C. Vitamin C D. Zinc E. Active immunization

658 A.

The most common Nephrotic Syndrome in children is? A. Lipoid Nephrosis B. Minimal change disease C. Membranoproliferative glomerulonephritis D. A or B E. A or C

659 D.

Which of the following is/are contraindications for immunization? A. Mild acute illness in an otherwise well child B. Prematurity C. Allergy to egg protein D. A and b E. AOTA

666 C. Contraindications for vaccination includes fever, moderate - serious illness, egg allergy.

How should Hepatitis B vaccine be given for routine vaccination? A. Infant of Hepa B positive mother should receive 1st doe of HBV plus Hepa B Ig at 2 different sites within 12 hours of birth B. Total of 6 does by 18 mos of age C. all children not immunized need not begin vaccination due to acquired immunity D. b and c E. none

667 A. Hepatitis B vaccine is given at birth and a total of 3 doses os given by 18 mos of age. All children not immunized or does not have booster, must be started on the regimen.

The recommended Breastmilk storage period is: A. 1 hour at room temperature more than 25C B. 10 days in a refrigerator C. 3 weeks in a freezer of a 1 door refrigerator D. 4 months in a freezer of a 2 door refrigerator E. 7 months in a deep freezer with constant temperature of -20C

173 A., D., E. Based on the Philippine Pediatric Society Preventive Pediatric Health Care Handbook page 24, the ONLY answer is A but, cross referencing it to pedia topnotch handout, E can also be considered. If CDC guidelines will be used, A D and E are the answers.

Mantoux testing of 7 year-old patient in remission from acute leukemia after receiving immunosuppressive doses of corticosteroids showed 7 mm of induration. Interpret this finding. A. Positive B. Negative C. Indeterminate D. Latent E. Active

216 A. Induration of >5 mm is considered positive among patients receiving immunosuppressive therapy or with immunosuppressive conditions including HIV infection.

A ball was placed in front of a young child afterwhich a piece of cloth was placed over the ball, hiding it in sight. The child lifted the cloth and found the ball beneath it. The youngest age that this child could possibly be is: A. 1 month old B. 4 months old C. 9 months old D. 15 months old E. 18 months old

218 C. Object permanence seen around 9 months of age.

The Steeple sign of croup represents: A. Deviation of the trachea B. Subglottic narrowing C. Edema of the glottis D. Retropharyngeal abscess E. Hyperinflated lungs

219 B. This sign may be absent in patient with croup, present in patients without croup as a normal variant, and may occasionally be seen in patient with epiglottitis.

A 15/F presents with pallor and easy fatiguability. CBC was done revealing a low Hgb. You decide to start the patient on iron supplementation. When will you repeat the CBC in order to check if the Hgb is already rising? A. 12-24 hours after B. after 3 days C. after 1 month D. after 3 months E. after 6 months

380 C. 12-24 hours subjective improvement; 26-48 hours initial BM response; 72 hours retic count increases; 4- 6 weeks Hgb increases; 1-3 months repletion of iron stores Topnotch

A child is considered enuretic if he/she has not attained full bladder control by what age? A. 2 years B. 3 years C. 4 years D. 5 years E. 6 years

381 D. A child is expected to be toilet trained by day at 2 years old and dry by night at 3 years old. He/she is considered enuretic if he/she remains unable to exhibit bladder control by 5 years old.

A child is expected to have a complete set of primary teeth by what age? A. 2 years B. 2.5 years C. 3 years D. 4 years E. 6 years

382 C. The first permanent molars erupt at 6 years of age.

An infant is expected to double his/her birth weight by what age? A. 2-3 months B. 4-5 months C. 8-9 months D. 12 months E. 15 months

383 B. Birth weight triples by 1 year old.

A 2-day old full-term infant was brought to you due to multiple 1-2 mm papules, each with a distinct erythematous base, scattered over his trunk. There are no other associated symptoms. Biopsy of these lesions will reveal predominance of which of the following cells? A. Lymphocytes B. Macrophages C. Neutrophils D. Eosinophils E. No cells will be seen.

384 D. Diagnosis: Erythema toxicum

Which of the following findings would make you consider a pathologic cause for neonatal jaundice? A. Bilirubin level peaks at 2nd to 4th day of life B. Jaundice resolves within 14 days C. Total bilirubin of 10 mg/dL D. Direct bilirubin is 7% of total bilirubin E. Total bilirubin increases by 1.0 mL/dL/hr

386 E. The rest of the choices are consistent with physiologic jaundice.

Which of the following is not consistent with breastfeeding jaundice? A. It results from the presence of glucuronidase in breastmilk. B. It typically occurs within the 1st week of life C. It is corrected with continued breastfeeding >10x/day D. It presents with laboratory finding of indirect hyperbilirubinemia. E. It typically lasts for only a few days.

387 A. Choice A refers to breastmilk jaundice.

Based on Philippine guidelines, deworming every 6 months should be performed on children between what ages? A. 12 months to 5 years B. 12 months to 14 years C. 5 years to 14 years D. 5 years to 18 years E. 6 months to 5 years

388 B.

A 7 year old child with severe cough and petechial hemorrhages was brought tou you by his mother, apparently no fever was noted, no malaise, no sore throat was noted, However on CBC, The WBC count was 40,000/L, Neutrophils: 20%, Lymphocytes 70%, Monocytes 8%, Eosinophiles 2%. What would be the drug of choice for your patient? A. Penicillin G B. Racemic Epinephrine C. Oseltamivir D. Erythromycin E. None, conservative treatment only

418 D. Classic case of Pertussis. Erythromycin is the drug of choice

Inheritance of glucose-6-phosphate dehydrogenase deficiency A. Autosomal dominant B. Autosomal recessive C. X-linked dominant D. X-linked recessive E. Non-mendelian

42 D. Mutation in the G6PD gene on the X chromosome. G6PD deficiency is an X-linked recessive disorder.

A 6 month old patient presents with low-grade fever, rhinorrhea and cough. On PE wheezing was noted all over with hyperresonance on percussion. Hyperinflated lungs are noted on CXR, among which of the following is your initial diagnosis: A. Bronchiolitis B. Bronchial Asthma C. Pneumonia D. Croup E. Epiglotitis

420 A.

A 5 year old child presents with fever of 6 days duration associated with nonpurulent conjunctivitis, dry and fissured lips, nonvesicular rashes on the trunk, and cervical lymphadenopathy. What is the most specific diagnostic test? A. antibody determination B. acute phase protein level determination C. complete blood count D. urinalysis E. none of the choices

465 E. This is a case of Kawasaki disease in which there is no specific diagnostic test. The diagnosis of Kawasaki is primarily clinical.

A 3 week old neonate presents with nonbilious vomiting. On examination of the abdomen, there is olive-shaped mass. What is the diagnosis? A. Pyloric stenosis B. Duodenal atresia C. Volvulus D. Meckel diverticulum E. GERD

466 A. Pyloric stenosis is the most common cause of nonbilious vomiting with a firm, movable, olive- shaped mass in the abdomen.

A 30 year old father with a homozygous Vitamin D resistant rickets transmitted the trait to all of his daughters but to none of his sons. Therefore it is: A. Autosomal dominant B. Autosomal recessive C. X-linked recessive D. X-linked dominant E. multifactorial inheritance

467 D. When an affected male transmits the trait to all his daughters but to none of his sons, it is X-linked dominant. Vitamin D resistant rickets is an example.

One of the following does not describe the colostrum. A. Produced in the first 0-7 days B. More than 80% water C. High levels of fat and carbohydrates D. Rich in protein E. None of the choices

468 C. colostrum has low fat and carbohydrates

A 3 old child was brought for consultation due to rashes. Upon review of history, the mother said that the child had fever for 4 days, and that rashes appeared when fever abated. What is the etiologic agent? A. RNA virus from paramyxoviridae B. RNA virus from Togaviridae C. Human herpes virus 6 D. Human herpes virus 1 E. Coxsackie virus A

469 C. this is a classic roseola infection, rashes appear when fever abates. The etiologic agent is human herpes virus 6.

A 5 y/o M came into the ER because of marked increase in cyanosis followed by loss of consciousness. He was previously diagnosed with Tetralogy of Fallot. Which among the following physical examination finding will support the diagnosis of a hypercyanotic spell? A. Loss of murmur B. Bounding pulses C. Arrhythmia D. Retractions E. NOTA

47 A. Tet spell or hypercyanotic spell (total occlusion of right ventricular outflow)

A 1 year old child was brought to the emergency department due to seizure at the height of the fever. The seizure lasted for 2 minutes and then the patient had another 3 seizure episodes within the 30 minute period without regaining consciousness. What is the drug of choice to give? A. valproic acid B. diazepam C. phenytoin D. phenobarbital E. carbamazepine

470 C. this is a case of status epilepticus and the drug of choice is phenytoin.

In congenital syphilis, one of its manifestations is mulberry molar. What tooth is affected? A. First lower molar B. Second lower molar C. First upper molar D. Second upper molar E. Upper central incisors

471 A. mulberry molar - abnormal first lower molar; hutchinson teeth - barrel shaped upper central incisors

A 17 year old sex worker came in for consultation, complaining of fever for 3 days associated with fatigue and generalized lymphadenopathy. On examination, there is splenomegaly and severe pharyngitis with marked tonsillar enlargement, with petechiae at the junction of the hard and soft palate. What is the most feared complication? A. carcinoma B. splenic rupture C. heart disease D. secondary bacterial infection E. hepatic rupture

472 B. This is a case of infectious mononucleosis caused by EBV. The most feared complication is splenic rupture caused due to trauma in the 2nd week of illness

A 3 year old child presents with 2 day fever and white discrete papules with central umbilication in the extremities, head and neck. What is the diagnosis? A. Yellow fever B. Molluscum contagiosum C. Allergic reaction D. Chicken pox E. Erythema multiforme

473 B.

What is the most common congenital heart disease? A. Ventricular septal defect B. Atrial septal defect C. Tetralogy of fallot D. Patent ductus arteriosus E. Tricuspid atresia

474 A.

A 1.8 kg newborn presents with chorioretinitis, microcephaly and hepatomegaly. On imaging, there is intracerebral calcifications seen. What is the diagnsosis? A. Herpes simplex virus infection B. Congenital rubella C. Congenital syphilis D. Cytomegalovirus infection E. Toxoplasmosis

475 E. intracerebral calcification is seen in toxoplasmosis, while periventricular calcificartion is seen in CMV. Other choices do not have cerebral calcification.

What type of rhabdomyosarcoma will present with vaginal grape-like mass? A. Sarcoma botryoides B. Alveolar C. Embryonal D. Pleomorphic E. Undifferentiated

476 A.

Periorbital violaceous or heliotropic rash followed by proximal muscle weakness is seen in: A. Scleroderma B. Systemic lupus erythematosus C. Dermatomyositis D. Erysipelas E. Systemic sclerosis

478 C.

The following are the blood pictures of herediatry spherocytosis except: A. Loss of biconcave shape B. Decreased MCHC C. Normal MCV D. High reticulocyte count E. None of the choices

480 B. In hereditary spherocytosis, MCHC is increased

A child with unexplained bone pain, easy bruisability, and hepatomegaly was suspected to have lipid storage disease. Enzyme assay of cultured fibroblasts showed defects in B- glucosidase activity. Which of the following diseases does he have? A. Gaucher Disease B. Niemann-Pick Disease C. Tay-Sachs Disease D. Sandhoff disease

683 A. Answer: A. Gaucher Disease Notes: • Gaucher â€" B-glucosidase defect • Niemann-Pick â€" Acid sphingomyelinase defect • Tay-Sachs â€" B-hexosaminidase defect • Sandhoff â€" B-hexosaminidase defect

A mother is concerned about her daughter’s weight. The 9 yo girl weighs 26 kg. Nutritional history reveals a well-balanced diet. What advice will you give? A. Enroll her daughter on a weight loss program B. Prescribe multivitamins and mineral supplements C. Assure her that weight for age is normal and continue with her diet D. Tell her to wait for her menarche and then you will reassess

684 C. Answer: C. Assure her that weight for age is normal and continue with her diet Notes: • Weight is the best index of growth and nutrition • Mnemonics: o Infants <6 mos: Wt (gm) = age (mos) x 600 + BW o 6-12 mos: Wt (gm) = age (mos) x 500 + BW o >2 yo: Wt (kg) = age (years) x 2 + 8

A 14 year old boy has 3+ protein and 4+ blood with RBC casts on urinalysis. Further questioning revelaed 2 prior episodes of tea coloured urine concurrent with upper respiratory tract infection during the last 3 years. Which of the ff is the most likely diagnosis? A. Membranous nephropathy B. SLE nephritis C. IgA nephropathy D. Membranoproliferatifve GN E. Post streptococcal GN

80 C. This patient's clinical presentation are consistent with IgA nephropathy. They usually present with recurrent bouts of gross hematuria associated with respiratory infections.

A 3 year old female comes to you with complaints of a rapidly growing left abdominal mass that crosses the midline associated with a purplish rash and weight loss. Which of the following is the most likely diagnosis? A. rhabdomyosarcoma B. hepatoblastoma C. wilm's tumor D. neuroblastoma E. nephroblastoma

93 D. Nelson’s Textbook of Pediatrics 19th edition

A 6 week old infant is brought to you because of difficulty breathing of 3 days duration. The patient was born vaginally, at term, and with no fetomaternal complications. On PE, you note that the patient has clear rhinorrhea, bilaterally erythematous conjunctivae with discharge; and scattered crackles without wheezing. What is the most likely causative agent in this infection? A. Rhinovirus B. Streptococcus pneumoniae C. Respiratory syncytial virus D. Chlamydia pneumoniae E. Chlamydia trachomatis

546 E. Pneumonia in an infant with eye discharge is consistent with Chlamydia. Treat with oral erythromycin x 2 weeks for either pneumonia / conjunctivitis.

A 2 year old male was seen in the ER due to fever of 5 days duration. This wasassociated with vomiting and anorexia. There were no cough, colds associated. CBC revealed leukocytosis with predominance of Segmenters. Urinalysis revealed TNTC pus cells and 5-10/hpf RBC. Impression : UTI. The gold standard for the diagnosis of UTI is: A. Urine Culture and sensitivity B. Leukocyte esterase test C. Urinalysis D. Nitrite test

561 A. Urine CS is the gold standard in the microbiologic diagnosis

A box-shaped heart on Chest Xray is seen in: A. TGA B. Ebstein Anomaly C. Pericardial effusion D. TAPVR

568 B.

Unilateral Moro reflex was noted in a newborn after a difficult labor and delivery. The affected arm was adducted, extended and medially rotated while the forearm is pronated and the carpal flexors flex the hand at the wrist. What is the diagnosis? A. Winged scapula B. Klumpke's paralysis C. Erb-Duchenne syndrome D. Clavicular fracture E. Ulnar nerve injury

189 C.

A 6 month old infant was brought to the clinic due to severe dyspnea and coughing, he was noted to have subcostal retractions, bilateral crackles and wheezes on both lung fields, RR of 70, slightly febrile at 37.6 degrees celsius, which of the following is are appropriate management? A. IV corticosteroids B. salbutamol nebulization C. ribavirin D. intubation as necessary E. all of the above

249 E. SIMILAR TO PREVIOUS BOARD EXAM CONCEPT/PRINCIPLE, all of the above are correct possible management

The most common symptom of rheumatic fever: A. Migratory polyarthritis B. Carditis C. Sydenham chorea D. Erythema marginatum E. Subcutaneous nodules

370 A. Choices arranged from most common to least common with the following frequencies: 75%, 50-60%, 10- 15%, <3%, <1% Nelsons 19th pg 921-922

A 3-day old infant with a single 2nd heart sound has had progressively deepening cyanosis since birth but no respiratory distress. Chest radiography demonstrates no cardiomegaly and normal pulmonary vasculature. ECG shows an axis of 1200 and RV prominence. Which of the following is most likely responsible for the cyanosis? A. Tetralogy of Fallot B. Transposition of the great vessels C. Tricuspid atresia D. TAPVR

695 B. Answer: B Transposition of the great vessels w/ an intact ventricular septum presents w/ early cyanosis, a normal-sized heart (classic egg on string in xray). Normal or slightly inc.pulmo.vascular markings and r axis deviation w/ RVH in ECG.

A 2 years old male had chronic constipation since infanthood. PE revealed globular abdomen and the rectal vault is empty, no feces on examining finger. What test should be done to confirm the diagnosis? A. Abdominal x-ray B. CT scan C. Ultrasound D. Colonoscopy E. Suction biopsy

200 E. Diagnosis is Hirschprung Disease. Gold Standard for diagnosis is rectal suction biopsy.

A neurologic disease with Albuminocytologic dissociation on Lumbar tap is: A. Myasthenia Gravis B. Meningitis C. Neuroblastoma D. Tuberous Sclerosis E. Guillane-Barre Syndrome

419 E. Albuminocytologic Dissociation: increased protein with very low or normal Cell count on CSF analysis.

Parahemophilia is deficiency of what coagulation factor? A. Factor 5 B. Factor 8 C. Factor 9 D. Factor 11 E. None of the above

44 A. Factor 5 - Parahemophilia Factor 8 - Hemophilia A Factor 9 - Hemophilia B Factor 11 - Hemophilia C

What 4 elements comprise the tetralogy of Fallot, except? A. VSD B. Pulmonic stenosis C. Overriding of the aorta D. RVH E. None of the above

45 E.

A 9 year old male patient was brought to your clinic due to cough for more than a month accompanied by marked weight loss. Her mother also has chronic cough. If Mantoux test shows an induration of 10 mm, the most likely diagnosis is: A. TB disease B. TB infection C. TB exposure D. Latent TB E. Miliary TB

135 A. TB disease : 3 or more of the following criteria: 1. Exposure to an adult/adolescent with active TB disease; 2. (+) PPD; 3. s/sx suggestive of TB; 4. (+) Abnormal cxr findings; 5. Lab findings. TB infection : +/- exposure, (+) PPD, no s/sx, (-) cxr findings

which of the following is not a clinical criteria in the diagnosis of perinatal asphyxia? A. Acidosis (pH 7.0) B. Evidence of neurological injury C. APGAR score of 0-5 in > 2 mins D. Multisystem organ failure E. None of the above

231 C. APGAR score of 0-5 in > 5 mins

Major features of atopic dermatitis in children include all of the following except: A. Pruritus B. Facial and extensor eczema C. Angioedema D. Chronic or relapsing course

355 C.

Failure to thrive is defined as: A. Growth at less than 5th percentile on the growth chart B. Weight or heigt less than the 5th percentile C. Growth patterns cross two major percentiles within the past 6 months D. All of the above

365 D. All are correct. Topnotch

A 10 year old girl presents for evaluation of fatigue, diminished appetite, and weakness. On physical examination, a periorbital violaceous heliotrope rash is evident. Which of the following statements is most accurate regarding the probable diagnosis? A. Children with this diagnosis typically present with distal muscle weakness with an ascending pattern B. This patient has a 25 % likelihood of developing a subsequent malignancy C. Steroids are contraindicated D. The clinical course may be complicated by calcium deposition in the muscle, fascia, and subcutaneous tissue E. This patients's disease is more common in males than in females

635 D. This is a case of dermatomyositis. Calcinosis or calcium deposition, in msucle, fascia and subcutanous tissue occurs in 70 % OF CASES.

The ability to copy forms develops in a regular order. Which of the following is the correct sequence? A. Copy a square, a cross, a circle B. Copy a square, a circle, a cross C. Copy a cross, a circle, a square D. Copy a circle, a square, a cross E. Copy a circle, a cross, a square

642 E.

Infant presented with barking cough, hoarseness and inspiratory stridor which was relieved by administration of nebulized epinephrine. What is the causative agent? A. RSV B. H. influenzae type b C. Parainfluenza virus D. Influenza E. Strep. pneumoniae

668 C. Croup (or laryngotracheobronchitis) is a respiratory condition that is usually triggered by an acute viral infection of the upper airway, usually parainfluenza The infection leads to swelling inside the throat, which interferes with normal breathing and produces the classical symptoms of a "barking" cough, stridor, and hoarseness.

An 8 year old has a chest radiograph that shows cadiomegaly with a snowman appearance. What is the most likely diagnosis? A. Tetraloogy of Fallot B. Transposition of the great arteries C. Truncus arteriosus D. Total anomalous pulmonary venous connection E. Tricuspid atresia with intact ventricular septum

76 D. The classic CXR appearance of TAPVR is the snowman appearance

A febrile 10-year old patient was brought to you by his mother due to the appearance of maculopapular rashes, which occurred the night before consult where the highest fever grade was noted at 39.2 deg C. She then asks you how early can her child go back to school? A. 4 days after the onset of rash B. 6 days after the onset of rash C. 4 days after the onset of fever D. 6 days after the onset of fever E. 8 days after the onset of fever

406 A. Measles. Period of communicability is 4 days before and 4 days after the onset of rash.

What is the treatment of choice in vaso-occlusive crisis of sickle cell anemia? A. Hydration and pain medications B. Exchange transfusion C. Splenectomy D. Antibiotic prophylaxis E. None of the above

41 A. For vaso-occlusive crisis - Hydration and pain medications; For CVA - Exchange transfusion with chronic transfusion protocol to keep Hgb S <30%; For splenic sequestratrion - Emergent transfusion with subsequent splenectomy; For aplastic crisis - Supportive care; transfusion if necessary; For infections -Appropriate antibiotics, penicillin prophylaxis, vaccines for S. pneumoniae, H. influenzae, N. meningitidis

A 6 year old Male patient presents with a 4-day fever, strawberry tongue, Erythema of the hand and Bilateral cervical lymphadenopathy.Among which of the following would be the appropriate treatment for the patient? A. IV Ig 2g/kg over 10-12 hours B. Aspirin 3-5 mg/kg orally once daily C. Observe and treat conservatively D. Give steroids E. Antibiotics

413 E. The following characteristics does not match the criteria for Kawasaki, Scarlet fever is a more appropriate option, hence antibiotics should be administered.

A 12 month-old born term, presented to the ED because of an abnormally enlarging head, Macewen sign, and a setting-sun sign. Among which of the following statements is the reason for the Occular manifestation of the child? A. Impingement of the cranial nerves B. Impingement of the tectum C. Retraction of the orbit D. Enlargement of the cranium creates the illusion of a setting sun E. Impingement of the Occipital lobe

414 B.

A 2- year old patient was diagnosed with Acute Bacterial Meningitis. CSF culture was performed revealing a H. influenza infection. Prior to antibiotic treatment Dexamethasone was given, what is the rationale for this action? A. To reduce CSF production B. To reduce brain inflammation C. To reduce brain edema D. To reduce auditory nerve damage E. To reduce optic nerve damage

415 D. Dexamethasone is given to reduce auditory nerve damage, given 1-2 hours before antibiotics are started.

10 year old patient developed periorbital edema and oliguria 2 weeks after treatment for a throat infection. Urine findings showed hematuria and cylindriuria. Which among the following findings is diagnostic of this case? A. low C3 B. leukopenia C. ASO titer 180 D. A and C E. All of the above

188 A. SIMILAR TO PREVIOUS BOARD EXAM CONCEPT/PRINCIPLE.. low C3 is the only compatible finding for post-strep gn in this. ASO titer should be gretaer than 200 in adults and 400 in children. Anti- DNAse is used in PSGN if it is preceded by streptococcal skin infection.

Which antibody mediates the vasculitis present in Henoch-Schonlein purpura? A. IgA B. IgG C. IgM D. IgD E. IgE

208 A. There is deposition of IgA and C3 in the small vessels of the skin and the renal glomeruli.

A 4 week-old infant is brought to the emergency room by his mother. She claims that there is a recent onset of projectile vomiting after feeding and that the infant usually appears hungry and wants to feed more. Physical examination of the child reveals mild dehydration and a firm, movable mass is palpated. What is the best test to order to confirm this diagnosis? A. Ultrasound B. Abdominal X-ray C. CT scan D. MRI E. None of the above

277 A. Ultrasound is the best test to conform cases of congenital pyloric stenosis

Which of the following is a known teratogenic effect that results from cocaine abuse during pregnancy? A. Ebstein anomaly B. Cystic cortical lesion C. Phocomelia D. Patent ductus arteriosus E. Spina bifida

385 B. Lithium: Ebstein anomaly. Thalidomide: phocomelia. Rubella: PDA.

Which of the following is the mainstay of therapy in idiopathic thrombocytopenic purpura? A. Corticosteroid therapy B. Plasma exchange C. Platelet transfusion D. IVIg therapy E. None of the above

389 A. Plasma exchange is the mainstay of management in thrombotic thrombocytopenic purpura. Platelet transfusion is contraindicated in both ITP and TTP.

A 5-year-old girl is brought to the physician because of temperatures to 40 C (104 F), tachypnea, and a nonproductive cough for 12 hours. Four days ago she was treated with an oral antibiotic for suspected pneumococcal pneumonia. Examination shows diminished breath sounds over the lower right lung fields and dullness to percussion at the right costophrenic angle. Which of the following is the most likely diagnosis? A) Bronchopleural fistula B) Empyema C) Lung abscess D) Pleurodynia

531 B.

A newborn in the delivery room cried vigorously, with regular respirations, pinkish body and bluish extremities, active, with a heart rate of 146/min. What is the APGAR score? A. 6 B. 7 C. 8 D. 9 E. 10

639 D. APGAR of 9/10

At your clinic in the province, a mother brought her 3 year old child with maculopapular rashes on trunk and limbs. Mother told you that 2 days prior the rash, her child had fever of 38-39 degrees Centigrade. With this history, you know this is a case of? A. Measles B. German Measles C. Roseola D. Rubella E. Rubeola

647 C. the rasher after fever indicates Roseola, locally known as tigdas-hangin

A 2 year old child was brought for consult due to poor feeding. Mother told you that patient had been experiencing high grade fever for the past 6 days. PE showed erythematous eyes, unilateral lymphadenopathy, strawberry tongue, edematous hands and feet with desquamating rash. What is the most appropriate treatment? A. Immunoglobulins B. IV hydrocortisone C. Aspirin D. A and B E. A and C

650 E.

An infant born at 39 weeks of gestation and weighing 2000 grams should be classified as? A. Low birth weight B. premature C. Small for gestational age D. A and C E. B and C

653 D.

handedness is usually established at an age of? A. 2 y/o B. 3y/o C. 4y/o D. 5y/o E. None of the above

313 B. Nelson 18th ed p54

A 6 week old infant has gained no weight since birth. Her skin appears mottled, extremities are cold, and her temperature is 35C. The most likely diagnosis is: A. Sepsis B. Galactosemia C. Hypothermia D. Hypothyroidism

358 D.

The MOPP regimen (mustine, oncovin, procarbazine, prednisone) is used in the treatment of which of the following malignant diseases? A. Lymphoma B. Wilms tumor C. Neuroblastoma D. Seminoma E. Acute lymphoblastic leukemia

398 A. SIMILAR TO PREVIOUS BOARD EXAM CONCEPT/PRINCIPLE.

Very low birth weight infants weigh less than ___ grams: A. 1000 grams B. 1500 grams C. 2000 grams D. 2500 grams E. 3000 grams

449 B. Low birth weight infants weigh <2500 grams, very low birth weight infants <1500 grams, extremely low birth weight infants <1000grams

Anaphylaxis results in what type of shock A. Hypovolemic B. Cardiogenic C. Distributive D. Septic E. Adrenal crisis

59 C.

The immediate postnatal changes in a term newborn includes the following, EXCEPT A. Decrease in pulmonary vascular resistance B. Decrease in right to left shunting via ductus arteriosus C. Increase in venous return to the left atrium D. Increase right to left shunting via foramen ovale

602 D. Foramen ovale functionally closes immediately after birth.

An 8 year old girl presents with sore throat, fever and a rough sandpaper-like rash over her trunk and extremities. A throat culture is positive for group A beta hemolytic streptococcus. Treatment of her infection with antibiotics will prevent which of the following complications? A. Reactive arthritis B. Rheumatic Fever C. Pneumonia D. RPGN E. Meningitis

623 B.

The age that a patient most likely has when she can dress with help, ride a tricycle, knows her own age and can speak in short sentences is A. 1 year B. 2 years C. 3 years D. 4 years

692 C.

Administration of this vaccine component in older children causes a neuroparalytic side effect: A. DPT B. TT C. Rota Virus D. MMR-V

577 A. the pertussis (cellular) component is not encouraged among older children and adults because of its possible neuroparalytic side effect. Therefore an acellular pertussis component is substituted (DTaP) or is disregarded (Td

to diagnose Systemic Lupus Erythematosus in a patient, you need to satisfy how many of the criteria? A. 3 B. 4 C. 6 D. 7 E. 8

177 B. only four criteria are needed

A neonate was born via emergency cesarean section from a mother who had seizurers secondary to eclampsia, the infant was noted to be floppy, cyanotic, with weak cry and HR of 80, initial resuscitation was instituted, the criteria for the diagnosis of perinatal asphyxia involves all EXCEPT? A. blood pH of <7.0 B. 5 minute apgar score of 0-3 C. hypoxic ischemic encephalopathy - altered tone decreased level of consciousness D. signs of multiorgan system dysfunction E. all of the above are true

246 E. SIMILAR TO PREVIOUS BOARD EXAM CONCEPT/PRINCIPLE, criteria for asphyxia

In the Philippines, the most common cause of neonatal meningitis is ? A. E. coli B. group B streptococci C. Listeria monocytogenes D. streptococcus pneumonia E. Haemophilus influenzae

250 A. SIMILAR TO PREVIOUS BOARD EXAM CONCEPT/PRINCIPLE, in developing countries usually E coli is the most common cause of neonatal meningitis

The newborn with cytomegalovirus (CMV) infection at birth most commonly presents with? A. hepatosplenomegaly B. hepatitis C. thrombocytopenia D. Cerebral calcifications E. No symptoms

654 E.

You have been following an 8 year old child in your iffce for the past several years and have noted that durign the past year, his height has remained below the third percentile. You are conerned about his short stature and decide to begin a workup. On bone age determination it was discoviered that the px's bone age is 4 years younger than his chronological age. Which of the ff diagnosis should be considered? A. Genetic short stature B. Skeletal dysplasias C. Intrauterine growth retardation D. Turner's syndrome E. Growth hormone deficiency

72 E. BA<CA - GH deficiency, constitutional growth delay, hypothyroidism, hypercortisolism BA=CA - genetic short stature, skeletal dysplasias, IUGR, Turner's syndrome

1. A 5 year old female came in for complaints of cough and watery nasal discharges 1 week PTA. Upon examination at the ER of CVG hospital, T = 38C RR = 42 with nasal flaring and subcostal retractions. Crackles are heard on both lung fields. Abdominal distention was also noted. CXR revealed hyperinflation with bilateral interstitial infiltrates and peribronchial cuffing. Which of the following is the most likely diagnosis? A. bronchiolitis B. pneumonia C. acute bronchitis D. asthma E. None of the above

81 B. Nelson's Textbook of Pediatrics 19th edition p.1477

You are the medical intern assigned at the NICU, you know that the importance of the 1 minute APGAR score is? A. The risk of development of neurologic sequelae B. The adequacy of resuscitation C. The need for resuscitation D. Both A and B E. None of the above

145 C. The one minute APGAR score signifies the need to perform resuscitative measures to save the life of the newborn while the other options are the implications of the 5 minute APGAR score

In treatment Plan B for diarrhea using ORS, ___ cc/kg is given to a patient for 4 hours. A. 50 B. 75 C. 100 D. 125 E. 150

179 B.

A 9-year-old child has developed headaches that are more frequent in the morning and are followed by vomiting. Over the previous few months, his family has noted a change in his behavior (generally more irritable than usual) and his school performance has begun to drop. Imaging of this child is most likely to reveal a finding in which of the following regions? A. Infratentorial B. Supratentorial C. Intraventricular D. Spinal canal E. Peripheral nervous system

40 A. Brain tumors are the most common solid tumor in childhood, and account for 25% to 30% of all pediatric malignancies. While supratentorial tumors predominate in the first year of life (including choroid plexus tumors and teratomas), brain tumors in children 1 to 10 years old are more frequently infratentorial (posterior fossa) and include cerebellar and brainstem tumors such as medulloblastoma or cerebellar astrocytoma. After 10 years of age, supratentorial tumors (eg, diffuse astrocytoma) are again more common.

What is the duration of antibiotic treatment for meningitis caused by penicillin-sensitive pneumococcus? A. 3-5 days B. 5-7 days C. 7-10 days D. 10-14 days E. NOTA

48 D.

The anterior fontanel usually closes by: A. 10-12 months B. 13-15 months C. 16-18 months D. 19-21 months E. NOTA

49 C.

The plasticity of the brain continues into A. Infancy B. Childhood C. Adolescence D. Adulthood E. NOTA

50 C. The plasticity of the brain continues into adolescence.

True of object permanence (constancy) except: A. Defined as the understanding that objects continue to exist even when not seen B. Infants persist in searching C. Peek-a-boo D. Achieved at 10 months old E. None of the above

51 D. A major milestone is the achievement by 9 mo of object permanence (constancy). Stranger anxiety almost same time. Introduction of a transitional object may allow the infant to self comfort in the parents' absence. The object cannot have any potential for asphyxiation or strangulation.

A 9 month M with a purpuric rash is unresponsive when brought to the ER. The pulse is barely palpable. After several attempts, you are unable to establish peripheral venous access. Of the ff, which is the BEST site for the next attempt to provide fluid resuscitation for this patient? A. Femoral artery B. Internal jugular vein C. Intraosseous space D. Saphenous vein E. Superficial temporal artery

52 C.

What is the ideal dose to give a 10 kg child with iron deficiency anemia? A. Elemental iron, 3-6 mg/day B. Elemental iron, 30-60 mg/day C. Elemental iron, 300-600 mg/day D. Elemental iron, 33-66 mg/day E. Elemental iron, 0.3-0.6 mg/day

54 B. The ideal dose for iron supplemention is 3-6 mg/kg/day. For ths child, it should be 30-60 mg/day of elemental iron.

Which pf the following Plasmodium species causes the most severe form of malaria A. P. falciparum B. P. malariae C. P. knowlesi D. P. ovale E. P. vivax

55 A. Plasmodium falciparum is the most severe form and has the highest fatality rate.

A 5 y/o boy come in the ER with generalized edema, BP 90/60, HR 80 bpm, BUN 250 mg/dL, Cr 0.5 mg/dL, urine protein is 3600 mg/day. The most common histologic finding in this patient is A. Membranous nephropathy B. Focal segmental glomerulosclerosis C. Minimal change disease D. Membranoproliferative nephropathy E. Mesangioproliferative nephropathy

57 C. The most common cause of nephrotic syndrome amongst children is MCD, responsive to steroids most of the time.

A previously healthy girl presents with a 1 week history of cough, runny nose, fever, sore throat and red eyes. She went to her pediatrician 2 days ago and was prescribed Augmentin (amoxicillin and clavulanate) for presumed pharyngitis. Yesterday, Ana developed a red rash which started on her face and has spread to her trunk. Her mother insists that the rash is from her new medication. What is the most likely diagnosis? A. Exanthem subitum B. Rubella C. Rubeola D. Erythema infectiosum E. Fixed drug eruption

66 C. Rubeola will classically present with the the prodrome of fever, malaise, conjunctivitis, cough, coryza and an exanthem of erythematous macules and papules beginning on the face and spreading cephalocaudally and centrifugally (by the 3rd day, the whole body is involved).

A previously healthy girl presents with a 1 week history of cough, runny nose, fever, sore throat and red eyes. She went to her pediatrician 2 days ago and was prescribed Augmentin (amoxicillin and clavulanate) for presumed pharyngitis. Yesterday, Ana developed a red rash which started on her face and has spread to her trunk. Her mother insists that the rash is from her new medication. All of the ff are true regarding this condition EXCEPT? A. In measles with no bacterial infection, the ESR and CRP are normal B. The portal of entry is both the respiratory tract and the conjunctivae C. Measles infection causes necrosis of the respiratory tract epithelium with lymphocytic infiltrates D. Patients are infectious from 3 days before up to 4- 6 days after the onset of rashes. E. In individuals with passively aquired antibody such as infants and recipeinets of blood products, a subclinical form of measles may occur called atypical measles

67 E. In individuals with passively aquired antibody such as infants and recipeinets of blood products, a subclinical form of measles may occur callled inapparent measles. Atypical measles is a more severe form of measles due to the use of the original formalin containing vaccine. All other statemetns are true.

What is the most common cause of death of pediatric patients afflicted with measles? A. Otitis media B. Meningitis C. Encephalitis D. Dehydration due to diarrhea and vomiting E. Pneumonia

68 E. Pneumonia is the most common cause of death in measles. It may manifest as giant cell pneumonia caused directly by the viral infection or as superimposed bacterial infection.

Vitamin A supplementation for infants 6-11 months is given at a single dose of _______IU as recommended by PPS and DOH. A. 50,000 B. 100,000 C. 150,000 D. 200,000 E. 300,000

171 B. 100,000 IU for infants 6-11 months and 200,000 IU for children 12-59 months.

A term male infant delivered via emergency CS at 39 weeks AOG presented with early onset jaundice, the mother is O+ and the infant is B+, infant was breastfed, which is the most likely diagnosis: A. RH incompatibility B. Breastfeeding jaundice C. Breastmilk jaundice D. Sepsis E. ABO incompatibility

105 E. Page 5 of handout. ABO incompatibility is the most common cause of hemolytic disease of the newborn. Mother is usually type O and baby is either A or B. Most cases are mild jaundice and hepatosplenomegaly.

A 6 y/o boy developed anasarca. On PE, BP 100/70 (p50). Laboratory findings showed Urine protein on dipstick 4+, Urine protein/urine creatinine ratio was elevated. He was started on oral prednisone. After 2 weeks upon follow up, the edema resolved and urinalysis was normal. If this is a nephrotic syndrome, it fits the course of: A. Membranoproliferative GN B. Focal segmental glomerulosclerosis C. Mesangial proliferation D. Minimal change disease E. IgA nephropathy

107 D. SIMILAR TO PREVIOUS BOARD EXAM CONCEPT/PRINCIPLE. This nephrotic syndrome fits minimal change disease. Minimal change disease usually responds well to initial treatment and over 90% of patients will respond to oral steroids within 6â€" 8 weeks, with most of these having a complete remission.

Deworming is contraindicated to children with the following conditions except: A. Severe malnutrition B. Profuse diarrhea C. Abdominal pain D. Low grade fever E. Serious illness

172 D. only high grade fever is contraindicated.

A 15 month old male presents with high fever of 3 days, coryza, dry cough, red eyes with purulent discharge and rashes appearing around the hairline spreading downwards. Discrete red lesions with white spots are noted in the buccal mucosa at the level of the premolars. The most likely diagnosis is: A. Roseola infantum B. Kawasaki disease C. Rubeola D. Rubella E. Exanthem subitum

108 C. SIMILAR TO PREVIOUS BOARD EXAM CONCEPT/PRINCIPLE. Page 57-58 of Topnotch Handout. This is measles. The spots at the level of the premolars are koplik spots.

To prevent cardiac damage, the cumulative dose of doxorubicin should be limited to what value in patients with Wilms tumor? A. 50-100 mg/m2 B. 100-200 mg/m2 C. 200-300 mg/m2 D. 300-400 mg/m2 E. 400-500 mg/m2

217 C. Congestive cardiac failure may occur after treatment with doxorubicin particularly if administered in combination with radiation therapy to the lungs.

A 3 year old is brought to the ER. On PE, inspiratory stridor and a barking cough were noted. There is mild respiratory distress, tachypnea, and fever. The signs and symptoms are aggravated by agitation and crying. The most likely diagnosis is A. Tracheomalacia B. Laryngotracheobronchitis C. Epiglottitis D. Bacterial tracheitis E. Peritonsillar abscess

10 B. This is a case of croup and is common in this age group. Barking cough = croup Epiglotitis and bacterial tracheitis patients are more toxic appearing and not in merely mild respiratory distress Source: Nelson's Textbook of Pediatrics 19th edition p1446

A 5 y/o girl with a two month history of progressive difficulty in ambulation particularly in climbing up and down the stairs. After going on a field trip, she came home with an erythematous rash in the malar area and violaceous discoloration with swelling over the periorbital areas. On the day of consult, she was noted to have papules over the fingerjoints and elbows. The most likely diagnosis is: A. Juvenile dermatomyositis B. Juvenile rheumatoid arthritis C. Systemic lupus erthematosus D. Juvenile ankylosing spondylitis E. Systemic scleroderma

109 A. SIMILAR TO PREVIOUS BOARD EXAM CONCEPT/PRINCIPLE. Page 15 of Topnotch Handout. The rash over the periorbital area is not malar rash, it is a heliotrope rash. Its onset is insidious. Rash over sun exposed areas appear first followed by proximal muscle weakness after 2 months.

A 10 y/o girl presents to your clinic with a history of fever for 4 days, headache, muscle aches and anorexia. The fever lyses but she still feels very weak and complains of some epigastric pain. VS are within the normal range. She appears flushed. What is the initial diagnostic test that you will request? A. CBC with platelet count B. Dengue NS1 C. PT, PTT, BT D. Typhidot E. Dengue blot

112 A. SIMILAR TO PREVIOUS BOARD EXAM CONCEPT/PRINCIPLE. The most important initial test to do first is a CBC.

A 5 y/o child was brought for consult due to 9 days fever (39 C), generalized body malaise, anorexia, and hepatomegaly 3 cms below the costal margin. He was extremely irritable, had bilateral conjunctival injection, with severe dryness and fissuring of the lip, markedly red erythematous tongue, right cervical lymphadenopathy (2cm) with erthema and swelling of the hands and feet. What is the most likely diagnosis? A. Typhoid fever B. Infectious mononucleosis C. Scarlet fever D. Dengue Fever E. Kawasaki disease

120 E. SIMILAR TO PREVIOUS BOARD EXAM CONCEPT/PRINCIPLE.Page 75 of Topnotch Handout. This is typical of kawasaki disease.

Which of the following causes of diarrhea in children warrants antibiotic treatment? A. Rotavirus B. Shigella C. Staphylococcus aureus D. Bacillus cereus E. All of the above

130 B. All other causative agents warrants only supportive treatment such as hydration. S. aureus and B. cereus symptoms are due to preformed toxin. Shigella may be treated with Cefixime or Erythromycin. (Cotrimoxazole in IMCI). SIMILAR TO PREVIOUS BOARD EXAM CONCEPT.

A 10-year-old obese female presented with polyuria, polydypsia, and hyperpigmentation of the skin of the beck arrives at your clinic for consult. Laboratory findings showed a high insulin level, normal C peptide and FBS of 186 mg/dL. The most likely diagnosis is: A. Impaired glucose tolerance B. Type I DM C. Type II DM D. Hypercortisolism E. Cushing's Syndrome

131 C. T1DM is characterized by low or absent level of endogenously produced insulin. T2Dm is chacaracterized by insulin resistance. Acanthosis nigrican or dark pigmentation of skin creases/flexural areas, a sign of insulin resistance, is present in the majority of patients with T2DM and is accompanied by a relative hyperinsulinemia at the time of diagnosis. Impaired glucose tolerance refers to a metabolic stage that is intermediate between normal glucose homeostasis and diabetes. (FBS 100-125 mg/dL). Source: Nelson's. SIMILAR TO PREVIOUS BOARD EXAM CONCEPT.

A 2 year old child came at the ER due to convulsions and upon PE, you noted the child is in active seizure, the mother told you that the child had fever prior to the seizure episode, the first step you should do as the ER ROD is: A. Give diazepam 0.2mg/kg/dose B. Place tongue guard C. Give anti pyretics D. Secure the airway E. None of the above

141 D. Secure and protect the airway first before doing any other procedures by giving supplemental oxygen and placing the patient on his side to prevent aspiration

An 18 month old child was brought to your clinic because of developmental delay, you noted a sweet smelling urine similar to a maple syrup, you know that the enzyme deficient in this case metabolizes which amino acids A. Leucine, lysine, valine B. Isoleucine, phenylalanine, tryptophan C. Leucine, isoleucine, aspartate D. Valine, leucine, isoleucine E. Phenylalanine, tryptophan, glycine

153 D. This is a case of MSUD in which there is a deficiency of the enzyme branched-chain alpha-keto acid dehydrogenase complex causing accumulation of the above amino acids causing the characteristic sweet, maple odor urine.

A child was rushed by bystanders to the ER after a vehicular accident. Her preschool ID indicates that she is 4 years old. What is the approximate weight of the patient in kilograms? A. 12 B. 14 C. 16 D. 18 E. 20

176 C. age in years x 2+8

8 year old male was brought to the clinic due to 3 day history of fever and development of rashes.The rashes were characterized with an appearance described as "dewdrops on a rose petal". Which of the the following is true about the diagnosis? A. posterior auricular lymphadenopathy is positive B. cough, coryza and conjunctivitis are also present C. the described skin lesion is known as the Herman's rash D. there may be reactivation of the infection E the drug of choice is ganciclovir

190 D. The diagnosis is chickenpox/ varicella. It is caused by varicella-zoster virus. The infection becomes latent in dorsal root ganglion and reactivation is possible causing herpes zoster.

2 years old patient developed stridor. Vaccination history is unrecalled. Upon examination, you noted the patient is sitting up on hands, with tongue out and the head forward. There is also drooling noted and the voice is muffled. Imaging test revealed "thumb sign". What is the drug of choice? A. Penicillin B. Racemic epinephrine C. Ceftriaxone D. Azithromycin E. Tetracycline

191 C. The diagnosis is epiglottitis caused by H. influenza. The drug of choice for this condition is Ceftriaxone.

All of the following are live vaccines except? A. Rotavirus vaccine B. MMR vaccine C. BCG vaccine D. HPV vaccine E. OPV vaccine

192 D. All vaccines for viral infections are live except for "RIP Always". Rabies, Influenza, Papilloma and hep A vaccine. BCG is the only live vaccine against a bacterial infection.

A 4 years old male tripped while playing in the streets. He sustained abrasions in the knees and forearms and his right hand was punctured by a fallen branch of a tree. He was brought to your clinic for treatment. The child is completely immunized for age. After cleaning and disinfecting the wound, what should be given for tetanus prophylaxis? A. Give Tetanus Toxoid and TIG B. Give TT only C. Give TIG only D. Give DPT E. None of the above

193 E. Contaminated wounds in patients with >3doses of anti-tetanus vaccine and last dose <5 years do not require tetanus prophylaxis.

A newborn infant presents with generalized cyanosis, irregular respirations, and some flexion of the extremities. Heart rate is 80 bpm. There is no response when a catheter was placed on the nostril. Which of the following statements is NOT true about this infant? A. The APGAR score is 3. B. This infant requires immediate resuscitation. C. This infant is at high risk of developing cerebral palsy. D. A low Apgar score predicts neonatal death. E. A low Apgar score may be present in the absence of fetal acidosis or hypoxia.

201 C. APGAR score is not designed to predict neurologic outcome. Immaturity, adminstration of analgesics and sedatives to the mother, precipitous delivery are some of the causes of a falsely low Apgar score.

During measles outbreaks, at what earliest age can you vaccinate chilldren with the measles vaccine? A. At birth B. 2 months C. 4 months D. 6 months E. 8 months

220 D.

in what phase of kawasaki disease will have the greatest risk for coronary aneurysm formation? A. Acute febrile phase B. Subacute phase C. Convalescent phase D. Early recovery phase E. Risk is not defined by the phase

221 B. kawasaki disease is divided into three clinical phases. The acute febrile phase (1-2 wks) presents with myocarditis, fever and other classic signs of acute illness. Subacute phase - abatement of fever, associated with desquamationm thrombocytosis and development of coronary aneurysm. it is also in this stage that there is highest risk of sudden cardiac death. convalescent phase - when all clinical symptoms disappear up to the time there is normalization of ESR and CRP

An 8 y/o male child presented to the ER with fever, cough, colds, and photophobia. There was also presence of generalized maculopapular rash. Which of the following is true about his condition ? A. There may be an increaesed rate of pulmonary tuberculosis activation B. low serum retinol is associated with increased risk for pneumonia or encephalitis C. pathologic hallmark is the presence of warthin finkeldey giant cells in the respiratory tissue D. all of the above. E. none of the above.

230 D. patient is most likely suffering from measles infection. All of the choices are true.

Which of the following parameters is last to normalized during the clinical course of APSGN? A. Microscopic hematuria B. Low C3 C. Oliguria D. proteinuria E. ASO titier

232 A. oliguria -2 weeks, hypertension - 3 weeks, gross hematuria -3 weeks, low C3 - up to 8 weeks, proteinuria - 6 months,microscopic hematuria- >1 year.

the purpose of administering BCG vaccine is? A. Prevention of miliary TB B. Reduce the incidence of primary complex TB C. Increase herd immunity against PTB D. All of the above E. None of the above

240 A. BCG vaccination prevents the development of life threatening forms of TB disease ( meningeal, miliary, hepatic). It does not prevent one person from acquiring pulmonary TB or primary complex TB.

A 14 year old male patient was admitted due to dehydration, mother reported that the patient was urinating large volumes of urine and was said to be frequently thirsty and drinks a lot of water. Serum sodium was 128, plasma osmolality was 310, urine osmolality was only at 200mosm/kg of water, after administration of DDAVP, urine osmolality increased to 230mosm/kg, which of the following conditions does the patient most likely have? A. renal tubular acidosis B. nephrogenic diabetes insipidus C. central diabetes insipidus D. gittelman's syndrome E. none of the above

241 B. SIMILAR TO PREVIOUS BOARD EXAM CONCEPT/PRINCIPLE., unresponsive to DDAVP - nephrogenic DI

An 8 year old female patient was reported by her teacher to be constantly daydreaming, lacked focus and motivation, her academic performance had started to deteriorate for the past 3 months, mother also noted the patient to be occasionally day dreaming at home for regular short intervals and that she would not remember the things that were said or had happened a few moments before, which is the appropriate next step in evaluation of this patient? A. plain cranial CT scan B. plain cranial MRI C. serum electrolytes D. EEG E. sleep study

243 D. SIMILAR TO PREVIOUS BOARD EXAM CONCEPT/PRINCIPLE, EEG for evaluation of childhood non induced seizures about 3 questions came out

A 7 day old neonate was rushed to the ER because the mother noticed that the patient had difficulty breathing and was unable to breastfeed. On closer examination, patient was jaundiced, tachypneic with subcostal retractions, crackles all over bilateral lung bases, difficult to arouse, cold clammy skin, mottled in appearance, what is the appropriate course of action? A. give oral amoxicillin by dropper and supportive therapy B. give IV ampicillin + gentamicin and supportive therapy C. nebulize with salbutamol immediately and supportive therapy D. hydrate the infant immediately E. none of the above

251 B. SIMILAR TO PREVIOUS BOARD EXAM CONCEPT/PRINCIPLE, a case of sepsis, in selecting the best answer = IV antibiotics should be given

You are assessing a child's heart sounds. Which of the following will you most likely suspect as pathologic? A. Heard during fever, anxiety, or an infectious illness B. Vibrating and musical in quality C. Systolic murmur heard over the left lower sternum with a grade of 2/6 D. A soft, faintly heard diastolic murmur with a grade of 1/6 E. None of the above

262 D. An innocent physiologic murmur is grade 2/6 or less, systolic, soft, musical, and usually heard better during fever or anxious states

What is the most common cause of acute renal failure in children? A. Hemolytic uremic syndrome B. Post-streptococcal glomerulonephritis C. Minimal change disease D. Henoch Schonlein purpura E. SLE nephritis

263. A.Post-streptococcal glomerulonephritis and minimal change disease do not usually lead to renal failure. Most cases resolve

A newborn infant was delivered via elective CS at 38 weeks AOG, with an apgar score of 8,9. On feeding, he was observed to become cyanotic and would often suck in his lips. The cyanosis would be relieved once he begins to cry. Which of the following will establish the diagnosis in this case? A. CT scan B. Administering 100% oxygen C. Cranial MRI D. Inserting an NGT E. Rhinoscopy

286 D. Dx: Choanal atresia. Infants with bilateral choanal atresia who have difficulty with mouth breathing make vigorous attempts to inspire, often suck in their lips, and develop cyanosis. Distressed children then cry (which relieves the cyanosis) and become calmer, with normal skin color. Diagnosis is established by the inability to pass a firm catheter through each nostril 3â€"4 cm into the nasopharynx. The atretic plate may be seen directly with fiberoptic rhinoscopy. The anatomy is best evaluated by using high-resolution CT

Which of the following statements best describes a "physiologic" jaundice in the newborn? A. Direct bilirubin >10% of the total bilirubin B. Present in the first 24 hours of life C. Resolves on the 10th day of life D. Rate of rise of bilirubin is 6 mg/dL/24hr E. None of the above

292 C. Pathologic jaundice: (1) it appears in the 1st 24â€"36 hr of life, (2) serum bilirubin is rising at a rate faster than 5 mg/dL/24 hr, (3) serum bilirubin is >12 mg/dL in full-term infants (especially in the absence of risk factors) or 10â€"14 mg/dL in preterm infants, (4) jaundice persists after 10â€"14 days of life, or (5) direct- reacting bilirubin is >2 mg/dL at any time.

A 15 month old male presented with a high grade fever of 3 days, cough, coryza, conjunctivits and multiple erythematous maculopapular rashes appearing around the hairline and spreading downwards. The rashes are more confluent in the upper extremities and exhibit branny desquamation. Discrete red lesions with white spots are noted in the buccal mucosa at the level of the premolars. What is the most likely diagnosis for this case? A. Infectious Mononucleosis B. Varicella C. Roseola infantum D. Rubella E. Rubeola

293 E. The presence of koplik spots is highly pathognomonic for measles

Which of the following statements does not describe Chronic Granulomatous Disease (CGD)? A. It is characterized by the inability of neutrophils and macrophage to kill catalase- positive microorganisms B. The most common pathogen involved in recurrent infections is S. aureus C. Bacteremia and fungemia are more common than local infections D. For screening of CGD, the nitroblue tetrazolium (NBT) dye test is used E. Patients with CGD should be given daily oral trimethoprim-sulfamethoxazole for prophylaxis of infections

294 C. Pneumonia, lymphadenitis, osteomyelitis, and skin infections are the most common illnesses encountered in CGD. Bacteremia or fungemia occur but are much less common than focal infections

Delayed dental eruption is usually considered when there are no teeth noted at what age (in months)? A. 24 months B. 8 months C. 16 months D. 13 months E. 48 months

295 D. Delayed eruption is usually considered when there are no teeth by approximately 13 mo of age (mean + 3 standard deviations). Common causes include hypothyroid, hypoparathyroid, familial, and (the most common) idiopathic.

At what age does the apex beat of a child shift from the 4th left ICS MCL to the 5th left ICS MCL? A. 7 years old B. 10 years old C. 3 years old D. 15 years old E. None of the above

300 A. the apex beat is normally located at the 4th ICS MCL until 7 y/o when it shifts to the 5th ICS MCL

A 4 weeks old male manifests cyanosis and hepatomegaly. There is a grade 4/6 systolic ejection murmur without an audible ejection click. The ECG tall P waves and right ventricular hypertrophy. What is the most likely diagnosis A. Tetralogy of fallot B. pulmonic stenosis C. Truncus arteriosus D. tricuspid atresia E. TAPVR

317 B. Critical pulmonic stenosis often presents in the neonate.Cyanosis is due to elevated right-sided pressures and right-to-left shunting at the patent foramen ovale.

which of the following is not a characteristic of Fabry's disease? A. Mental retardation B. acroparesthesia C. Lenticular opacities D. Hypohidrosis E. None of the above

318 A. fabry disease is an x linked recessive disorder caused by deficiency in a-galactosidase A. the manifestation includes angiokeratoma, hypohidrosis, corneal and lenticular opacities, acroparesthesias and vascular diseases of the kidney,heart and brain. mental retardation is not a feature of this disease. Nelson 18th ed 598 ( SIMILAR TO PREVIOUS BOARD EXAM CONCEPT/PRINCIPLE)

The most frequent cause of food protein induced enteropathy in young infants is sensitivity to: A. Soy B. Egg C. Chicken D. Cow milk

344 D.

The first laboratory evidence of response to iron deficiency anemia treatment occurs in: A. 12-24 hours B. 36-48 hours C. 48-72 hours D. 4-30 days E. 1-3 months

331 C. 12-24 hours - subjective improvement 36-48 hours - initial bone marrow response 48-72 hours - reticulocytosis 4-30 days - increase in hemoglobin 1-3 months - repletion of stores NelsonTextbook of Pediatrics 18th edition p.2017

The most common hereditary bleeding disorder is: A. Glanzmann thrombasthenia B. von Willebrand factor deficiency C. hemophilia A D. hemophilia B E. Bernard Soulier syndrome

332 B. Nelson Textbook of Pediatrics 18th edition p.2071

True of methimazole intake in Graves Disease, except: A. Transient granulocytopenia should be taken seriously as it is a harbinger of agranulocytosis. B. The most severe reactions are hypersensitive. C. It is ten times more potent than PTU on weight basis. D. It has a longer serum T1/2 than PTU. E. It inhibits incorporation of trapped inorganic iodide into organic compounds.

333 A. Transient granulocytopenia is not a harbinger of agranuloctyosis in methimazole intake. E is also true of PTU. Nelson Textbook of Pediatrics 18th edition p.2335

What is the preferred drug in the management of infantile spasms? A. prednisone B. ACTH C. vigabatrin D. leviractam E. zonisamide

334 B. ACTH is the preferred drug in the management of infantile spasms. Prednison is equally effective. Vigabatrin is also effective for infantile spasms. Leviractam is an adjunct treatment for partial seizure; its mechanism of action is unknown. Zonisamide is also an adjunct treatment for partial seizures and in myoclonic syndromes with an unclear mechanism of action. Nelson Textbook of Pediatrics 18th edition p.2469

SMR stage that corresponds to the appearance of enlarged breasts and areola with no contour separation in girls? A. 1 B. 2 C. 3 D. 4 E. 5

335 C. 1 - preadolescent 2 - breast and papilla elevated as small mound 4 - areola and papilla form a secondary mound 5 - nipple projects, areola part of generalized breast contour

A 7 year old male with Dengue fever developed upper GI bleeding. He was then noted to have decreased urine output. On PE: BP 80/50 PR 120/min with cold, clammy skin and weak pulses. He was given several boluses of plasma expanders. This resulted in improved urine volume. His renal failure is classified as: A. Pre-renal B. Intrinsic renal C. Post-renal D. Initially pre-renal then post-renal

345 A. Improvement of urine output with fluid replacement points to a pre-renal cause of renal failure.

A 15 year old female developed low grade fever, malaise, sore throat, and anorexia. Irregular pink rashes were noted on the face, spreading to the trunk and extremities, with multiple large lymph nodes on the suboccipital, posterior auricular and anterior cervical areas. The most likely diagnosis is: A. Varicella B. Rubeola C. Rubella D. Dengue fever

346 C.

All of the following statements concerning allergic reaction to foods are true, except: A. Cow’s milk sensitivity is the most common cause of protein-induced enteropathy B. Gastrointestinal anaphylaxis is mediated by IgA C. Majority of children with positive results on skin prick tests to a food will not react when the food is ingested D. Elimination diets are the only means to establish the diagnosis of food allergies

357 B. It is mediated by IgE.

TRUE of simple febrile seizures EXCEPT A. generalized tonic-clonic B. lasts no more than 15 minutes C. more than 1 episode in 24 hours D. B and C E. All of the above

362 C. not recurrent within a 24-hour period Nelsons 19th pg 2017

Most common cause of hemolytic disease of the newborn A. ABO incompatibility B. Rh incompatibility C. Autoimmune D. Kell incompatibility E. None of the above

363 A. ABO incompatibility is the most common cause of hemolytic disease of the newborn. Nelsons 19th pg 619

A 1 day old neonate was brought to the hospital for frothing and bubbling at the mouth & nose. A few hours prior, the mother attempted to feed the baby which resulted to coughing and cyanosis. Which is the most common form of this disorder? A. EA with distal fistula B. EA with no fistula C. H type fistula with no EA D. EA with proximal and distal fistula E. EA with proximal fistula

364 A. Choices arranged from most common to least common with the following frequencies: 85%, 8%, 4%, 2%, 1%

An 16/M presents with recurrent muscle cramps and spasms. Serum chemistries were done revealing hypokalemia, hypomagnesemia and metabolic alkalosis. Urinary electrolyte analysis showed low urinary calcium levels and elevated urinary magnesium levels. What is your diagnosis? A. Gitelman syndrome B. Bartter syndrome C. RTA type IV D. RTA type I E. None of the above

374 A. Patients with Gitelman syndrome often have a history of recurrent muscle cramps and spasms, presumably caused by low serum magnesium levels. Biochemical abnormalities include hypokalemia, metabolic alkalosis, and hypomagnesemia. The urinary calcium level is usually very low (in contrast to the elevated urinary calcium level often seen in Bartter syndrome), and the urinary magnesium level is elevated. Nelsons 19th pg 1814

A 1/M presents with a 4 day history of low-grade fever, cough and colds. On PE you noted hyperresonance on percussion and wheezing on auscultation. The etiologic agent of the patient's illness is? A. Parainfluenza virus B. Respiratory syncitial virus C. Hemophilus influezae type B D. Streptococcus pneumoniae E. Mycoplasma sp.

377 B. This is a case of bronchiolitis which is an acute inflammation of the small airways in children < 2y Topnotch

Which of the following features is not consistent with absence seizures? A. 3-Hz spike waves on EEG B. No postictal state C. Impaired consciousness during attacks D. Typically lasts for less than 30 seconds E. Most common among children less than 5 years old

390 E. Absence seizures are rare in children below 5 years old.

Which of the following disorders is a classic example of pure T-cell deficiency? A. Chediak-Higashi syndrome B. Severe combined immunideficiency C. Wiskott-Aldrich syndrome D. Di George syndrome E. Chronic granulomatous disease

391 D.

Which of the following best describes proteinuria in the pediatric population? 2 A. > 4 mg/m /hr B. > 300 mg / day 2 C. > 40 mg/m /hr D. protein:creatinine ratio > 1.7 2 E. > 150 mg/m /hr

392 A. SIMILAR TO PREVIOUS BOARD EXAM CONCEPT/PRINCIPLE. Definitions of proteinuria in the pediatric population: > 4 mg/m2/hr OR > 150 mg/day OR protein:crea ratio > 0.2-0.3. Values of > 40 mg/m2/hr describes nephrotic range proteinuria.

A 5-year old female presented to your clinic due to sudden onset of fever with sore throat and vomiting. On physical examination, you noted tender vesicles and ulcers on her tonsils and oropharynx. Which of the following is the most likely causative agent for her disease? A. Epstein-Barr virus B. Coxsackievirus A C. Parvovirus B19 D. Corynebacterium diphtheriae E. Poliovirus

393 B. Diagnosis: Herpangina

Which of the following findings would you see during the subacute phase of Kawasaki disease? A. Periungual desquamation of the fingers and toes B. Elevated ESR C. Thrombocytosis D. Development of coronary artery aneurysm E. All of the above

394 E.

A 7-year old male presented to your clinic with symptoms of multiple raised erythematous lesion on both lower legs, abdominal pain and mild hematuria. On further history, you noted an episode of upper respiratory tract infection 1 week prior to the development of the said problems. What is your most likely diagnosis? A. Lupus nephritis B. Hemolytic uremic syndrome C. Henoch-Schonlein purpura D. Dengue hemorrhagic fever E. Immune thrombocytopenic purpura

395 C. SIMILAR TO PREVIOUS BOARD EXAM CONCEPT/PRINCIPLE.

A 3-year old boy was brought to the emergency department due to moderate cramping abdominal pain, accompanied by nausea and vomiting. On further history, you noted that he has had previous similar episodes in the past and has been asymptomatic in between episodes. There has been no febrile episodes. Which of the following conditions would be your primary diagnosis at this point? A. Acute appendicitis B. Intussusception C. Colonic volvulus D. Irritable bowel syndrome E. Celiac disease

396 B.

In a patient with ASD, which of the following would you least expect? A. Enlarged Left Atrium B. Systolic ejection murmur C. Widely split S2 D. Increased pulmonary venous return E. Right bundle branch block

411 A. An ASD usually does not result in LA enlargement due to the increased pulmonary venous return.

What is the most important risk factor that predisposes a newborn to sepsis? A. An HIV infected mother B. Chorioamnionitis C. Prematurity D. Severe combined immunodeficiency (SCID)

430 C.

A 3 year-old boy had fever and sore throat which uneventfully recovered spontaneously. A week later, he awakens with eyelid puffiness and scanty tea-colored urine. This is most likely: A. Nephrotic syndrome B. Acute pyelonephritis C. Acute glomerulonephritis D. Nephrolithiasis E. Acute cystitis

458 C. Acute postrep glomerulonephritis follows infection of the throat by nephritogenic strains of group A Beta- hemolytic streptococci. Clinical manifestations include gross hematuria, periorbital edema, hypertension and oliguria.

A newborn infant is noted to have pale blue extremities, HR of 150, grimaces to stimulation and is active with good cry. What is her APGAR score? A. 4 B. 6 C. 7 D. 8 E. 5

501 D. Appearance=pale, blue extremities=1; Pulse of 150=2; Grimace=1, Active=2; Respiration-good cry=2

Which of the following late manifestations describe Higouménaki sign of congenital syphillis? A. Unilateral or bilateral thickening of the sternoclavicular third of the clavicle B. Anterior bowing of the midportion of the tibia C. Peg-shaped upper central incisors with notch along the biting surface D. Linear scars that extend in a spoke-like pattern from previous mucocutaneous fissures of the mouth, anus, and genitalia E. none of the above

507 A. B. Anterior bowing of the midportion of the tibia- saber shins C. Peg-shaped upper central incisors with notch along the biting surface - Hutchinson's teeth D. Linear scars that extend in a spoke-like pattern from previous mucocutaneous fissures of the mouth, anus, and genitalia - Rhagades p. 1017 nelson's pediatric 19th ed

Deficiency of what vitamin leads to dermatitis presenting initially as erythema which then becomes dry, rough, thickened, cracked, and hyperpigmented. There is also diarrhea, disorientation and delirium. A. Riboflavin B. B1 C. Niacin D. B9 E. Pantothenic acid

509 C. this is a case of pellagra (3 D's= Dermatitis, diarrhea and delirium) caused by Vitamin b3 or niacin deficiency.

A child is already able to run well and walk up and down stairs, one step at a time. He is also able to put three words together in sentences. These milestones are expected for a child aged ___: A. 12 months B. 18 months C. 24 months D. 36 months E. 48 months

542 C.

A 17 year old female was brought to you by her mother because she has not yet had her menses. On PE, you note that the patient is below the 5th percentile for height. You also not ethat she has webbed neck and redundant nuchal skin. You suspect a chromosomal disroder and order karyotyping which would most likely reveal this karyotype: A. 47,XXY B. 45,XO C. 47, XXX D. 47,XYY E. 46,XY

550 B. Turner syndrome is a condition characterized by complete or partial monosomy of the X chromosome and defined by a combination of phenotypic features including redundant nuchal skin (in utero cytic hygroma), short stature and ovarian dysgenesis (causing primary amenorrhea).

A 7/F comes in to your clinic due to a rash. On examination, you note that the patient has bright red macules on his cheeks and a lacy, reticulated erythema over the extremities. Patient has no other symptoms or remarkable examination findings. What is the most likely diagnosis? A. Scarlet fever B. Rubella C. Roseola infantum D. Erythema infectiosum E. Measles

560 D. Erythema infectiosum is a childhood illness caused by infection with parvovirus B19. A "slapped cheek" appearance with a lacy eruption on the torso and extremities is the most common presentation.

The best single antibody titer to document cutaneous streptococcal infection in PSGN is: A. Deoxyribonuclease B antigen (DNase) B. ASO titer C. Streptozyme test D. Phadebact test

562 A. Anti-DNAse is the marker for PSGN due to streptococcal skin

Erythema toxicum is? A. More common among term than premature infants B. Usually associated with fever and a general toxic state C. Uncommon before the 5th day of life D. Usually associated with an elevated peripheral WBC count E. Manifested by a papulo-vesicular rash

652 A.

Hemolytic anemia in premature infants is being linked to lack or deficiency of A. Vitamin C B. Vitamin D C. Vitamin E D. Vitamin K E. Vitamin A

656 C. Hemolytic anemia= Vit. E. Hemorrhagic disease of the newborn=Vit. K.

A woman was noted to have a large volume of amniotic fluid at the time of her delivery of her child. At 6 hours of age, her baby begins regurgitating small amounts of mucus and bile- stained fluid. P.E. is normal. Abdominal x-ray obtained showed a “double-bubble†sign. This condition is most commonly associated with: A. Edward syndrome B. Patau syndrome C. Down syndrome D. Li-Fraumeni syndrome E. Fragile X syndrome

611 C. Down syndrome occurs in 20â€"30% of patients with duodenal atresia. Other congenital anomalies that are associated with duodenal atresia include malrotation (20%), esophageal atresia (10â€"20%), congenital heart disease (10â€"15%), and anorectal and renal anomalies (5%).

Which of the following is the most important procedure in the diagnosis of infective endocarditis? A. Complete blood count B. Echocardiography C. Erythrocyte sedimentation rate D. Blood culture E. Rheumatoid factor

617. D.The critical information for appropriate treatment of infective endocarditis is obtained from blood cultures. All other laboratory data are secondary in importance. (Nelson's, 11th ed)

A 3 year old boy is brought to the office by his parents who are concerned because he has hard, painful stools. For the past 4 months, their son defecates evry 3-4 days and cries during stooling. The resulting stool is very hard. PE of the child is nomal. Which of the ollowing is correct regarding his constipation? A. His constipation is unlikely to lead to encopresis B. A barium enema should be ordered to evaluate for Hirschprung's disease C. His constipation likely resulted from a traumatic triggering event, such as severe, painful diaper rash or painful diarrhea D. Abdominal radiographs should be ordered to evaluate for an underlying organic cause of his constipation E. The parents should be instructed to encourage their son to drink juice with each meal, and the boy should be reevaluated in 3-4 months

626 C. This patients constipation is most likely functional or non-organic. Functional constipation results from an inappropriate constriction of external anal sphincter . Most commonly, toddlers retain stool PURPOSELY because oftraumatic event, such as PAINFUL diaper rash, painful diarrhea, or even physical abuse.

Early diagnosis of cerebral palsy is important because it permits: A. Genetic counseling to prevent subsequent cases B. Treatment of underlying lesion and prevention of progression C. Guidance that may minimize or prevent secondary physical and emotional problems D. Prophylactic anticonvulsant treatment before onset of seizures E. Avoidance of unrealistic expectations

630 C.

A 5 year old female was brought to your clinic due to rashes on vulval area. Patient's mother noticed her to keep on scratching her pudendum especially at night. What is your differential diagnosis? A. pinworm B. Enterobius infection C. candidiasis D. A or B E. None of the above

660 D.

A newborn infant is noted to have acrocyanosis, HR of 90/min, weak passive tone, with active withdrawal and good cry. What is the apgar score? A. 5 B. 6 C. 7 D. 8 E. 9

661 C. The Apgar score is determined by evaluating the newborn baby on five simple criteria on a scale from zero to two, then summing up the five values thus obtained. The resulting Apgar score ranges from zero to 10. The five criteria are summarized using words chosen to form an acronym (Appearance, Pulse, Grimace, Activity, Respir ation).

Described as a lacy, reticulated vascular pattern over most of the body when the the baby is cooled? A. Milia B. Erythema toxicum C. Hemangioma D. Cutis marmorata E. Nevus sebaceous

662 D. When a newborn infant is exposed to low environmental temperatures, an evanescent, lacy, reticulated red and/or blue cutaneous vascular pattern appears over most of the body surface. This vascular change represents an accentuated physiologic vasomotor response that disappears with increasing age, although it is sometimes discernible even in older children.

Included in newborn screening in the Philippines, except? A. MSUD B. CAH C. PKU D. Galactosemia E. None of the above

663 E. Disorders screened in the Philippines includes the above plus G6PD Deficiency and congenital hypothyroidism. Newborn screening is ideally done on the 48th - 72nd hour of life. However, it may also be done after 24 hours from birth.

True of physiologuc jaundice, except? A. Appears on the 1st DOL B. Peak bilirubin 10 - 12 mg/dL (term) C. Rate of bilirubin rise < 5mg/dL D. Treatment is exposure to UV light E. None

664 A. Physiologic jaundice appears on the 2nd -3rd DOL and peaks at 5th DOL.

Type of leukemia most responsive to treatment? A. ALL B. AML C. CLL D. CML E. All of the above

675 A. Acute lymphoblastic leukemia (ALL) is a form of leukemia, or cancer of the white blood cells characterized by excess lymphoblasts. The earlier acute lymphocytic leukemia is detected, the more effective the treatment.

A newborn was brought to the ER for seizures. She was born at home and a sharpened bamboo stick was used to cut the umbilical cord. The mother never had prenatal check-up and her immunization status was unknown. Your diagnosis is Neonatal Tetanus. The drug of choice is most likely A. Penicillin B. Metronidazole C. Ceftriaxone D. Chloramphenicol

688 A. Answer: A. Penicillin Notes: Neonatal tetanus is an acute spastic paralystic illness caused by the neurotoxin (Tetanospasmin) produced by Clostridium tetani. In infants causes progressive difficulty in feeding, associated hunger and crying. In older children, causes trismus, sardonic smile. Treatment includes Human Tetanus Ig, PCN G (DOC), surgery.

A 19-yr old primi develops toxemia in her last trimester of pregnancy and during the course of her labor ,is treated with MgSO4. At 38 weeks AOG, she delivers a 2100gms infant with APGAR score of 1 at 1 min. And 5 at 5 mins. At 18 hrs of age, hematocrit was 79%, plt.ct. 100,000/uL, glucose 38 mg/dl, Mg 2.5 mEq/L, Ca 8.7 mg/dl. Soon after, infant has a generalized convulsion. The most likely cause is A. Polycythemia B. Hypoglycaemia C. Hypocalcemia D. Hypermagnesemia E. Thrombocytopenia

696 A. Answer: A An infant of 2100 gms is considered SGA. Pregnancy- induced hypertension can produce a dec.in uteroplacental blood flow and areas of placental infarction. This can result in fetal nutritional deprivation and intermittent fetal hypoxemia, w/ a dec.in glycogen storagemand relative erythrocytosis.

A 1-yr old boy presents w/ complaint from his parents of “not developing normallyâ€. He was a product of uneventful pregnancy and delivery, and reportedly normal at birth. His previous health care provider noted developmental delay, and child seemed to have enlarged spleen & liver. On P/E you notice that child has short stature, macrocephaly, hirsutism and coarse facies w/ decreased joint mobility which you suspect to be A. Beckwith-Wiedemann syndrome B. Crouzon syndrome C. Edwards syndrome D. Hurler syndrome

697 D. Answer: D Hurler syndrome other features: umbilical hernia, kyphoscoliosis, deafness, cloudy corneas and claw hand deformity.

A 15-yr old girl is seen in your clinic with a sprained ankle, which occurred the previous day while she was exercising in her room. You realize that you have not seen her for quite some time and begin to expand your P/E beyond her ankle. You find relatively minimal swelling on her reddened, irritated uvula. She seems to be somewhat hirsute on her arms and legs but has thinning of hair on her head, resting HR of 60 bpm, and her oral temp.is 96%F. Further questioning suggests that she has developed secondary amenorrhea. Your next step in management would be A. HIV testing B. Radiograph of ankle C. Thyroid function panel D. Comparison of current and past weights E. Pregnancy testing

698 D. Answer: D The patient could have any number of problems.However, s close look at her weight in comparison to previous ones is in order: she has all of the physical exam findings seen in bulimia.

Two weeks after a viral syndrome, a 2-yr old child develops bruising and generalized petechiae, more prominent over the legs. No hepatomegaly or LN enlargement is noted. P/E is unremarkable. Labs show normal Hgb, Hct, WBC Ct.and differential. Platelet count is 15,000/uL. Your impression is A. Von Willebrand disease B. Acute Leukemia C. Idiopathic thrombocytopenic purpura D. Aplastic anemia E. Thrombotic thrombocytopenic purpura

699 C. Answer: C ITP is the most common form of thrombocytopenic purpura. In most cases, it is preceeded by a viral infection. No diagnostic test solely identifies this disease; exclusion is necessary.

A 17-yr old is brought to the ER by the parents with complaint of coughing up blood. He is stabilized and his haemoglobin and hematocrit levels are in the safe range. During confinement, patient developed hematuria and proteinuria. You suspect patient to have A. Hemolytic uremic syndrome B. Goodpasture syndrome C. Nephrotic syndrome D. Post-streptococcal glomerulonephritis E. Renal vein thrombosis

700 B. Answer: B Goodpasture disease has both lung & renal involvement. Kidney biopsy and finding autoantibodies to glomerular BM.

8. A previously healthy male 3 year old child followed up in your clinic for official reading of TST which turned out to be 5mm. Which of the following is true? A. patient is immunosuppressed B. results is equivocal C. negative skin test D. positive skin test E. confirmation of history of exposure

88 C. Nelson's Textbook of Pediatrics 19th edition p.1002

A 3 year old female presents to your office with unilateral nasal discharge. The discharge is described by the caretaker to be malodorous and lately has been blood tinged. The most likely diagnosis is: A. Tertiary syphilis B. Unilateral choanal atresia C. Nasopharyngeal carcinoma D. Foreign body E. Angiosarcoma of the nose

9 D. Source: Nelson's Textbook of Pediatrics 19th edition p 1431

An 8 year old presented to the OPD of CVGH with multiple subcutaneous nodules. On physical examination, HR = 120, RR = 42, O2 sats = 89%, multiple palatal petechiae, pale palpebral conjunctivae, coarse crackles on left lung field, subcostal retractions, generalized lymphadenopathy. Patient was then transported immediately to ER and the following labs were taken: CBC = anemia with leukocytosis; PTT and PT were elevated. Patient had difficulty breathing and went into respiratory arrest. Upon intubation, pinkish frothy sputum was noted and IV sites were oozing with blood. Which of the following is not true? A. This is diagnosed through bone marrow biopsy B. A t(15;17) has favorable prognosis C. M2 subtype usually presents with DIC D. remission occurs in 85 - 90% with chemotherapy E. cell of origin is myeloblast

97 C. M3 subtype acute promyeloblastic leukemia presents with DIC due to granules present in the blast cells which trigger the coagulation cascade. Nelson's Textbook of Pediatrics 19th edition

Which phase of Kawasaki disease is associated with coronary aneurysms? A. Acute febrile phase B. Subacute phase C. Convalescent phase D. Phase of complications E. All of the above

1 B. Kawasaki disease presents with characteristically high, unremitting fever and 4 out of the 5 principal features: Bilateral nonexudative bulbar conjunctival injection with limbal sparing; Erythema of the oral and pharyngeal mucosa with strawberry tongue and dry, cracked lips; Edema and erythema of the hands and feet; Rash of various forms (scarlatiniform, maculopapular, erythema multiforme); Nonsuppurative cervical lymphadenopathy (usually unilateral, with node size >1.5cm). Acute febrile phase - fever and the acute signs of illness and usually lasts 1-2wks Subacute febrile phase - desquamation, thrombocytosis, coronary aneurysms and highest risk of sudden death usually lasts 2 wks Convalescent phase - All clinical signs have disappeared until ESR normalizes typically 6-8wks after onset on illness Source: Nelson's Textbook of Pediatrics 19th edition p864

Your last patient that day was a 7 year old female who was also brought for consult due to rashes. This was accompanied by cough, coryza and red eyes. Your initial impression should be? A. Roseola B. Rubeola C. Rubella D. Fifth disease E. Infectious mononucleosis

649 B.

The WHO and DOH both recommend the use of the following regimen in deworming children: A. Use of albendazole 200mg single dose every 6 month for ages 12 to 24 months B. Use of albendazole 400mg single dose every 6 month for ages 24 months and above C. use of mebendazole 500mg single dose every 6 months for ages 12 months and above D. A and C e. all of the above

170 E.

A 5 year old girl presenting with mental reatrdation associated with fair complexion is most likely due to: A. Maple syrup urine disease B. Phenylketonuria C. Galactosemia D. Methylmalonic aciduria E. Tay-Sachs disease

102 B. Phenylketonuria is an autosomal recessive disorder. Infants with classic PKU appear normal until they are a few months old. Without treatment, these children develop permanent intellectual disability. Seizures, delayed development, behavioral problems, and psychiatric disorders are also common. Untreated individuals may have a musty or mouse-like odor as a side effect of excess phenylalanine in the body. Page 34 of Topnotch Handout.

A 1 year old patient came in after 3 days of vomiting and diarrhea. Last urine output was 12 hours ago. After giving fluid bolus and hydrating for 4 hours, urine output was 20 ml. Serum creatinine was 1.2 mgs/dl and urine sp gr was 1.010. If intrinsic renal failure has set in, which finding will be consistent? A. Elevated urine osmolality B. Urine Na >30 C. FENa <1 D. BUN/Crea ratio >20 E. None of the above

103 B. Urine indices may be useful in differentiating prerenal from intrinsic ARF. Intrinsic renal: Specific gravity <1.010 Low urine osmolality High urine sodium >40mEq/L FeNa >2% Pre renal: Elevated specific gravity Elevated urine osmolality Low urine sodium <20 mEq/L Low FeNa <1%

Urine culture finding that best indicate a diagnosis of UTI: A. E.Coli 7,000 cfu/ml and S. epidermidis 10,000 cfu/ml by suprapubic aspiration B. E. Coli 30,000 cfu/ml by midstream catch C. Proteus 2,000 cfu/ml urine collected by cathetherization D. E.Coli 5,000 cfu/ml by suprapubic aspiration E. E.Coli 70,000 cfu/ml by urine bag collection

104 D. Urine culture and sensitivity testing is the gold standard for the diagnosis of UTI. Suprapubic count-any count of gram neg bacteria, Transurethral cath >50,000 cfu/ml of single organism Midstream sample >100,000 cfu/ml of single organism, if count is 10,000 cfu/ml but symptomatic, infection is likely.

A newly born female was noted to have jaundice, hepatomegaly, platelet count of 25,000/cu mm. Cranial ultrasound showed periventricular calcifications. What is the most likely diagnosis? A. Congenital Rubella B. Congenital Syphilis C. Toxoplasmosis D. VZV infection E. CMV infection

106 E. CMV infection is the most common congenital infection. Presents with chorioretinitis, microcephaly, and periventricular calcifications. Tx: is Ganciclovir Page 5 of Topnotch Handout

A newborn was noted to be cyanotic. Unfortunately the imaging facilities are unavailable for the day. A hyperoxia test was thus done which showed no improvement of PaO2 after administration of 100% O2. The next step in the management of this neonate is: A. Low tidal volume mechanical ventilation B. Positive pressure ventilation C. Digoxin D. Emergency laparotomy E. Prostaglandin E

11 E. The hyperoxia test is used to differentiate whether the etiology of cyanosis is cardiac or not. PaO2 does not improve in cardiac etiologies. In the absence of imaging for definitive diagnosis of the congenital heart disease, the clinician should not hesistate to administer prostaglandin in case the patient has a ductus dependent congenital heart disease. Prostaglandin prevents the ductus from closing. Source: Nelson's Textbook of Pediatrics 19th edition p1572

An 8 year old girl was brought in for declining academic performance. Her teacher reported that she is frequently inattentive in class. The doctor hyperventilates her in the office and she manifests with blank stares and clonic movements of the eyes, there was no post-ictal drowsiness noted. The whole event lasted a mere 10 seconds. What is the treatment of choice for this type of seizures? A. Valproic acid B. Phenobarbital C. Ethosuximide D. Diazepam E. None of the above

110 C. This is an absence seizure. The DOC is Ethosuximide.

In the management of benign febrile seizure, which of the following is advisable? A. Do a lumbar tap and CSF analysis to rule out CNS infection B. Give an anticonvulsant to prevent recurrence C. Education of the mother to manage the baby on next attack D. Do an EEG as this might ba a seizure disorder E. Do a CT/MRI to rule out a brain mass

111 C. SIMILAR TO PREVIOUS BOARD EXAM CONCEPT/PRINCIPLE. In benign febrile seizure the most important management is education of the mother. Lumbar tap, anticonvulsants, EEG and CT/MRI are not indicated.

The lab results of the patient in the above case showed: Hgb 142, Hct 0.48, WBC 2,000/mm3, lymphocytes 0.78, segmenters 0.22, platelet 180,000. Prologed PT. By this time you are considering a diagnosis of Dengue. What is the pathophysiological hallmark of this phase of illness she is in? A. Rapid increase in platelet count B. Fluid redistribution from the ECF to ICF C. Progressive drop in platelet count D. Increased capillary permeability with extravasation of fluids E. None of the above

113 D. SIMILAR TO PREVIOUS BOARD EXAM CONCEPT/PRINCIPLE. Patient is in the critical phase of illness and its pathophysiological hallmark is increased capillary permeability with extravasation of fluids.

A 13 y/o boy came in alert, afebrile with a swelling over the neck area. On examination, he was noted to have a grayish white membrane on the nostrils and pharynx which bled upon removal. Which statement is true of this disease? A. Specific treatment is penicillin for 5 days. B. The most serious late complication is tachycardia out of proportion to fever suggesting cardiomyopathy and autonomic dysfunction. C. Immunization is not required after treatment. D. The most serious immediate complication is septicemia. E. All of the above.

114 B. SIMILAR TO PREVIOUS BOARD EXAM CONCEPT/PRINCIPLE. This is a case of diptheric croup. Page 52 of Topnotch Handout. Tx is PenG IM or IV 100,000-150,000 U/Kg/day every 6 hours for 14 days. Immunization with DPT still should be given.

Which of the following is characteristic of complex febrile seizures: A. 1 episode in a 24 hour febrile period B. Duration <15 minutes C. Preferential gaze towards the left side D. Generalized tonic-clonic movements E. None of the above

115 C. SIMILAR TO PREVIOUS BOARD EXAM CONCEPT/PRINCIPLE. Page 48 of Topnotch Handout. Duration greater than 15 ins, repeated convulsions in 24 hours, focal seizure activity often manifested by a left preferential gaze.

A 10 y/o girl presents with fever, headache, malaise, she also refuses to eat and drink. On PE you find painful vesicular lesions on the cheeks and gingival area, halitosis, and submandibular lymphadenitis. What is the most probable etiologic agent? A. Coxsackie A16 B. Varicella Zoster virus C. HSV-1 D. HSV-2 E. EBV

116 C. SIMILAR TO PREVIOUS BOARD EXAM CONCEPT/PRINCIPLE. Page 60 of Topnotch Handout. Remember the difference between herpangina caused by coxsackie from herpetic gingivostomatitis caused by HSV1

A newly born infant has her right arm extended and her left arm flexed. If she has Erb-Duchenne palsy, which of the ff is true? A. The baby will have an absent Moro on the right side B. Grasp reflex will be absent on the right hand C. Miosis and anhydrosis may be present D. The left arm should be partially immobilized and properly positioned E. All of the above

117 A. SIMILAR TO PREVIOUS BOARD EXAM CONCEPT/PRINCIPLE. Page 3 of Topnotch Handout. Erb-duchenne palsy (C5-C6, upper trunk) affected arm is adducted, pronated and internally rotated. Hence the baby will have an absent on the affected arm.

An LGA infant was born to an insulin dependent diabetic mother. He presented with jitteriness, hypotonia, temperature instability, seizures and lethargy after 24 hours what is the most likely cause? A. Hypocalcemia B. Hyperkalemia C. Hyponatremia D. Hypoglycemia E. Hyperglycemia

118 D. SIMILAR TO PREVIOUS BOARD EXAM CONCEPT/PRINCIPLE. These infants may have periods of low blood sugar (hypoglycemia) shortly after birth because of increased insulin level in their blood due to compensation for the high blood sugar levels of the mother.

A baby whose antenatal ultrasound showed renal agenesis is prone to this type of congenital anomaly: A. Deformation B. Dysplasia C. Disruption D. Malformation E. Syndrome

119 A. SIMILAR TO PREVIOUS BOARD EXAM CONCEPT/PRINCIPLE. Due to the renal agenesis there would be oligohydramnios causing deformation.

A patient was diagnosed with Tetralogy of Fallot. His chest x-ray reveals the classic Le Couer en Sabot. Which of the following components of TOF is responsible for this shape? A. Pulmonary stenosis B. VSD C. RVH D. Overriding of the aorta E. Right sided aorta

12 C. The typical configuration on AP view consists of a narrow base, concavity of the left heart border in the area usually occupied by the pulmonary artery and normal overall heart size. The hypertrophied right ventricle causes the rounded apical shadow to be uptilted so that it is situated higher above the diaphragm than normal and pointing horizontally to the left of the chest wall. The cardiac shadow has been likened to that of a boot or a wooden shoe. Source: Nelson's Textbook of Pediatrics 19th edition p1575

A pediatric patient arrives at your clinic for a well- baby check up. You observed that he can walk with one-held, and plays a ball. What is the most likely age of the child? A. 8 months B. 10 months C. 1 year D. 2 years E. 3 years

121 C. A one year old child can walk with one hand held, rises independenly, says a few words besides "mama" and "dada", plays simple ball game, and makes postural adjustment to dressing. (Nelson's). SIMILAR TO PREVIOUS BOARD EXAM CONCEPT.

Which of the following will show an increase in pulmonary circulation? A. Left-to-right shunt disorders B. Right-to-left shunt disorders C. Hypoplastic right ventricle D. Coarctation of aorta E. All except B

122 A. The lesions that cause left-to-right shunting (ASD, VSD, PDA) resuls in shunting of fully oxygenated blood back into the lungs. The resulting increase volume of blood in the lungs decreases pulmonary compliance and increases the work of breathing. The chronic exposure of the pulmonary circulation to high pressure and blood flow results in a gradual increase in pulmonary vascular resistance (Eisenmenger syndrome) (Source: Nelson's) SIMILAR TO PREVIOUS BOARD EXAM CONCEPT.

A 12-year old female presented with a easy fatigability. On physical examination, there is a blowing holosystolic murmur heard at the left sternal border. The most likely diagnosis is: A. Tetralogy of Fallot B. Mitral regurgitation C. Ventricular septal defect D. Aortic insufficiency E. Pulmonary stenosis

123 C. A harsh, blowing holosystolic murmur heard at the left sternal border is characteristic of VSD.

A 2-year old male presented with frequent coughing and wheezing. Radiologic finding showed a collapsed trachea. The findings are suggestive of: A. Bronchiolitis B. Bronchial asthma C. Cystic fibrosis D. Tracheomalacia E. Bronchopulmonary dysplasia

124 D. Tracheomalacia is a congenital tracheal obstruction characterized by complete absence of cartilaginous rings. They presented with wheezing, coughing which generally increases with activity and colds. They usually maintain a normal oxygenation and disposition and grow and and develop normally, despite the chronic airway noises. SIMILAR TO PREVIOUS BOARD EXAM CONCEPT

A 6 year-old male presented with low grade fever, colds, flushed cheeks, and diffuse macular rashes of the trunk and extremities, with notable sparing of the palms and soles. His condition is most likely caused by: A. Varicella zoster virus B. Rubeola C. Rubella D. HHV-6 E. Parvovirus

125 E. This is a case of Erythema Infectiosus caused by Parvovirus B19, most prevalent in ages 5-15 years old. The hallmark sign is erythamatous facial flushing (slapped cheek appearance) and rashes that spreads rapidly to the trunk and proximal extremities as a diffuse macular erythema. SIMILAR TO PREVIOUS BOARD EXAM CONCEPT

An infant born to a mother with active varicella at delivery should immediately receive: A. VZ Ig B. Acyclovir C. Active vaccine against varicella D. High dose of Vitamin A E. MMR

126 A. Newborns whose mothers develop varicella 5 days before to 2 days after delivery should receive 1 vial of varicella-zoster immune globulin (VariZIG). Dosage: 125 units/10 kg IM given within 96 hrs after exposure. Although neonatal varicella may occur in about half of these infants despite administration of VariZIG, it is usually mild. Perinatally acquired varicella may be life threatening, the infant should be treated with acyclovir (10 mg/kg every 8 hr IV) when lesions develop. (Nelson's). SIMILAR TO PREVIOUS BOARD EXAM CONCEPT

A child was delivered preterm via emergency CS to a 35 year-old primigravid because of fetal distress. At the first 5 minute of life, he was noted to have a heart rate of 90, cyanotic on the extremities, irregular respiration, weak tone, and grimaces with suction. This indicates/predicts A. Low umbilicard cord pH B. A need for rescucitation C. Presence of cerebral palsy D. A poor neurologic outcome E. Neonatal death

127 B. APGAR score=5; 8-10 good cardiopulmonary adaptation, 4-7 need for rescucitation esp. ventilatory support; 0-3 immediate rescucitation. The APGAR score helps to rapidly assess th need to rescucitate neonates after birth. SIMILAR TO PREVIOUS BOARD EXAM CONCEPT.

An infant has a hemangioma that grows rapidly in size, leading to thrombocytopenia and microangiopathic hemolytic anemia. Which term describes this condition? A. Kaposi like form of infantile hemangioma B. McCune Albright syndrome C. Kasabach-Merritt syndrome D. Maffucci syndrome E. Evan's syndrome

14 C. McCune Albright syndrome - polyostotic fibrous dysplasia, cafe-au-lait skin pigmentation, autonomous endocrine hyperfunction Mafucci syndrome - Multiple enchondromas and hemangiomas Evan's syndrome - autoimmune anemia + thrombocytopenia Source: Nelson’s Textbook of Pediatrics 19th edition p1719

A 7 year-old boy arrives at the emergency department due to hematuria and scanty urine output for the past 24 hours. Ten days prior to consult, he had fever and throat pain but did not seek consult. Which of the following will be the most important in his diagnosis. A. ASO 180 B. elevated ESR C. Decreased C3 D. (+) RBC in the urine E. Diminished renal function

128 A. This is a case of PSGN. Confirmation of the diagnosis of Post-streptococcus glomerulonephritis requires clear evidence of invasive streptococcal infection. A rising antibody titer to streptococcal antigen confirms a recent streptococcal infection. Importantly, antistreptolysin O titer is commonly elevated after a pharyngeal infection. (Nelson's). ASO normal level (0- 100 IU/mL). Although decreased C3 is also diagnostic of PSGN, other conditions may also present with depressed (MPGN, Lupus nephritis). SIMILAR TO PREVIOUS BOARD EXAM CONCEPT.

Which of the following correctly describes proteinuria? A. Urine protein/crea ratio of +0.15 B. Urine protein of 0.15 g/24 hrs C. Urine protein of 120 mg/24 hrs D. Urine protein of 20 mg/m2/hr E. None of the above.

129 D. Upper limit/Normal protein: Urine protein/creatinine ratio (Upr/Ucr) <0.5 in children younger than 2 yrs and <0.2 in children 2 yrs or older (A ratio >3 suggest nephrotic range proteinuria); 150 mg/24 hr or 0.15g/24 hr; Normal protein excretion: <4 mg/m2/hr (ABNORMAL 4-40 mg/m2/hr and NEPHROTIC RANGE is defined as >40 mg/m2/hr) (SOURCE:NELSON'S) SIMILAR TO PREVIOUS BOARD EXAM CONCEPT.

A 5 year old presents with multiple petechiae on her lower extremities and oral-mucosal bleeding of 3 days duration. 2 weeks prior, she had a mild respiratory tract infection but other than that, her caretakers describe her to be active and generally fine. On PE, she is afebrile. No lymphadenopathy nor hepatosplenomegaly is noted. Which is the best next diagnostic step to confirm the diagnosis? A. CBC with platelet count B. PT C. PTT D. Bleeding time E. Clotting time

13 A. CBC with PC would reveal decreased platelets and normal other parameters suggesting that this is a case of ITP (idiopathic thrombocytopenic purpura). Source: Nelson's Textbook of Pediatrics 19th edition p1715

A 12-year old female presented with short stature, voracious appetite, and delayed sexual maturity. You would suspect: A. Hypopituitarism B. Hypothyroidism C. Constitutional short stature D. Psychosocial dwarfism E. Growth hormone insensitivity

132 D. Emotional deprivation is an important cause of retardation of growth and mimics hypopituitarism. The condition is known as psychosocial dwarfism, maternal deprivation dwarfiss, or hyperphagic short stature. Emotionally deprived children frquently have perverted or voracious appetites, enuresis, encopresis, insomnia, crying spasms, and sudden tantrums. (Nelson's). Deprivation dwarfism is described as a triad of extreme short stature, voracious appetite, and markedly delayed sexual maturation. SIMILAR TO PREVIOUS BOARD EXAM CONCEPT.

A 7-year old female patient had recent history of cough, colds, and fever which lasted for 1 week. However, after a few days, patient presented with recurrrence of cough, now with high fever, extreme dyspnea and tachypnea. Physical findings revealed inspiratory crackles with decreased breath sound in the right lower lung field. Chest radiograph revealed pneumatocele. Her condition now is most likely caused by: A. Pseudomonas aeruginosa B. Streptococcus pneumoniae C. Mycoplasma pneumoniae D. Staphylococcus aureus E. Haemophilus influenzae

133 D. Pneumonia due to S. aureus may be primary (hematogenous) or secondary after a viral infection such as influenza. More common are high fever, abdominal pain, tachypnea, dyspnea, and localized or diffuse bronchopneumonia or lobar disease. S. aureus often causes a necrotizing pneumonitis; empyema, PNEUMATOCOELE pyopneumothorax, and bronchopleural fistulas develop frequently. Staphylococcal pneumonia occasionally produces a diffuse interstitial disease characterized by extreme dyspnea, tachypnea, and cyanosis. (Nelson's). SIMILAR TO PREVIOUS BOARD EXAM CONCEPT.

The empiric treatment for neonatal sepsis consists of? A. Cloxacillin + Aminoglycoside B. Ampicillin + Sulfamethoxazole C. Aminoglycoside + Amplicillin D. Erythromycin + Cloxacillin E. Imipinem + Cilastatin

134 C. SIMILAR TO PREVIOUS BOARD EXAM CONCEPT.

A 17-year old female presented with itching and pain on the genital area. On examination, she had vesicular lesions over the genital area. Based on the presentation, this is most likely caused by: A. HSV-1 B. HSV-2 C. VZV D. Neisseria gonorrhea E. Chlamydia trachomatis

136 B. Herpes simplex virus-2 is the cause of genital herpes. Most primary genital herpes simplex virus infections are asymptomatic. Lesions evolve from vesicles to pustules to wet ulcers and heal by crusting. Lesions occur on the labia, mons pubis, vaginal mucosa, cervix and on the shaft of the penis. Inguinal lymphadenopathy, dysuria, and vaginal discharge are frequent complaints. SIMILAR TO PREVIOUS BOARD EXAM CONCEPT.

A 16 -year-old male presented with painful lesion in the inguinal node with "groove sign". This is a case of: A. Lymphogranuloma venereum B. Donovanosis C. Chancroid D. Herpes Genitalis E. Early syphilis

137 A. Lymphogranuloma is a sexually transmitted disease caused by Chlamydia trachomatis. Inguinal syndrome occurs 2-6 weeks after exposure, and presents with large circular lymph nodes (buboes), which are painful, tender inguinal or femoral lymph adenopthy. SIMILAR TO PREVIOUS BOARD EXAM CONCEPT.

Which of the following sign/s point to a primary immunodeficiency problem in children? A. More than 2 pneumonias in a year B. More than 4 new ear infections in 1 year C. Two or more serious sinus infections within 1 year. D. Sepsis 3x a year E. All of the above

138 E. 10 warning signs of primary immunodeficiency: 4 or more new ear infections within 1 year, 2 or more serious sinus infections within 1 year; 2 or more months on antibiotics with little effect; 2 or more pneumonias within 1 year; Failure of an infant to gain weight or grow normally; recurrent, deep skin or organ abscess; persistent thrush in mouth or fungal infection on skin; need for IV antibiotics to clear infections; 2 or more deep seated infections including septicemia; a family history of primary immunodeficiency. SIMILAR QUESTION IN THE BOARDS.

A 10 year-old patient presented with retro-orbital pain, nausea, vomiting, abdominal pain, and 4-day history of high grade fever. Which of the following diagnostic test will you request first? A. Typhidot B. Dengue DNS-1 antigen C. Dengue IgG D. Complete blood count E. LeptoMAT

139 D. Complete blood count will be helpful in determining presence/absence of thrombocytopenia and hemoconcentration as these are part of clinical diagnosis of Dengue fever. Note: 4-5 stem questions were asked about Dengue in the Feb 2015 boards.

A 4 week old female was brought to your clinic with parental reports of increasing jaundice over the last week. Her parents reported that 2 weeks ago, she began to have yellowing of her eyes with subsequent yellowing of her skin. Her stools have been pale in color for the past 10 days. On history, she was born by spontaneous vaginal delivery to a G2P1 withno complicated noted at birth or at discharge. The patient was not jaundiced at discharge. What is your primary impression? A. Biliary atresia B. Choledochal cyst C. Breastfeeding jaundice D. Duodenal atresia E. Breastmilk jaundice

140 A. Biliary atresia is characterized by progressive fibrosis and obliteration of extra- and intra-hepatic bile ducts. Usually, patient is born at term with normal birth weight. Jaundice can be present at birth or it be present as late as 3-5 weeks of life. Acholic stool is another common complaints, highly suggestive of cholestasis. On physical exam, patient will have an enlarged liver. Lab exams will show an elevated TB, increased DB, normal or elevated aminotransferases, and highly elevated Alk phos. Choledochal cyst may present with intermittent jaundice. Symptoms of duodenal atresia may be present right after birth. BF jaundice and breastmilk jaundice are less likely to cause progressive jaundice. Note: SIMILAR TO PREVIOUS BOARD EXAM CONCEPT

After the seizure episode, the mother asked about his son's condition, you tell her that benign febrile seizures are A. A serious illness that should be seen by a neurologist for prescription of maintenance medications B. A benign episode and should tell her that she needs to give the child antipyretics and keep him cool during febrile episodes C. An isolated case that will never happen again D. A sign of serious CNS infection E. None of the above

142 B. Benign febrile seizures are benign episodes of seizures related to elevated body temperature, therefore educating the mother about the use of antipyretics and cooling will keep her calm during such attacks.

SAO, a 4 year old boy was brought to the ER due to bleeding from the gums, CBC was done revealing WBC of 35,000, ANC was 4,200, BM aspiration showed >25% of BM cells are lymphoblasts, what is the treatment? A. Chemotherapy B. Radiation therapy C. All trans-retinoic acid D. Plasmapheresis E. None of the above

143 A. The case is ALL, the treatment is remission induction with vincristine, prednisone and L-aspariginase.

MO, a 7 year old male was brought by her mother due to passage of cola like urine, upon careful history, the patient had a skin infection 1 month prior. What is the diagnostic test of choice to document the antibody titer of the skin infection. A. Serum C3 B. ASO titer C. DNAse B D. Serum IgA E. None of the above

144 E. Anti-DNAse B is the antibody titer test of choice for documenting antibody titers of streptococcal skin infection, ASO is for throat infection.

The vaccine BCG is made up of what A. Purified protein derivative of Mycobacterium tuberculosis B. Live attenuated form of Mycobacterium tuberculosis C. Killed Mycobacterium tuberculosis D. Antigenic parts of Mycobacterium tuberculosis E. Attenuated live Mycobacterium bovis

146 E. BCG is a live attenuated form of Mycobacterium bovis that is grown on the laboratory and has similar antigenic properties as Mycobacterium tuberculosis

A child was brought to your clinic for a wellness check up, you noted that the child walks held on one hand by her mother, who knows mama and dada. You know that the approximate age of this child is? A. 10 months B. 12 months C. 24 months D. 48 months E. 36 months

147 B. At this age, the patient says dada/mama, the child's first words, is assisted to walk but still can't walk independently which is cruising.

KO, a 5 year old child was brought to your clinic because of fever, you noted the presence of fever at 37.8C and swollen posterior auricular nodes without colds or photophobia, there were maculopapular rashes all over the body, you know that the most likely causative agent is: A. Paramyxoviridae B. Togaviridae C. Human herpes virus 6 D. Herpes zoster E. Coxsackie virus

148 B. This is a case of german measles and the causative agent is a RNA virus from the Togaviridae family

A 3 year old child was brought to your clinic due to strong body odor noted by the mother, no newborn screening was done. Upon physical exam, you noted that the child had strong musty odor. You know that the defective enzyme in this patient is? A. Phenylethanolamine N-methyltransferase B. Dopamine B-hydroxylase C. Tyrosine hydroxylase D. Phenylalanine hydroxylase E. None of the above

149 D. This is a case of PKU, the enzyme defective in this case is Phenylalanine hydroxylase causing buildup of phenylalanine. This causes the formation of phenylacetate, phenylpyruvate and phenethylamine which causes the characteristic musty body odor.

A previously healthy 8 year old male presents with progressive painless proptosis and decreased visual acuity of the left eye during the past 10 weeks. The most likely diagnosis is: A. Orbital cellulitis B. Trichinosis C. Retinoblastoma D. Rhabdomyosarcoma E. None of the above

15 D. Rhabdomyosarcoma is the most common sarcoma of childhood. Orbital rhabdomyosarcoma is a common site for rhadomyosarcoma which produces local signs as it grows and displaces normal tissues. Retinoblastoma may be endophytic or exophytic from the retina. They present with leukocoria or strabismus. Source: Nelson's Textbook of Pediatrics 19th edition p1761, 1768

SO, a 15 year old male came into your clinic due to joint pains, you noted that the joint pains occur at the lower back, hips, knees and left ankle, x-ray of the lumbar spine shows sacroiliitis, labs showed slight elevation of ESR and (+) HLA B27. What is the most likely diagnosis? A. Juvenile Ankylosing Spondylitis (Juvenile Spondyloarthropathy) B. Juvenile Rheumatic Arthritis (Juvenile Idiopathic Arthritis) C. Systemic Lupus Erythrematosus D. Juvenile Dermatomyositis E. None of the above

150 A. This is a case of juvenile AS, supported by oligoarthritis, (+) x-ray of sacroiliitis and (+) HLA B27

FS, a 5 year old female was brought by her mother due to reddish purpuric lesions on her legs, upon PE, heart sounds were unremarkable, you noted that the purpura is palpable and mostly located on the legs and there is also knee pain, she also tested positive for HLA-DRB1*07 you noted that prior to the rashes' appearance, she had a bout of URTI, as her pediatrician, you know that she most likely has: A. Dengue Hemorrhagic Fever B. Kawasaki disease C. Henoch-Schonlein purpura D. Thrombotic Thrombocytopenic purpura E. None of the above

151 C. This is a case of Henoch-Schonlein Purpura. The characteristic of the rash as well as a prior bout of URTI and HLA DRB1*07 (+) is suggestive of this case.

A newborn was born to a mother known to have SLE, PE findings include annular or macular rash typically affecting the face especially the periorbital area. The most dreaded complication from this disease is: A. Lupus nephritis B. Congenital heart block C. Serositis D. Arthritis E. None of the above

152 B. This is a case of neonatal lupus and the most dreaded complication from this is congenital heart block due to Ro and La antigens that may be exposed on the surface of cardiac cells in the proximity of the atrioventricular node, thus making these antigens accessible to maternal autoantibodies. Binding incites a local immune response, resulting in fibrosis within the conduction system. Lupus nephritis is seen in older children and adults who manifest lupus later in life. (Nelson 19th ed., pp 845-846)

Which of the following is true regarding MHC proteins A. Are involved in marking body cells as being “self†B. Can be found on virtually all cells (Type I) C. Can be found on antigen-presenting cells (Type II) D. Both A and B E. All of the above

154 E. All of the options are properties of MHC proteins

Proteins on the surface of mitral valve cells resemble proteins on the surface of the bacterium streptococcus pyogenes. This: A. Is beneficial because it will help increase the body’s anti-bacterial defenses. B. Can result in the production of antibodies that target mitral valve cells C. Is an example of cell-mediated immunity D. Is an example of passive artificial humoral immunity E. None of the above

155 B. This is an example of molecular mimicry as the M protein of strep pyogenes causes rheumatic heart disease.

The only antibody that could attack the Rh complexes on fetal red blood cells is the: A. IgA B. IgD C. IgE D. IgG E. IgM

156 D. The Rh blood group in the fetus would be attacked by Rh IgG from the mother.

Which of the following is a possible result of activated complement A. Formation of a membrane attack complex B. Lysis of bacterial cells C. Opsonization D. Inflammation E. All of the above

157 E. All of the following are possible consequences of activated complement

The anterior pituitary gland is an embryologic remnant of what germ layer? A. Endoderm B. Mesoderm C. Ectoderm D. Yolk sac E. None of the above

158 C. The anterior pituitary gland is an invagination of the ectoderm of the mouth.

The superior parathyroid glands are derived from which pharyngeal pouch A. 1st B. 2nd C. 3rd D. 4th E. 5th

159 D. The superior parathyroid and the C cells of the thyroid develop from the 4th pouch while the inferior parathyroid and the thymus develop from the 3rd pouch

A 4 year old male experienced an upper respiratory tract infection that was followed in 2 weeks by generalized edema. His blood pressure is normal. Urinalysis reveals 2-5 rbc/hpf and 4+ protein. His BUN is 19mg/dl. Serum albumin 0.9g/dl and C3 was 93mg/dl. The most likely diagnosis is: A. Post streptococcal glomerulonephritis B. Membranous glomerulonephritis C. Minimal change disease D. Focal sclerosis E. IgA nephropathy

16 C. Patient is presenting with nephrotic syndrome. The most common of which is minimal change disease in children. Hypoalbuminemia, proteinuria, edema and hyperlipidemia constitute the nephrotic syndrome. Source: Nelson's Textbook of Pediatrics 19th edition p1804

Which of the following is not true regarding Immunoglobulins A. IgA is a dimer often found in the same secretions as lysozyme B. IgG can be involved in hemolytic disease of the newborn C. IgM and IgG can activate complement D. IgD can be found acting as a receptor protein on the B cell plasma membrane E. None of the above

160 E. All of the following statements about immunoglobulins are true.

Aspirin is usually contraindicated in children because of the risk of developing Reye's syndrome. However, there are certain diseases in which the benefit outweighs the risk. Which of among the following diseases can you use ASA? A. Kawasaki B. Rheumatic Fever C. JRA D. A and B E. all of the above

161 E. these 3 diseases are the only indication for giving ASA in children

A premature newborn was just delivered vaginally. You noticed that his body is pink but the extremities are blue, HR of 115, limp, with good cough but slow and irregular respiration. What is the APGAR score of the baby? A. 4 B. 5 C. 6 D. 7 E. 8

163 C. Color 1, HR 2, irritability 2, activity 0, respiration 1. body pink but extremities are blue- 1; HR 115- 2; limp- 0; good cough- 2; slow and irregular respiration- 1... A good coughing reflex can also mean that the patient has good irritability

It is a major criteria in the diagnosis of Rheumatic Fever that can stand alone and does need ASO titer. A. Migratory arthritis B. Myocarditis C. Subcutaneous nodules D. Erythema marginatum E. Sydenham's chorea

164 E. SIMILAR TO PREVIOUS BOARD EXAM CONCEPT/PRINCIPLE

Definitive diagnosis of infective endocarditis is give if it fulfills: A. 2 major criteria B. One major and 3 minor criteria C. 5 minor criteria D. A and C E. All of the above

165 E.

A 14 year old patient came in at your clinic for check up. He told you that he has not had chicken pox before and he wants to get vaccinated. How will you give the vaccine? A. 1 dose only B. 2 doses with at least 4 weeks apart C. 2 doses with at least 6 months apart D. 3 doses with at least 4 weeks apart E. 3 doses with at least 6 weeks apart

166 B. for individual >13 years old and without immunity, they should receive 2 doses of the vaccine with at least 4 weeks apart.

The last dose of the Rotavirus vaccination should NOT be administered to infants older than how many weeks? A. 24 B. 26 C. 28 D. 30 E. 32

167 E.

Measles vaccine may be given as early as how many months specially in cases of epidemics? A. 5 B. 6 C. 7 D. 8 E. 9

168 B.

in the 2014 guidelines of PPS, all of the following are considered part of the EPI except: A. MMR B. measles C. Rotavirus vaccine D. Infuenza E. Td

169 D. EPI includes BCG, DTwP-Hib-Hepatitis B, OPV, measles,MMR, rotavirus, PCV, and Td

The following listed phrases are characteristics of simple partial seizures EXCEPT: A. Loss of consciousness B. Duration of 10-20 sec C. Versive seizures D. May have secondary generalization E. Abnormal EEG

17 A. There is never a loss of consciousness in simple partial seizures. Some patients may actually be conversant and talk to you during the event. Source: Nelson's Textbook of Pediatrics 19th edition p2021

The following factors indicate worse prognosis in a patient with Acute Lymphoblastic Leukemia EXCEPT: A. Age under 2 B. Presentation in adolescence C. Peripheral blood blast greater than 100,000 D. Presence of Philadelphia chromosome or t(9;22) E. translocation of chromosome 12 and 21 or t(12;21)

174 E. t(12:21) usually indicates better prognosis. the presence of t(9:22) in ALL means that it has also a CML property which makes the case worse. Robbins 8th ed page 603. Factors associated with worse prognosis are (1) age under 2; (2) presentation in adolescence or adulthood; (3) peripheral blood blast counts greater than 100000; (4) the presence of Philadelphia chromosome. Favorable prognostic markers include (1) an age of 2 to 10; (2) a low white cell count; (3) hyperploidy; (4) trisomy of chromosomes 4, 7, and 10, and; (5) the presence of a t(12:21)

a 15 year old male came in to your clinic complaining of deep knee pain which often cause night time awakening. X ray was done and revealed a sunburst pattern. The most likely diagnosis of the patient is: A. Ewing sarcoma B. osteosarcoma C. Rhabdomyosarcoma D. osteochondroma E. chondroblastoma

175. B.SIMILAR TO PREVIOUS BOARD EXAM CONCEPT/PRINCIPLE

Bird's beak sign, inverted U sign and coffee bean sign are seen in abdominal Xray with barium in what disease? A. Pyloric stenosis B. Duodenal atresia C. intussusception D. Volvulus E. Hirschprung's disease

178 D.

Pincer grasp is usually noted at what age? A. 2 mos B. 4 mos C. 6 mos D. 8 mos E. 10 mos

18 D. Pincer grasp/thumb finger grasp Source: Nelson's Textbook of Pediatrics 19th edition p27

The murmur heard in Atrial septal defect originates from: A. Pulmonary valve B. Tricuspid valve C. ASD D. Aortic valve E. Mitral valve

180 A. due to increased blood flow passing through this normal sized valve producing a relative stenosis of pulmonary valve----> systolic ejection murmur at the 2nd left ICS. Blood flow across the ASD does not cause a murmur at the site of the shunt because no substantial pressure gradient exists between the atria. However, ASD with moderate-to-large left-to-right shunts result in increased right ventricular stroke volume across the pulmonary outflow tract creating a crescendo-decrescendo systolic ejection murmur. This murmur is heard in the second intercostal space at the upper left sternal border.

The 1st teeth to erupt at 6 months are the A. 2nd upper molars B. Upper cental incisors C. Lower central incisors D. Upper canines E. Lower canines

181 C. SIMILAR TO PREVIOUS BOARD EXAM CONCEPT/PRINCIPLE.. 1st teeth to erupt at 6 to 6 1/2 months are the lower central incisors

Steroids are used in the management of Rheumatic Fever if which of the following develops? A. carditis B. chorea C. arthritis D. erythema marginatum E. subcutaneous nodules

182 A. SIMILAR TO PREVIOUS BOARD EXAM CONCEPT/PRINCIPLE.. Patients with carditis and cardiomegaly or congestive heart failure should receive corticosteroids.

A patient with evidence of previous Streptococcal throat infection is diagnosed with Rheumatic fever if there is which among the following? A. 0 major, 3 minor Jones criteria B. 1 major, 1 minor Jones criteria C. chorea D. 0 major, 4 minor criteria E. None of the above

183 C. SIMILAR TO PREVIOUS BOARD EXAM CONCEPT/PRINCIPLE.. The diagnosis of acute rheumatic fever can be established by the Jones criteria when a patient fulfills 2 major criteria or 1 major and 2 minor criteria and meets the absolute requirement. There are 3 circumstances in which the diagnosis of acute rheumatic fever can be made without strict adherence to the Jones criteria. Chorea may occur as the only manifestation of acute rheumatic fever. Similarly, indolent carditis may be the only manifestation in patients who 1st come to medical attention months after the onset of acute rheumatic fever. Finally, although most patients with recurrences of acute rheumatic fever fulfill the Jones criteria, some may not.

2 year old patient was brought to your clinic due to neurocognitive delay. Mother recalled that for the past year, the walls of their house have peeling and cracking paint. What should be given to the child? A. EDTA B. Penicillamine C. succimer D. activated charcoal E. deferoxamine

184 A. EDTA should be given as a chelation therapy for chronic lead poisoning

5 year old patient has arthritis of the knees, rashes on the legs and intermittent abdominal pain for the past 3 days. History revealed previous upper respiratory tract infection. If the patient's kidney is involved, IgA mesangial deposition.is expected to be seen in kidney biopsy. What is the most probable diagnosis? A. Post-strep glomerulonephritis B. Kawasaki disease C. Wilms tumor D. Henoch-schonlein purpura E. IgA nephropathy

185 D. Henoch-schonlein purpura cause palpable purpuric rash, arthritis and intermittent abdominal pain. It is the most common cause of non-thrombocytopenic purpura in children. The cause is unknown but it typically follows an URTI.

What is the most common esophageal disorder in children of all ages? A. Achalasia B. Tracheoesophageal fistula C. GERD D. Foreign body obstruction E. None of the above

186 C. GERD is the most common esophageal disorder in children of all ages

11 months old infant was brought in the emergency room due to fever of 2 days duration. Upon arrival at the ER, patient started to have a seizure episode, generalized tonic-clonic, which lasted for 2 minutes until diazepam was given. Temperature was 39.4 deg C. What is the next important step in the management? A. do an EEG B. support ventilation C. give antipyretics D. search for the cause of fever E. give phenobarbital as anticonvulsant prophylaxis

187 D. Routine management of a normal infant with simple brief febrile convulsions includes a careful search for the cause of the fever and reassurance and education of the parents. Although antipyretics have not been shown to prevent seizure recurrences, active measures to control the fever, including the use of antipyretics, may reduce discomfort and are reassuring.

A 12 year old male presents with complaints of polyarthritis and an evanescent rash which spares the face. On PE, patient appears to be in respiratory distress and auscultation revealed tachycardia and a gallop rhythm, and some hard painless freely movable swellings over the extensor surfaces like the elbows. Chest x-ray reveals cardiomegaly and ASOT is 350 Todd units. The following are appropriate in the management of this patient except: A. Penicillin V 200-500mg QID x 1 week B. Complete bed rest C. Morphine D. Digoxin E. O2 supplementation

19 A. The antibiotic treatment for eradication of streptococcus can be either: Penicillin VK x 10days (not just one week) Benzathine PCN 0.6-1.2 MU IM Erythromycin 250mg TID x 10 days Source: Topnotch handout in pediatrics

Decreased C3 levels that does not normalize in 6-8 weeks is suggestive of A. PSGN B. MPGN C. MCD D. MGN E. FSGS

194 B. MPGN

3 years old patient had 8 episodes of passing loose watery stool. Patient was lethargic, not able to drink, with sunken eyes and skin pinch goes back slowly. How will you manage the patient? A. Give 200ml fluid after each loose stool B. Give 1050 ml ORS for 4 hours then reassess C. Administer IVF PLR 420ml for the first 30 minutes then 980ml for the next 2 1/2 hours D. Administer IVF PLR 420ml for the first hour then 980ml for the next 5 hours E. Administer IVF PLR 300ml for the first hour then 700 ml for the next 5 hours

195 C. Patient has severe dehydration. Management is to administer IVF PLR 420ml for the first 30 minutes then 980ml for the next 2 1/2 hours

4 years old male had history of fever. Physical examination revealed strawberry tongue, redness of hands and feet, rash on trunk and cervical lymphadenopathy. What is the most appropriate management? A. Penicillin G 25 000 units/kg/day B. Ceftriaxone 50mg/kg C. Prednisone 1 mg/kg/day D. IVIG and aspirin E. Metronidazole 15mg/kg/day

196 D. Diagnosis for this case is Kawasaki Disease. Management is IVIG and aspirin.

In what condition will you see absence of CD18 which manifests with leukocytosis and inability to form pus? A. Leukocyte adhesion defect B. Chronic granulomatous disease C. Chediak-Higashi Syndrome D. Job's syndrome E. Wiskott-Aldrich Disease

197 A. In Leukocyte adhesion defect, there is absence of CD18. This manifests as leukocytosis and inability to form pus. Another presentation would be a delay in umbilical cord sloughing

A 12 month old infant, born premature, developed low grade fever, rhinorrhea and cough. On PE, there was wheezing and hyperresonance to percussion. CXR showed hyperinflation and interstitial infiltrates. What drug can be given to the patient for this condition? A. Isoniazid B. Amoxicillin C. Ribavirin D. Azithromycin E. Oxacillin

198 C. Diagnosis is bronchiolitis which is most commonly caused by Respiratory Syncitial Virus. Risk factors include prematurity, immunodeficiency, heart or lung disease. Ribavirin is the drug of choice.

Breastfeeding is contraindicated in which of the following? A. Malaria B. Galactosemia C. Active TB D. All of the above E. B and C

199 D. Contraindications to breastfeeding are the following: (Mnemonics- Girl, Bawal Ako Sa SuSo Mo). Galactosemia, Breast Ca, Active TB, Septicemia, Substance abuse, Severe Psychosis, Malaria

A four year old presents with low-grade fever, intermittent crampy abdominal pain with emesis and swollen knees of 3 days duration. A purpuric rash distributed below the knees of both lower extremities is noted on physical examination. This patient most likely has: A. Meningococcemia B. Idiopathic thrombocytopenic purpura C. Henoch-Scholein purpura D. SLE E. Juvenile Rheumatoid arthritis

2 C. HSP is a common vasculitis among children and presents with the classic findings of abdominal pain with or without rectal bleeding, vasculitis rash, arthritis and nephritis. The platelet count is normal. Gastrointestinal involvement may progress to intussusception. Source: Nelson's Textbook of Pediatrics 19th edition p868

The earliest possible age for an infant to receive his first dose DPT is: A. At birth B. 4 weeks of life C. 6 weeks of life D. 8 weeks of life

578 C. At least 1 ½ months after birth, the infant may be eligible for a first DPT dose together with OPV following the EPI recommendation.

The ratio of chest compressions and respiration in a 2 health care rescuer setting is: A. 30: 1 B. 30: 2 C. 15: 1 D. 15:2

579 D. A 15:2 ratio of compression and respiration is advised in a 2 rescuer setting. A 30:2 ratio is advised in a lone rescuer situation.

The ECG of a patient with Tetralogy of Fallow showed right axis deviation. Which component of the tetralogy is directly responsible for this finding? A. Pulmonary stenosis B. Overriding of the aorta C. Ventricular septal defect D. Right ventricular hypertrophy E. Coarctation of the aorta

215 D. A dominant R wave appears in the right precordial leads or an RSR' pattern.

A 5 year old presents with complaint of chronic cough of 1 month duration with associated weight loss and recurrent fever. PPD was done. Which of the following is true. A. Equal to or > 5 is positive. Anything below this value is negative. B. Equal to or > 10 is positive. Anything below this value is negative. C. Equal to or > 15 is positive. Anything below this value is negative. D. Equal to or > 20 is positive. Anything below this value is negative. E. Equal to or > 25 is positive. Anything below this value is negative.

20 B. Equal to or > 5 - positive if with history of close contact, with suggestive clinical findings, suggestive CXR, immunocompromised Equal to or > 10 = positive Source: Topnotch handouts in pediatrics

In a newborn infant, the presence of a third fontanel suggests: A. Patau syndrome B. Edward syndrome C. Down syndrome D. Johanson-Blizzard syndrome E. Turner syndrome

202 C. This is also seen among preterm infants.

Which among the following patients with measles is most likely to develop SSPE? A. A 2 year-old male patient B. A 6 year-old patient who was not given vitamin A C. A 10 year-old patient who has developed pneumonia D. A 14 year-old patient who is immunocompromised E. A 16 year-old with subclinical measles

203 A. Measles at an early age favors the development of SSPE. 50% of SSPE patients had primary measles before 2 years of age and 75% before 4 years of age. Males are affected twice as often as females.

A 2 week-old infant has developed the bronze baby syndrome. This infant most likely has: A. Hyaline membrane disease B. Congenital diaphragmatic hernia C. Truncus arteriosus D. Choledochal cyst E. Hemachromatosis

204 D. The terms bronze baby syndrome refers to a sometimes-noted dark, grayish-brown skin discoloration in infants undergoing phototherapy. Almost all infants observed with this syndrome have had significant elevation of direct-reacting bilirubin and other evidence of obstructive liver disease.

A 1 year-old female was brought in by her mother because of 3 days of loose bowel movement. The patient is lethargic and does not tolerate oral fluids. Her mother noted her last urine output to be scanty. She is tachycardic with very slow capillary refill. Pulses are thready. What should be your immediate management? A. Trial of ORS solution, 50-100 mL/kg body weight over 4 hours B. Administer antibiotics C. Intubate patient D. Send stool exam for fecalysis E. Administer PNSS 20mL/kg IV bolus

205 E. This is severe dehydration.

A 1 week-old SGA infant born at 35 weeks AOG presents with jaundice, hepatomegaly, a blueberry- muffin like rash, bleeding and microcephaly. Which statement is correct? A. The disease is the second most common cause of congenital infection. B. Transmission occurs by direct person-to- person contact; indirect transmission is not possible. C. Acyclovir and foscarnet are treatment options. D. Primary infections in pregnant women cause the most severe symptomatic congenital infections. E. Treatment is indicated for all infected persons.

206 D. A: CMV is the most common. B: indirect transmission is possible. C: Ganciclovir, not acyclovir. E: treatment is not indicated for immunocompetent persons.

In the diagnosis of febrile sezires, a lumbar puncture is strongly considered among children of what age group? A. <6 months B. <12 months C. <18 months D. <24 months E. <36 months

207 B. A LP should be strongly considered in children less than 12 months of age and considered in those 12-18 months old.

Which is not consistent with a diagnosis of Hirschsprung disease? A. Delayed passage of meconium B. A premature infant C. Rectum is empty of feces D. Increased acetylcholinesterase staining on rectal biopsy E. Absent Auerbach's plexus

209 B. Prematuriy is uncommon with this condition.

Which one of the following forms of juvenile idiopathic arthritis is most likely to be associated with serious eye complications? A. Polyarticular arthritis that is seropositive for rheumatoid factor B. Polyarticular arthritis that is seronegative for rheumatoid factor C. Oligoarticular arthritis without axial spine involvement D. Oligoarticular arthritis with axial spine involvement E. Systemic-onset juvenile rheumatoid arthritis

21 C. Patients with oligoarticular arthritis without axial spine involvement are most likely to develop chronic and potentially severe anterior uveitis, which can be clinically quite subtle even as it leads to progressive visual loss. Up to 25% of patients in this subset may develop anterior uveitis, and the group that is ANA positive appears to be at highest risk. Patients with axial spine involvement can also develop anterior uveitis, but this tends to be acute, self-limited, and easily treatable.

A 4 year-old male with Kawasaki disease has a normal echocardiography at the time of diagnosis. Repeat test after 2 weeks revealed no coronary abnormalities. When should you schedule the next 2D-echo? A. 1 months after the onset of illness B. 2 months after the onset of illness C. 3 months after the onset of illness D. 4 months after the onset of illness E. 6 months after the onset of illness

210 B. A repeat study should be performed 6-8 weeks after the onset of illness if both test were negative.

What is the daily maintenance fluid requirement of a 19 kg-patient? A. 1,900 mL B. 1,450 mL C. 1,180 mL D. 1,950 mL E. 1,680 mL

211 B. For 11-20 kg: 1,000 mL + 50 ml for each kg above 10 kg

Which among the following is INCORRECT about necrotizing enterocolitis? A. Preamturity is the greatest risk factor. B. In most cases, no pathogen is identified. C. Premature infants exhibit symptoms earlier than term infants. D. The finding of pneumatosis intestinalis is diagnostic. E. Exclusively breast-fed infants have a reduced risk.

212 C. Age of onset is inversely related to gestational age.

Which finding in a patient with post-streptococcal acute glomerulonephritis is the last to resolve? A. Edema B. Proteinuria C. Elevated ESR D. Hypertension E. Hematuria

213 E. The acute phase generally resolves within 6-8 weeks. Although urinary protein excretion and hypertension usually normalize by 4-6 weeks after onset, persistent microscopic hematuria may persist for 1-2 years after the initial presentation.

Folic acid supplementation should be given at what dose to a woman with who has given birth to a child with meningocele? A. 0.3 mg daily B. 0.4 mg daily C. 3 mg daily D. 4 mg daily E. 5 mg daily

214 D. 0.4 mg daily of folic acid is recommended for women of childbearing age and is capable of being pregnant. This case, however, is a high risk woman.

A 7-year-old boy presents with palpable purpura on the buttocks and legs, fever, abdominal pain and vomiting, arthritis in his knees and ankles, melena, and hematuria. His mother states that he had an upper respiratory illness approximately 1 week ago, but has otherwise been well. Blood tests reveal mild renal insufficiency. The most likely cause of the bleeding into the skin observed in this patient is A. coagulation factor deficiency B. qualitative platelet dysfunction C. quantitative platelet dysfunction D. vasculitis E. vitamin deficiency

22 D. The clinical description is that of Henoch-Schonlein purpura, a form of leukocytoclastic angiitis (hypersensitivity vasculitis) resulting from an immune reaction that damages the vascular endothelium. Henoch-Schonlein purpura is closely related to IgA nephropathy, a glomerulopathy resulting in nephritic syndrome, and may represent a systemic version of this disease.

which of the following is true about systemic type of juvenile rheumatoid arthritis ? A. It presents with pruritic evanescent maculopapular rash on proximal extremities and trunk B. It exhibits koebner phenomenon C. It highly resembles characteristic presentation of rheumatoid arthritis and with similar HLA profile D. visceral involvement ( hepatosplenomegaly) although part of the spectrum is rare in this type of JRA E. all of the above

222 B. A- the evanescent salmon colored rash which only exhibit during the height of the fever is non pruritic. C- among the types of JRA, it is the polyarticular type and not the systemic type that highly resembles classic rheumatoid arthritis. D- there is prominent visceral involvement in systemic JRA.

A 2 y/o male presents with rapidly enlarging abdominal mass. During PE, you noted that there was also bilateral proptosis and echymosis of both periorbital region. Neurological examination showed, miosis of the right eye as well as difficulty in performing finger to nose test. what is the most likely diagnosis? A. Wilms tumor B. ALL with brain metastasis C. Neuroblastoma D. pheochromocytoma E. none of the above

223 C. neuroblastoma is the 3rd most common pediatric cancer. Its classic presentation is an rapidly enlarging abdominal mass that crosses the midline. It also presents with proptosis, ecchymosis, opsomyoclonus and sometimes horner syndrome.

a 4 y/o female was referred to your clinic secondary to a mass occupying the nasopharyngeal area. this is accompanied by congestion, recurrent epistaxis and multiple cranial nerve palsies. Patient is lethargic with complains of headache and recurrent projectile vomitting. what is the most likely diagnosis for this patient? A. rhabdomyosarcoma B. neuroblastoma C. medulloblastoma D. chloroma E. all of the above

224 A. similar to aSIMILAR TO PREVIOUS BOARD EXAM CONCEPT/PRINCIPLE. Rhabdomyosarcoma (RS) is the most common pediatric soft tissue sarcoma. They are usually found in the head and neck, genitourinary, and retroperitoneal area. nasopharyngeal RS presents with mass accompanied by epistaxis, congestion, dysphagia. regional extension leads to cranial nerve palsies, blindness and signs of increased intracranial pressure. medullablastoma is the most common malignant primary brain tumor in pediatrics. it is intraaxial and will not present with mass in the nasopharyngeal area. its most common site is the cerebellar hemisphere. Neuroblastoma seldomly seen in head and neck region. along with wilms, it usually present with an abdominal mass. chloroma is a solid collection of leukemic cells in acute myelogenous leukemia. it was not stated in the case if the patient presents with signs and symptoms of leukemia.

what is the most common cause of persistent proteinuria in school aged children and adolescents? A. Fever induced proteinuria B. PSGN C. IgA nephropathy D. Orthostatic E. Minimal change disease

225 D. SIMILAR TO PREVIOUS BOARD EXAM CONCEPT/PRINCIPLE. A- fever induced, along with exercise, dehydration, cold exposure, CHF , seizure and stress are all types of transient proteinuria.B and C are most likely nephritic in nature. E- minimal change disease is the most common glomerular type of nephrotic syndrome in pediatrics. however, orthostatic proteinuria still accounts for 60% pediatric population with incidental finding of proteinuria. Nelson 18 th ed 2189

which is a correct statement regarding Fever of unknown origin? A. cause cannot be identified after 3 weeks OPD evaluation or 1 week as an Inpatient B. It is usually an atypical presentation of a common disease C. The most frequently associated connective tissue disorder in FUO is SLE and JRA D. all of the above E. none of the above

226 D. Nelson 18th ed pp 1090-1091

During a check up of a well child, what would you expect a 3 y/o child can perform? A. Knows age and sex B. Plays with other children with beginning social interaction C. Hops on one foot D. Throws ball overhand E. Identifies longer of two lines

227 A. a 3 y/o child can do the following: rides tricycle, stands momentarily on one foot, makes tower of 10 cubes, copies circle, imitates cross, knows age and sex, play simple games, washes hands. All the other choices are general characteristic of a 4 y/o child

which of the following is not a cause of abnormal early exfoliation of teeth? A. Histiocytosis X B. neutropenia C. leukemia D. hypoparathyroidism E. None of the above

228 D. delayed eruption ( no teeth at 13 months of age) - hypothyroid, hypoparathyroid, idiopathic. Early exfoliation- histiocytosis x, cyclic neutropenia, leukemia, trauma. Nelson 18th ed p 73

what vitamin/mineral deficiency is associated with intention tremor, decreased proprioception, nystagmus, dysarthria, and retinopathies? A. Vitamin A B. Vitamin B6 C. Vitamin E D. Zinc E. Selenium

229 C. Vitamin A - nyctalopsia, xeropthalmia, bitot spits, epiphyseal bone abnormalities. Vitamin B6 - irritability, convulsion, hypochromic anemia, oxaluria, peripheral neuritis. Zinc - abdominal pain, diarrhea, vomitting, acrodermatitis enterohepatica. Vitamin E deficiency is associated with severe neurologic manifestation. it usually presents with limb and truncal ataxia, ophthalmoplegia, positive rhomberg test, dysarthria

12 year old female patient came with with altered sensorium. Patient had 2 month history of chronic cough accompanied by occasional fevers, weight loss and body malaise. Over the course of 3 days, patient had on and off fevers, increasing daytime sleepiness, irritability and noted to complain of headaches. On physical examination, patient was irritable, conscious but confused, (+) nuchal rigidity, no motor or sensory deficits, no CN deficits. Lumbar puncture was done which revealed glucose of 30 serum glucose of 90, protein of 60mg, WBC of 5000 predominantly lymphocytes with pellicle formation. Which of the following conditions does the patient most likely have? A. acute bacterial meningitis B. subacute partially treated bacterial meningitis C. TB meningitis D. fungal meningitis E. viral encephalomeningitis

242 C. SIMILAR TO PREVIOUS BOARD EXAM CONCEPT/PRINCIPLE., csf picture and pellicle formation = TB meningitis

A mother calls you frantic because she has just been diagnosed with varicella (chicken pox). She delivered a term infant 7 days ago that appears to be eating, stooling, and urinating without difficulty. The child has been afebrile and seems to be doing well. Which of the following is the most appropriate step in management? A. Isolate the infant from the mother. B. Hospitalize the infant in the isolation ward. C. Administer acyclovir to the infant. D. Administer varicella-zoster immunoglobulin to the infant. E. Advise the mother to continue regular well-baby care for the infant.

23 E. Per CDC recommendations, varicella-zoster immunoglobulin (VZIG) should be administered to the infant immediately after delivery if the other had the onset of varicella within 5 days prior to delivery, and immediately upon diagnosis if her chicken pox started within 2 days after delivery. If untreated, about half of these infants will develop serious varicella as early as 1 day of age. If a normal full-term newborn is exposed to chicken pox 2 or more days postnatally, VZIG and isolation are not necessary because these babies appear to be at no greater risk for complications than older children. Acyclovir may be used in infants at risk for severe varicella, such as those infants exposed perinatally.

A 12 y/o male went for consult secondary to tea colored urine. Physical examination: BP 150/90, PR 102 RR 21 T 37.0C. Pale palpebral conjunctiva, anicteric sclerae, clear breath sounds, soft, nontender abdomen. There was grade 1 bipedal edema and with slight periorbital swelling. lab shows the following result (+) ASO, low C3, and subneprotic range proteinuria. patient was hospitalized for a week and was successfully treated. he was lost to follow up, and just come back after 3 months. laboratories still show persistently low C3. what is your diagnosis ? A. FSGS B. MPGN C. APSGN D. MGN E. RPGN

233 B. MPGN is the most common cause of chronic glomerulonephritis in children and adolescent. Its presentation is very similar to APSGN however, the only difference is a persistently low C3 even after 2 months.

A 3 mo old infant has a history of failure to thrive accompanied by infrequent bowel movement. During rectal examination, patient subsequently passes large amount of stool. Rectal tone was normal. Which of the following is not needed in establishing the diagnosis? A. rectal motility test B. rectal biopsy C. barium enema D. colonoscopy E. none of the above

234 D. this is a classic case of hirschprung disease. the gold standard for the diagnosis is rectal biopsy. Showing absence of ganglion. Rectal motility and barium enema will also highly yield a diagnostic clue for the presence of this disease. colonoscopy will provide normal result as there are no visible mucosal defect in hirschprung.

A 6 y/o female presents with a 2 day history of painless blood stained stool. During the consult, patient is comfortable and denies any feeling of pain. abdominal examination is normal. However, she has pale palpebral conjunctiva and noted to be tachycardic for her age. what is the best initial test to request to establish your diagnosis? A. Angiography B. Meckel scan C. air contrast enema D. ultrasound of the abdomen E. abdominal Xray

235 B. the highest yield to detect meckels diverticulum at this stage is the use of a meckels scan. Since it is located at the mid to late portion of ileum, colonoscopy will not yield any abnormal result. Xray and abdominal utz also will be normal. Angiography can be use if the patient is in the state of massive bleeding.

A 2 day old infant was noted to be cyanotic during its hospital stay. Cardiac examination showed 3-4/6 systolic murmur with prominent second heart sound without any splitting. 12LECG - LVH, CXR: decrease pulmonary markings with a normal cardiac size without displacement. which of the following medications is needed to administer ? A. Prostaglandin E1 B. Digoxin C. Ibuprofen D. Morphine E. Furosemide

236 A. based on the ancillary results, this might be a case of pulmonary atresia. Since this condition is ductal dependent. It is better to administer prostaglandin E1 to prevent ductal closure. Administration of ibuprofen will lead to obliteration of ductal patency and the patient will succumb to acidosis and later death.

A neonate was transferred to ICU d/t clinical findings of pneumonia, peritonitis and omphalitis. there was also a delayed separation of the umbilical stump. Which type of immune disorder is most likely affected in this case? A. Phagocyte function B. B cell C. T cell D. Combined B and T cell E. Phagocyte enzyme deficiency

237 A. delayed separation of umbilical stump is a primary characteristic of leukocyte adhesion defect. Phagocytes are unable to migrate to extravascular spaces d/t deficiency in integrin receptors. The standard treatment for this condition is bone marrow transplantation

A 8 y/o female presenting with hyperaldosteronism is unresponsive to dexamethasone. You requested for CT scan of the abdoment however results are equivocal. What is your next to establish your diagnosis ? A. Explorative laparotomy B. adrenal venous blood sampling C. metyrapone therapy D. increase the dose of dexamethasone E. abdominal MRI

238 B. a normal CT findings does not exclude the presence of an adrenal mass. This is particularly true especially if the mass is subcentimeter in size. Local production of aldosterone can be detected using adrenal vein catheterization.

which of the following is not a principal feature of osteogenesis imperfecta? A. microcephaly B. Defect in type 1 collagen C. Autosomal dominant trait D. Early conduction deafness E. Vertebral compression fractures

239 A. osteogenesis imperfecta is an autosomal dominant disorder affecting type 1 collagen. Its principal features include macrocephaly, fractures, blue sclerae and early deafness.

A 10-month-old baby boy has a 5-hour history of intermittent crying, and with intermittent drawing up of his knees to his chest in between crying episodes. On the way to the emergency room he passes a loose, bloody stool. He has had no vomiting and has refused his bottle since the crying began. Physical examination is noteworthy for an irritable infant whose abdomen is very difficult to examine because of constant crying. His temperature is 38.8°C (101.8°F). The rectal ampulla is empty, but there is some gross blood on the examining finger. Which of the following studies would be most helpful in the immediate management of this patient? A. Stool culture B. Examination of the stool for ova and parasites C. Air contrast enema D. Examination of the blood smear E. Coagulation studies

24 C. The usual presentation of intussusception is that of an infant between 4 and 10 months of age who has a sudden onset of intermittent colicky abdominal pain. The child can appear normal when the pain abates, but as it recurs with increasing frequency, the child can begin to vomit and become progressively more obtunded. The passage of stool containing blood and mucus, frequently described as resembling currant jelly, is often observed. Early examination of the abdomen can be unremarkable, but as the problem persists, a sausage-shaped mass in the right upper quadrant is frequently palpated. An air, barium, or saline enema examination under fluoroscopic or ultrasound control can be therapeutic as well as diagnostic when the hydrostatic effects of the contrast serve to reduce the intussusception, but should be performed with surgical backup, as a complication of attempted reduction is intestinal perforation. Rates of intestinal perforation are lowest with air reduction. Early diagnosis prevents bowel ischemia. The cause of most intussusceptions is unknown, but a Meckel diverticulum or polyp can serve as a lead point. None of the other choices would result in a correct diagnosis (and potential therapy) for the child with a classic presentation for intussusception.

A 7 year old female child had fever of 4 days duration, body aches, and malaise. The concerned mother brought her to the clinic for evaluation since according to her there were already a lot of dengue cases arising in the area where they lived, which of the following laboratory parameters would indicate a possible dengue infection? A. increased Hematocrit B. increased WBC neutrophilia C. (-) NS1 antigen test D. platelet count of 180,0000 E. none of the above

244 A. SIMILAR TO PREVIOUS BOARD EXAM CONCEPT/PRINCIPLE, increased hematocrit may indicate dengue due to increased capillary permeability

An 11 year old male patient presented to a clinic with 2 day history of fever accompanied by the onset of a maculopapular rash beginning at the face progressing downwards to the trunk and to the extremities, there was anorexia, body malaise as well. Physical exam revealed posterior auricular lymphadenopathy, which of the following is true regarding this condition? A. this condition is relatively self limiting B. usually called 5 day measles C. it is caused by a paramyxovirus D. virtually no complications arise among pregnant women E. can cause orchitis and infertility among adolescent males

245 A. SIMILAR TO PREVIOUS BOARD EXAM CONCEPT/PRINCIPLE, a rubella case, only A is correct it is 3 day measles, toga virus, no orchitis

A 6 year old male patient was admitted due to flank pain, peristently high fever, vomiting. Urinalysis revealed wbc 15-20, rbc 3-5, wbc casts and (-) crystals, (+) leukocyte esterase, patient was noted to have had 2 previous episodes of UTI over the past month, which of the following diagnostic procedures should be done to work up the possible etiology of the repeated infections? A. DMSA renal scan B. voiding cystourethrogram C. retrograde pyelography D. urine culture E. all of the above

247 B. SIMILAR TO PREVIOUS BOARD EXAM CONCEPT/PRINCIPLE, usually repeated infections on males are a result of a congenital anomaly most commonly posterior urethral valves work up would be VCUG

A neonate was born from a 32 year old G1P1 mother, he was noted to be acrocyanotic, the nurse noted difficulty in passing a suction catheter through the right nostril, unilateral choanal atresia is suspected, a nasogastric tube was tried but was unable to insert also through the right nostril, which is the next appropriate step of evaluation? A. cranial CT scan B. cranial MRI C. anterior rhinoscopy D. needle guided NGT insertion E. none of the above

248 A. SIMILAR TO PREVIOUS BOARD EXAM CONCEPT/PRINCIPLE, Cranial CT scan can diagnose choanal atresia conveniently

A 13-year-old boy presents to the pediatrician with extrapyramidal signs, including a resting and kinetic tremor. An ophthalmologic examination demonstrates the presence of dark rings that appear to encircle the iris of the eye, and his laboratory studies demonstrate elevated liver enzymes. It is likely that the patient's condition is associated with the accumulation of A. Copper B. Councilman bodies C. Eosinophilic hyaline inclusions D. Glycogen E. Iron

25 A. Wilson disease is a hereditary condition associated with the accumulation of copper in the liver, brain, and eye. Accumulation of copper in the Descemet membrane of the eye results in the pathognomonic lesion known as the Kayser-Fleischer ring. Accumulation in the liver results in cirrhosis. Accumulation in the brain, specifically in the basal ganglia, results in motor symptoms. Councilman bodies are apoptotic hepatocytes that were first identified in yellow fever. Eosinophilic hyaline inclusions, Mallory bodies, are seen in alcoholic liver disease. Glycogen accumulates in the liver in numerous glycogen storage diseases. Iron accumulates in hemochromatosis.

A 3 year old child was brought by the mother for consult, the mother noted decreased activity over the past few months, noted episodes of interrupted physical play, episodes of bluish tinge over the lips and fingers, noted behavioral changes like squatting, the degree of cyanosis is proportional to the severity of which anatomic defect? A. VSD B. overriding aorta C. RVH D. pulmonic stenosis E. aortic stenosis

252 D. SIMILAR TO PREVIOUS BOARD EXAM CONCEPT/PRINCIPLE, TOF, pulmonic stenosis determines cyanosis

An 8 year old male child was brought for consult for an 8 month history of gradually enlarging cheek mass 5 by 5cm on the right cheek, there was noted some difficulty of breathing and nasal obstruction, frequent complaints of headaches, and mild pain over the mass. A. acute lymphocytic leukemia B. non hodgkin's lymphoma C. rhabdomyosarcoma D. ewing's sarcoma E. salivary gland mixed tumor

253 C. SIMILAR TO PREVIOUS BOARD EXAM CONCEPT/PRINCIPLE, rapidly expanding mass, local mass effects causing signs and symptoms, not much systemic manifestations as heralded by other cancers especially hematological

A 12 year old male presented to the clinic jaundice and edema. Patient noted decreased urine output accompanied by bipedal pitting edema. 5 days prior, patient had a bout of watery diarrhea with fever which resolved spontaenously. Patient presented with some petechia over the lower extremities. Evaluation revealed anemia with Hgb of 10g/dL WBC of 12000, platelet count of 70,000 which of the following conditions might be present? A. Thrombotic thrombocytopenic purpura B. hemolytic uremic syndrome C. Iga nephropathy D. henoch schonlein purpura E. none of the above

254 B. SIMILAR TO PREVIOUS BOARD EXAM CONCEPT/PRINCIPLE, history of previous infection gastroenteritis, jaundice, renal failure, thrombocytopenia = HUS

A 16 year old female patient came in due to bipedal pitting edema. On further probing, patient had occasional abdominal pains, joint pains involving the ankles and knees intermittently during the past week after a bout of cough and colds after she came down with a flu. Occasional mild fevers were also noted with no other remarkable symptoms noted. CBC revealed Hgb of 13, WBC of 6000, Platelet count of 200,000, no other symptoms were noted. Urinalysis revealed RBC 5-10/hpf, WBC 0-1, with (+) RBC casts, protein +1, which of the following conditions does the patient most likely have? A. IgA nephropathy B. Hemolytic uremic syndrome C. Henoch Schnolen Purpura D. Focal Glomerulosclerosis E. None of the Above

255 A. patient most likely has IgA nephropathy, with signs of autoimmunity joint pains, abdominal pain, previous URTI, relatively normal laboratory parameters except hematuria in the urine, no other symptoms noted, FSGS causes nephrotic syndrome

16 year old male student in a dormitory presented with increasing daytime sleepiness, irritability and headache, patient noted to have nuchal rigidity and fever over the past 2 days. After a course of 8 hours, patient was noted to have purpuric rashes from the trunk extending to the lower extremities, which of the following found in the patient infers a better prognosis in terms of this disease? A. meningococcal bacteremia B. meningitis C. waterhouse friederichsen syndrome D. septicemia E. septic shock and low WBC count

256 B. SIMILAR TO PREVIOUS BOARD EXAM CONCEPT/PRINCIPLE, most deaths occur when there is blood infection, meningococcemia without meningitis = high rate of death compared to meningitis alone

A 5 year old male patient was brought to the clinic because his mother noticed a gradual enlarging abdominal mass since 6 months ago which she didn't mind at first until she noticed it growing in size while she was bathing the child and there was onset of tea colored urine observed for the past 2 months. Physical exam was unremarkable except for the 5x5cm painless abdominal mass which does not cross the midline. There was also noted hematuria on futher workup which of the following conditions does the child probably have? A. neuroblastoma B. nephroblastoma C. rhabdmyosarcoma D. lymphoma E. none of the above

257 B. SIMILAR TO PREVIOUS BOARD EXAM CONCEPT/PRINCIPLE, abdominal mass does not cross midline hematuria - wilms tumor, no other symptoms noted

A male neonate was born to a G1P1 mother, on grading the newborn's maturity, all extremities recall immediately to flexed position, square window less than 30 degrees, heal to ear test heal only up to the level of genitals, sparse lanugo, creases anterior 2/3, testes down with good rugae, raised areola 3-4mm bud, what is the approximate maturity in weeks of the baby? A. 32-34 weeks B. 36-38 weeks C. 40-42 weeks D. 28-30 weeks E. none of the above

258 B. recalled from the board exams, baby looks mature based on the limited data seems term 36-38 weeks same with what was given during our boards

A 5 year old female infant was noted by the mother to be always clumsy and falls easily while walking and playing, she was also noted to walk with an unsteady gait, further workup upon consult showed a mass located centrally on the cerebellum with heterogenous calcifications, which of the following is the most likely condition? A. glioblastoma B. astrocytoma C. medulloblastoma D. meningioma E. all of the above

259 C. recalled from the board exams, unsteady gait, heterogenous calcification over the mass on the cerebellar area

A 10-month-old infant on long-term aspirin therapy for Kawasaki disease develops sudden onset of high fever, chills, diarrhea, and irritability. A rapid swab in your office dentifies influenza A, adding her to the long list of influenza patients you have seen this December. Over the next few days, she slowly improves and becomes afebrile. However, 5 days after your last encounter you hear from the hospital that she has presented to the emergency center obtunded and posturing with evidence of liver dysfunction. Which of the following statements about her current condition is correct? A. With proper supportive care, the overall mortality rate is low. B. With her progressive liver dysfunction, increased total serum bilirubin is anticipated. C. Administration of N-acetylcysteine is first-line therapy D. Seizures are uncommon with this condition. E. Death is usually associated with increased intracranial pressures and herniation.

26 E. Reye syndrome is an acquired mitochondrial hepatopathy that results from the interaction of an influenza (or varicella) infection and aspirin use. While prevalence has decreased over the last few decades and it is now a rare disease, mortality remains the same at more than 40% of cases. Liver enzymes and ammonia are elevated, but total bilirubin is not. Patients initially present toward the end of a viral infection with sleepiness, emesis, and abnormal liver functions. As the disease progresses, the patient may develop seizures, coma, hyperventilation, and decorticate posturing. Ultimately they may develop respiratory arrest, loss of deep tendon reflexes (DTRs), and fixed and dilated pupils. Death is usually from cerebral edema and subsequent herniation. While aspirin is no longer routinely used in children as an antipyretic or pain reliever, the increase in the use of aspirin in adults with heart disease requires specific counseling for parents of children with influenza and varicella to avoid aspirin use. In addition, both of these infections are preventable with proper immunization. N-acetylcysteine is protective of hepatocytes in acetaminophen overdose.

The predominant pathophysiologic mechanism present in dengue shock syndrome is due to? A. increased capillary permeability B. septic shock from secondary bacterial infection C. hemmorhage secondary to thrombocytopenia D. dehydration secondary to fever and decreased intake E. none of the above

260 A. increased capillary permeability is responsible for 3rd spacing and dengue shock syndrome

A newborn patient born via NSVD was observed to be cyanotic in the nursery. There was noted difficulty upon trying to insert a feeding catheter through the nose. The neonate was observed to turn pink upon crying. What is the diagnosis? A. Tracheo-esophageal fistula B. Congenital Heart Disease C. Choanal Atresia D. Cleft palate E. Respiratory distress syndrome

261 C. Characteristic description of choanal atresia

You are treating a pediatric patient with anemia. Your primary impression is iron-deficiency. What laboratory test will you request on the 3rd day after the start of treatment to determine response? A. Hemoglobin level B. Serum ferritin C. Serum iron D. Reticulocyte count E. Bone marrow aspiration

264 D. An increase of reticulocyte count by the 3rd day indicates response to iron therapy. Hgb levels don't usually start to rise until after the 4th day

What is the most common type of congenital heart defect in patients with trisomy 21? A. VSD B. ASD C. Tricuspid atresia D. Patent ductus arteriosus E. Endocardial cushion defect

265 E. Endocardial cushion defect is the most common CHD in trisomy 21

A term neonate born to a 32 year old mother via cesarean delivery for CPD, becomes tachypneic. Chest X-ray reveals air trapping, fluid in fissures and perihilar streaking? What is the most likely diagnosis A. Hyaline membrane disease B. Meconium aspiration syndrome C. Transient tachypnea of the newborn D. Bronchopulmonary dysplasia E. None of the above

266 C. TTN usually occurs in term newborn, cesarean delivery, rapid 2nd stage of labor; usually resolves with minimal oxygen requirement

A 6 year old female patient is brought to you by her mother due to complaints of petechiae, purpura, and bleeding after a dental tooth extraction. She was previously healthy with no significant history. Which of the following will NOT support a diagnosis of Idiopathic thrombocytopenic purpura? A. Severe thrombocytopenia < 20,000 B. Normal to increased megakaryocytes in the bone marrow C. Presence of hepatosplenomegaly D. Preceded by a history of viral infection 2-3 weeks prior E. None of the above

267 C. Hepatosplenomegaly is not consistent with ITP and indicates a different diagnosis. All others are characteristics of ITP

A newborn patient born to a G1P1 mother was noted to have severe cyanosis. Physical examination revealed a holosystolic murmur along the left sternal border and ECG shows left axis deviation with left ventricular hypertrophy. What is the most likely diagnosis? A. Tricuspid atresia B. VSD C. ASD D. Transposition of the great arteries E. Tetralogy of Fallot

268 A. Severe cyanosis at birth with associated decreased pulmonary circulation at X-ray and LVH with left axis deviation is charfacteristic of tricuspid atresia

A 5 year old patient is brought to your clinic. Her parents claim that the child started to show difficulty in attempting to stand from a sitting position and has lordotic posturing. They can also recall that they noted poor head control when he was an infant. Physical examination reveals hypertrophy of the calf muscles. What is the diagnosis? A. Amyotrophic lateral sclerosis B. Duchenne muscular dystrophy C. Charcot Marie Tooth Disease D. Myotonic dystrophy E. Gullain-Barre syndrome

269 B. Gower sign, pseudohypertrophy of the calf muscles, scoliosis, lordotic posturing - signs of Duchenne muscular dystrophy, an X-linked recessive disorder

The most common etiologic agent of the most common major complication of MCD is A. Klebsiella oxytoca B. Streptococcus pneumoniae C. Pseudomonas aeruginosa D. Staphylococcus aureus E. Enterococcus faecalis

58 B. The most common cause of spontaneous bacterial peritonitis among patient with MCD is Streptococcus pneumoniae.

A 4-year-old boy in your practice has been having many infections. You suspect that he may have an immune dysfunction. Quantitative immunoglobulins were sent and were normal. Which of the following immunodeficiency disorders is associated with normal immunoglobulin G (IgG) levels? A. X-linked agammaglobulinemia B. DiGeorge syndrome C. Late-onset hypogammaglobulinemia D. Ataxia-telangiectasia E. Severe combined immunodeficiency

27 B. Although reduced levels of immunoglobulin A (IgA) or IgE may be seen in patients with DiGeorge syndrome (T cell deficiency disorder), the total serum immunoglobulin level usually is normal and IgG levels are normal In X-linked agammaglobulinemia (Bruton's) and late-onset hypogammaglobulinemia, IgG, IgM, and IgA levels are all reduced, and the total immunoglobulin level is less than 100 mg/dL. Patients with ataxia-telangiectasia have a defect in their DNA repair mechanism. One of the clinical features is low IgA and IgG. Severe combined immunodeficiency results in a decrease in all immunoglobulins.

A 10 year old patient is diagnosed with Acute Rheumatic fever. Which of the following is true regarding the treatment of this condition? A. Prednisone is usually given to patients with or without carditis or CHF symptoms B. Penicillin prophylaxis therapy is given to patients with no carditis or cardiac complications only up to their early twenties C. Aspirin is the drug of choice for patient who present with chorea alone D. A daily dose of intramuscular benzathine penicillin G is required for prophylaxis E. None of the above

270 B. Prednisone is only given to patients with carditis/CHF; Phenobarbital is the drug of choice if chorea is an isolated finding; Penicillin prophylaxis is may be given for life in patients with carditis but only until the early twenties in those without out. IM Pen G prophylaxis is given once every 4 weeks

An 11 month old infant is seen in the ER presenting with dyspnea. His mother tells you that it all started with coryza, cough, and low-grade fever and then with recent onset of barky cough that is getting worse. Upon examining the infant, you hear seal- like coughing and there is hoarseness and inspiratory stridor. Other than tachypnea, vital signs remain stable. What will be an appropriate management? A. IV antibiotic therapy B. Intubate immediately C. Nebulized epinephrine D. Ribavirin therapy E. None of the above

271 C. This is a case of viral croup which requires supportive management which may include nebulized epinephrine. No need to intubate unless patient is in severe respiratory distress/hemodynamically unstable

Which of the following pediatric diarrheal conditions will require antibiotic therapy? A. A child presenting with watery diarrhea with a suspected viral cause B. A child with diarrhea due to ingestion of nontyphoidal salmonella-infected, improperly cooked eggs C. A child who presents with bloody diarrhea that is associated with anemia, thrombocytopenia, and hypertension D. A child with high fever and mucoid stool with frank blood E. B, C, and D

272 D. Patients with viral gastroenteritis, HUS, and nontyphoidal salmonellosis will usually not require antibiotic therapy.

A neonate is born to a mother with known HIV infection. The mother did not receive any antiretroviral treatment during pregancy. What test will you request to screen the newborn for HIV? A. HIV-DNA PCR B. IgG Ab by ELISA C. IgG Ab by Western blot D. CD4 count E. None of the above

273 A. All infants < 18 months with or without HIV will test positive for IgG antibodies because maternal IgG crosses the placenta so testing should done using HIV- DNA PCR

A 8 year old patient who had a previous skin infection 3 weeks prior now presents with hematuria, elevated BP, and bilateral lower extremity edema. What test will you use to document previous GABHS infection? A. anti-ASO B. anti-Hyaluronidase C. anti-Streptokinase D. anti-NAD E. anti-DNase B

274 E. All of the choices are anti-streptococcal antibodies which may be elevated after a GABHS throat infection. However, in GABHS pyoderma or skin infection, anti- Dnase B is the best test because the others may be falsely low or negative

What is the most common associated congenital anomaly in a patient with esophgeal atresia and tracheoesophageal fistula? A. Vertebral anomalies B. Imperforate anus C. Cardiac anomalies D. Genitourinary anomalies E. Limb defects

275 C.

Which is of the following is true regarding brain tumors in children? A. Most commonly supratentorial in location in children <10 B. Benign astrocytoma is the most common brain tumor in children C. Brain is the most frequent solid malignancy in children D. A and B E. B and C

276 E. Infratentorial is most common. Benign astrocytoma is the most common brain tumor followed by medulloblastoma. ALL is the most common malignancy in children but brain tumors is the most common solid malignancy.

Which of the following immunizations may given to a child with diagnosed HIV/AIDS who is not severely immunocompromised? A. OPV B. Vaccinia C. BCG D. MMR E. None of the above

278 D. Live vaccines are usually contraindicated in children with HIV/AIDS but MMR and varicella may be given if the child is not severely immunocompromised

An 11 month old infant presented at the ER with fever, irritability, and nausea and vomiting. Currently, patient is now lethargic and still febrile. P.E. fails to reveal meningeal irritation but you still suspect meningitis so you perform a lumbar tap. Which of the following CSF findings will indicate a mycobacterial etiology? A. 3000 cells/mL, predominantly PMN, low glucose, high protein, high pressure B. 200 cells/mL, predominantly PMN, low glucose, high protein, high pressure C. 350 cells/mL, mixed PMN and lymphocytes, normal glucose, slightly high protein, normal pressure D. 400 cells/mL, predominantly lymphocytes, low glucose, high protein, high pressure E. None of the above

279 D. A is bacterial, B is partially treated bacterial, C is viral/aseptic

A 5-month-old child was normal at birth, but the family reports that the child does not seem to look at them any longer. They also report the child seems to startle more easily than he had before. Testing of his white blood cells (WBCs) identifies the absence of hexosaminidase A activity, confirming the diagnosis of which of the following? A. Niemann-Pick disease, type A B. Infantile Gaucher disease C. Tay-Sachs disease D. Krabbe disease E. Fabry disease

28 C. There were 2 questions about Tay-Sachs in our Pediatric Exam. Children who have Tay-Sachs disease are characterized by progressive developmental deterioration; physical signs include macular cherry-red spots and exquisite and characteristic sensitivity to noise. Diagnosis of this disorder can be confirmed biochemically by the absence of hexosaminidase A activity in WBCs. The other GM2 gangliosidosis, Sandhoff disease, results from a deficiency of both hexosaminidase A and B. Tay-Sachs disease is inherited as an autosomal recessive trait; frequently, affected children are of Eastern European Jewish ancestry. The other disorders listed in the question are associated with enzyme deficiencies as follows: Niemann-Pick disease (type A), sphingomyelinase, which results in a normal-appearing child at birth who then develops hepatosplenomegaly, lymphadenopathy, and psychomotor retardation in the first 6 months, followed by regression after that; Infantile Gaucher disease (type II Gaucher), glucosidase, which presents in infancy with increased tone, strabismus, organomegaly, failure to thrive, strider, and several years of psychomotor regression before death; Krabbe disease (globoid cell leukodystrophy), galactocerebroside-galactosidase, which presents early in infancy with irritability, seizures, hypertonia, and optic atrophy, with severe delay and death usually occurring in the first 3 years of life; Fabry disease, alpha-galactosidase, which presents in childhood with angiokeratomas in the bathing trunk area, ultimately resulting in severe pain episodes.

All of the following cases of urinary tract infection in pediatric patients will require additional assessment with Voiding cystourethrogram (VCUG) EXCEPT? A. All Males with UTI B. Females > 5 years old with a first episode of UTI C. Febrile UTI D. All cases will require VCUG

280 B. Cases that will require additional assessment with VCUG include: All males, females <5yo, females >5yo with a second UTI, Febrile UTI

What is the easiest way to confirm the diagnosis of bilary atresia in a 1 month old infant presenting with cholestatic jaundice? A. Presence of a triangular cord sign in the ultrasound of the liver B. Intact lobular structure on liver biopsy C. Poor hepatic uptake of radioactive material but with slow excretion into the gut D. Firm consistency of the liver on palpation E. Elevated levels of anti-mitochondrial antibodies

281 A. Abdominal ultrasound is a helpful diagnostic tool in the evaluation of neonatal cholestasis because it will identify choledocholithiasis, perforation of the bile duct, or other structural abnormalities of the biliary tree such as a choledochal cyst. In patients with biliary atresia, ultrasound may detect associated anomalies such as abdominal polysplenia and vascular malformations. Ultrasonographic triangular cord (TC) sign, which represents a cone-shaped fibrotic mass cranial to the bifurcation of the portal vein, may be seen in patients with biliary atresia

A 13 year old girl sought consult due to easy fatigability and poor weight gain. PE was essentially normal except for a grade 3/6 systolic murmur at the 2nd left interspace and a fixed splitting of S2. What Is the diagnosis for this case? A. Coarctation of the aorta B. VSD C. ASD D. PDA E. TGA

282 C. A child with an ostium secundum ASD is most often asymptomatic; the lesion may be discovered inadvertently during physical examination. Even an extremely large secundum ASD rarely produces clinically evident heart failure in childhood. In most patients, the 2nd heart sound is characteristically widely split and fixed in its splitting in all phases of respiration. . A systolic ejection murmur is heard; it is medium pitched, without harsh qualities, seldom accompanied by a thrill, and best heard at the left middle and upper sternal border

Based on the ARIA guidelines, a patient is classified to have persistent allergic rhinitis if which of the following is present? A. Symptoms occur < 4 days a week B. Symptoms occur < 4 consecutive weeks C. Impairment of activities of daily living D. Symptoms occur > 4 days a week OR > 4 consecutive weeks E. Symptoms occur > 4 days a week AND > 4 consecutive weeks

283 E. Intermittent AR: Symptoms occur < 4 days a week or <4 consecutive weeks. Mild AR: normal sleep, no impairment of ADL, school or work. Moderate to severe: troublesome sleep, impairment of ADL, work and school

Which of the following if present confers a better prognosis for meningococcemia? A. Leukopenia B. Presence of petechiae <12 hours before admission C. Nuchal rigidity D. Hypothermia E. Low ESR

284 C. Poor prognostic factors on presentation include hypothermia or extreme hyperpyrexia, hypotension or shock, purpura fulminans, seizures, leukopenia, thrombocytopenia (including DIC), acidosis, and high circulating levels of endotoxin and TNF-α. The presence of petechiae for <12 hr before admission, absence of meningitis, and low or normal erythrocyte sedimentation rate indicate rapid, fulminant progression and poorer prognosis.

An 8 year old girl was brought in for declining academic performance. Her teachers have reported that she is frequently "daydreaming" in class and inattentive. The doctor hyperventilates her in the clinic and she manifests with blank stares and clonic movements of the eyes, there was no post ictal drowsiness noted. If an EEG would be performed on this patient, what would be the expected findings? A. Normal tracing B. Typical 3/sec spike and generalized wave discharge C. Fast spike wave complexes of ≥2.5 Hz and a normal background rhythm D. 4â€"6/sec irregular spike and wave pattern, which is enhanced by photic stimulation E. Chaotic pattern of high-voltage, bilaterally asynchronous, slow-wave activity

285 B. Simple (typical) absence (petit mal) seizures are characterized by a sudden cessation of motor activity or speech with a blank facial expression and flickering of the eyelids. Patients do not lose body tone, but their head may fall forward slightly. Immediately after the seizure, patients resume preseizure activity with no indication of postictal impairment. Automatic behavior frequently accompanies simple absence seizures. Hyperventilation for 3â€"4 min routinely produces an absence seizure. The EEG shows a typical 3/sec spike and generalized wave discharge

A couple sought consult for genetic counseling. On history, they relayed that both their children, one male and one female died at 1 week of life due to an acute encephalopathy. Both parents are healthy, with no known co-morbids, asymptomatic and non- consanguineous. No other relatives are affected or have died under similar circumstances. What is the most probable pattern of inheritance in this case? A. Autosomal recessive B. Autosomal dominant C. X-linked recessive D. X-linked dominant E. All of the above

287 A. For X-linked recessive inheritance usually the male offspring would be affected, while the female offspring would be a carrier. For X-linked and autosomal dominant, the parents would also manifest with similar symptoms to their offsprings. Autosomal recessive pattern is more compatible since both parents are normal but carry the genes (carriers) both children were affected since they inherited both sets of genes from the mother and father

A 9 year old boy was brought to the clinic due to pallor, easy fatigability and numerous petechiae. A CBC was done which showed anemia, thrombocytopenia and leukocytosis. A bone marrow aspiration was subsequently done which showed numerous lymphoblasts. Which of the following statements regarding the patient's disease is true? A. Initial therapy is usually given for a period of 7 days. B. CSF examination is essential for staging the disease C. Children who are >10 yr of age or who have an initial leukocyte count of >50,000/μL are considered to be at a lower risk D. Hepatomegaly is a common manifestation of this disease E. Treatment is the single most important prognostic factor

288 E. Dx: ALL. In general, the initial therapy is designed to eradicate the leukemic cells from the bone marrow; this is known as remission induction. During this phase, therapy usually is given for 4 wk. Staging of ALL is based partly on a cerebrospinal fluid (CSF) examination. If lymphoblasts are found and the CSF leukocyte count is elevated, overt CNS or meningeal leukemia is present. Hepatomegaly is an uncommon manifestation. Children >10 and with an inital WBC count >50,000 are at higher risk

A 5 year old girl presented with a 2 month history of progressive difficulty in ambulation, particularly in climbing up and down the stairs. After going on a field trip, she came home with an erythematous rash in the malar area and violaceous discoloration with swelling over the periorbital area. On the day of consult, she was noted to have papules over the elbows and interphalangeal joints. What is the most likely diagnosis of this case? A. Systemic Scleroderma B. Juvenile Dermatomyositis C. Juvenile Rheumatoid Arthritis D. Systemic Lupus Erythematosus E. Ankylosing Spondylitis

289 B. Juvenile dermatomyositis (JDM), the most common of the pediatric inflammatory myopathies, is distinguished by a characteristic rash and proximal, symmetric muscle weakness that is often responsive to the immunosuppressive therapy. The rash often has onset in sun-exposed areas and develops as the first symptom in 50% of cases, and concomitantly with weakness in 25% of cases. The characteristic periorbital violaceous erythema (heliotrope) may cross the bridge of the nose, in a masklike distribution, and involve the ears as well. The rash is often florid and is usually palpable over joints, especially the metacarpal phalangeal, intercarpal phalangeal (Gottron papules), knees, elbows, and medial malleoli of the ankles

The parents of a 2-week-old infant who was born at home bring him to the emergency center in your local hospital for emesis and listlessness. On examination, you find a dehydrated, listless, and irritable infant. Although you do not have a birth weight, the parents do not feel that he has gained much weight. He has significant jaundice. His abdominal examination is significant for both hepatomegaly and splenomegaly. Laboratory values include a total bilirubin of 15.8 mg/dL and a direct bilirubin of 5.5 mg/dL. His liver function tests are elevated and his serum glucose is 38 mg/dL. His admit urinalysis is negative for glucose but positive for Gram-negative rods; his urine and his blood ultimately grow E. coli. Which of the following nutritional considerations should be considered in this child? A. Administration of high doses of vitamin B6 B. Initial diet free of branched-chain amino acids C. Lactose free formula D. Protein restriction and supplementation with citrulline E. Initiation of a diet low or free of phenylalanine

29 C. The patient has classic findings of galactosemia. Galactose is a component of lactose, found in breast milk and most infant formulas. Symptoms of galactosemia occur in the first weeks of life. While screening for classic galactosemia typically is part of the newborn metabolic panel, patients fitting the clinical presentation as outlined in the question must be evaluated promptly. Signs and symptoms in addition to those presented in the vignette include cataracts and ascites. While three different errors in galactose metabolism are known, most cases result from the deficiency in galactose-1-phosphate uridyl transferase. Urine-reducing substances can be positive, but a routine urinalysis will be negative, as the urine strips do not react with galactose. Patients are at increased risk for E coli sepsis, and this infection may precede the diagnosis of galactosemia. Prompt removal of galactose from the diet usually reverses the symptoms, including cataracts. Use of a phenylalanine-free diet would be appropriate for a patient with phenylketonuria (PKU) Protein restriction and supplementation with citrulline might be used for treating ornithine transcarbamylase deficiency A diet free from branched-chain amino acids would be appropriate for maple syrup urine disease. Part of the treatment for a patient with homocystinuria is high doses of vitamin B6.

A 3 year old girl was brough to the ER due to pallor, lethargy and decreased urine output. The said symptoms were preceded by a 5 day history of bloody diarrhea and fever. Blood tests revealed anemia, thrombocytopenia and an elevated creatinine level. What is the most common causative agent that initiated the events that led to the patient's currrent condition ? A. Giardia lamblia B. Vibrio cholerae C. Escherichia coli D. Staphylococcus aureus E. Cytomegalovirus

290 C. The hemolytic-uremic syndrome (HUS) is the most common cause of acute renal failure in young children. It is classically characterized by the triad of microangiopathic hemolytic anemia, thrombocytopenia, and uremia.acute enteritis with diarrhea caused by Shiga-like toxinâ€"producing Escherichia coli 0157: H7 precedes 80% or more of HUS cases

A 6 year old girl was admitted in the PICU because of multiple organ injuries secondary to a vehicular accident. On the 2nd hosptial day, there was note of sudden increase in urine output. Blood and urine tests were done which showed a serum sodium of 115 mEq/L , urine sodium of 160 mEq/L and a low serum ADH. What is the most likely diagnosis for this case? A. SIADH B. Cerebral Salt Wasting C. Central Diabetes Insipidus D. Nephrogenic Diabetes Insipidus E. Pseudohyponatremia

291 B. Cerebral salt wasting appears to be the result of hypersecretion of atrial natriuretic peptide and is seen primarily with central nervous system disorders including brain tumors, head trauma, hydrocephalus, neurosurgery, cerebral vascular accidents, and brain death. Hyponatremia is accompanied by elevated urinary sodium excretion (often >150 mEq/L), excessive urine output, hypovolemia, normal or high uric acid, suppressed vasopressin, and elevated atrial natriuretic peptide concentrations (>20 pmol/L). Thus, it is distinguished from SIADH, in which normal or decreased urine output, euvolemia, low uric acid, only modestly elevated urine sodium concentration, and an elevated vasopressin level occur.

A 10 year old boy presented with a 4 day history of fever, sore throat, dysphagia and odynophagia. On PE his VS are BP 100/60, PR 118, RR 21, T 38.6. There was a noted asymmetric tonsillar bulge with displacement of the uvula. The rest of the examination was normal. What is the most likely diagnosis for this case? A. Peritonsillar abscess B. Laryngeotracheobronchitis C. Retropharyngeal abscess D. Diphtheria E. Epiglottitis

296 A. Peritonsillar cellulitis/abscess, which is relatively common compared to the deep neck infections, is caused by bacterial invasion through the capsule of the tonsil, leading to cellulitis and/or abscess formation in the surrounding tissues. Clinical manifestations include sore throat, fever, trismus, and dysphagia. Physical examination reveals an asymmetric tonsillar bulge with displacement of the uvula. An asymmetric tonsillar bulge is diagnostic, but it may be poorly visualized because of trismus

Which of the following is not a parameter for physical maturity in the Ballard Maturational Assessment? A. Lanugo B. Posture C. Breast bud D. Plantar surface E. Ears

297 B. Posture, square window, arm recoil, popliteal angle, scarf sign and heel to ear are parameters for neuromuscular maturity

A 4 year old boy was brough in due to abdominal pain. The abdominal pain is paroxysmal in nature with the boy's leg drawn up and is accompanied by passage of bloody stools. On abdominal examination a palpable sausage shaped mass at the right upper quadrant was appreciated. Rectal examination revealed the presence of bloody mucus in the tactating finger. What is the most likely diagnosis for this case? A. Hypertrophic pyloric stenosis B. Inflammatory bowel disease C. Acute gastroenteritis D. Irritable bowel syndrome E. Intussusception

298 E. Intussusception occurs when a portion of the alimentary tract is telescoped into an adjacent segment. It is the most common cause of intestinal obstruction between 3 mo and 6 yr of age. In typical cases, there is sudden onset, in a previously well child, of severe paroxysmal colicky pain that recurs at frequent intervals and is accompanied by straining efforts with legs and knees flexed and loud cries. Palpation of the abdomen usually reveals a slightly tender sausage-shaped mass, sometimes ill defined, which may increase in size and firmness during a paroxysm of pain and is most often in the right upper abdomen

Ataxia, myoclonus ("dancing feet") and opsoclonus ("dancing eyes") are manifestations of a paraneoplastic syndrome of what form of malignancy? A. Wilm's Tumor B. Osteosarcoma C. Rhabdomyosarcoma D. Neuroblastoma E. Medulloblastoma

299 D. Neuroblastoma (NB) is an embryonal cancer of the peripheral sympathetic nervous system with heterogeneous clinical presentation and course. NB can present as a paraneoplastic syndrome of autoimmune origin manifesting as ataxia or opsomyoclonus (“dancing eyes and dancing feetâ€). In such cases, the primary tumor is in the chest or abdomen, and the brain is negative for tumor.

A 3 year old female presents with 1 week history of daily (quotidian) fever associated with arthritis of both ankles and her right knee. Her mother notes that she has decreased activity and also notes rashes that come and go in different parts of her body but not on the face. On physical examination, salmon colored patches were noted on the trunk and extremities. Palpation revealed splenomegaly. What is the diagnosis? A. Henoch Schoenlein Purpura B. Systemic Lupus Erythematosus C. Juvenile Rheumatoid arthritis D. Juvenile Dermatomyositis E. Dengue fever

3 C. Don't forget to study rheumatic diseases of childhood even if it's not your favorite topic. Familiarize yourself with the relatively common ones like JRA, SLE and dermatomyositis, HSP and scleroderma. Pay attention to the characteristics of the rash and other features. (Don't fall for the trap of rash + arthritis = SLE.. It's not always SLE!) JRA - Salmon colored patch, evanescent, spares the face SLE - Malar rash, photosensitive Dermatomyositis - heliotrope (periorbital) rash, gottron papules (rash on the knuckles) HSP - purpura in dependent portions For JRA, know the differences between subtypes (oligoarticular vs polyarticular vs systemic onset) This case in particular is a case of systemic onset Juvenile Rheumatoid arthritis (also called Still's Disease) Source: Nelson's Textbook of Pediatrics 19th edition p830

true of SIDS: A. Infants should be placed for sleep in supine or side sleeping position. B. Soft mattresses are preferred over firm ones. C. It is recommended that infants sleep in the same room as parents. D. Sleeping prone has consistently been shown to increase the risk for SIDS. E. Bed sharing has not been shown to increase the risk for SIDS.

336 D. Nelson Textbook of Pediatrics 18th edition p.1738

What is the only accurate method for diagnosis in sinusitis? A. transillumination of sinus cavities B. CT scan of sinuses C. rigid nasal endoscopy D. sinus aspirate culture E. clinical evaluation

337 D. Nelson Textbook of Pediatrics 18th edition p.1750

A 2-year-old child is admitted to your hospital. The child's pediatrician has been following the child for several days and has noted her to have had high fever, peeling skin, abdominal pain, and a bright red throat. You are concerned because two common pediatric problems that could explain this child's condition have overlapping presenting signs and symptoms. Which of the following statements comparing these two diseases in your differential is true? A. Neither has a cardiac complication. B. Serologic tests are helpful in diagnosing both. C. Only one of the diseases has mucocutaneous and lymph node involvement. D. Pharyngeal culture aids in the diagnosis of one of the conditions. E. A specific antibiotic therapy is recommended for one of the conditions, but only supportive care is recommended for the other.

30 D. The two conditions in consideration are Kawasaki disease and scarlet fever caused by a group A beta-hemolytic streptococci (GABHS). Kawasaki disease is an acute febrile illness of unknown etiology and shares many of its clinical manifestations with scarlet fever. Scarlatiniform rash, desquamation, erythema of the mucous membranes that produces an injected pharynx and strawberry tongue, and cervical lymphadenopathy are prominent findings in both. The most serious complication of Kawasaki disease and scarlet fever is cardiac involvement. Erythrogenic toxin-producing group A beta- hemolytic streptococci is the agent responsible for scarlet fever. Isolation of the organism from the nasopharynx and a rise in antistreptolysin titers will confirm the diagnosis. Serologic tests for a variety of infectious agents, both viral and bacterial, have been negative in Kawasaki disease. Rheumatic heart disease is a serious sequela of streptococcal pharyngitis, which can be prevented by appropriate treatment with penicillin. Coronary artery aneurysm and thrombosis are the most serious complications of Kawasaki disease. The current approach to treatment of Kawasaki disease, which includes specific therapy with aspirin and gamma-globulin administered within a week of the onset of fever, appears to lower the prevalence of coronary artery dilatation and aneurysm to shorten the acute phase of illness

A 4 y/o female was referred to your clinic secondary to a mass occupying the nasopharyngeal area. this is accompanied by congestion, recurrent epistaxis and multiple cranial nerve palsies. Patient is lethargic with complains of headache and recurrent projectile vomitting. what is the most likely diagnosis for this patient? A. rhabdomyosarcoma B. neuroblastoma C. medulloblastoma D. chloroma E. all of the above ?

301 A. SIMILAR TO PREVIOUS BOARD EXAM CONCEPT/PRINCIPLE. Rhabdomyosarcoma (RS) is the most common pediatric soft tissue sarcoma. They are usually found in the head and neck, genitourinary, and retroperitoneal area. nasopharyngeal RS presents with mass accompanied by epistaxis, congestion, dysphagia. regional extension leads to cranial nerve palsies, blindness and signs of increased intracranial pressure. medullablastoma is the most common malignant primary brain tumor in pediatrics. it is intraaxial and will not present with mass in the nasopharyngeal area. its most common site is the cerebellar hemisphere. Neuroblastoma seldomly seen in head and neck region. along with wilms, it usually present with an abdominal mass. chloroma is a solid collection of leukemic cells in acute myelogenous leukemia. it was not stated in the case if the patient presents with signs and symptoms of leukemia.

what vitamin/mineral deficiency is associated with intention tremor, decreased proprioception, nystagmus, dysarthria, and retinopathies? A. Vitamin A B. Vitamin B6 C. Vitamin E D. Zinc E. Selenium?

302 C. Vitamin A - nyctalopsia, xeropthalmia, bitot spits, epiphyseal bone abnormalities. Vitamin B6 - irritability, convulsion, hypochromic anemia, oxaluria, peripheral neuritis. Zinc - abdominal pain, diarrhea, vomitting, acrodermatitis enterohepatica. Vitamin E deficiency is associated with severe neurologic manifestation. it usually presents with limb and truncal ataxia, ophthalmoplegia, positive rhomberg test, dysarthria

A 12 y/o male went for consult secondary to tea colored urine. Physical examination: BP 150/90, PR 102 RR 21 T 37.0C. Pale palpebral conjunctiva, anicteric sclerae, clear breath sounds, soft, nontender abdomen. There was grade 1 bipedal edema and with slight periorbital swelling. lab shows the following result (+) ASO, low C3, and subneprotic range proteinuria. patient was hospitalized for a week and was successfully treated. he was lost to follow up, and just come back after 3 months. laboratories still show persistently low C3. what is your diagnosis ? A. FSGS B. MPGN C. APSGN D. MGN E. RPGN?

303 B. MPGN is the most common cause of chronic glomerulonephritis in children and adolescent. Its presentation is very similar to APSGN however, the only difference is a persistently low C3 even after 2 months.

A 4 months old infant develops protracted projectile vomiting, diarrhea and lethargy 2 hours after ingesting a powdered milk formula. What is the most likely diagnosis? A. Food protein induced enterocolitis B. Allergic eosinophilic esophagitis C. allergic eosinophilic gastroenteritis D. anaphylaxis E. none of the above

304 A. Food protein-induced enterocolitis syndrome typically manifests in the first several months of life with irritability, protracted vomiting and diarrhea, not infrequently resulting in dehydration. Vomiting generally occurs 1-3 hr following feeding, and continued exposure may result in bloody diarrhea, anemia, abdominal distention, and failure to thrive. Symptoms are most commonly provoked by cow's milk- or soy protein-based formulas but occasionally result from food proteins passed in maternal breast milk.

Which of the following is a false statement regarding DiGeorge syndrome? A. Very low absolute lymphocyte count B. Equal male and female predisposition C. Normal levels of serum Immunoglobulins D. Variable hypoplasia of the thymus and parathyroids is common E. none of the above

305 A. the absolute lymphocyte count is usually only moderately low for age. Lymphocyte responses to mitogen stimulation are absent, reduced, or normal, depending on the degree of thymic deficiency. SIMILAR TO PREVIOUS BOARD EXAM CONCEPT/PRINCIPLE

Recurrent meningococcal infection is correlated with what type of immunologic disorder? A. T cell defect B. B cell defect C. Combined T and B cell defect D. Phagocyte function defect E. None of the above

306 E. Congenital deficiencies of the terminal components of complement (C5, C6, C7, or C8) have been associated with repeated neisseria infection. SIMILAR TO PREVIOUS BOARD EXAM CONCEPT/PRINCIPLE

what is the most common complication of acute bacterial meningitis ? A. Recurrent seizures B. Mental retardation C. Decreased hearing acuity D. Behavioral changes E. Anterograde amnesia

307 C. Hearing deficits are most common complication followed by neuropsychiatric disturbances ( more predominant in H influenza infection). Nelson 17th ed page 594.

the abnormal proliferation of metanephric blastema is a characteristic finding of which malignancy? A. Neuroblastoma B. ARPKD C. Wilms tumor D. Rhabdomyosarcoma E. Medullary sponge kidney

308 C. nephroblastoma or wilms tumor is the 2nd most common malignancy in the pediatric population. It is characterized by proliferation of metanephric blastema.

which of the following is not associated with Kawasaki disease? A. Gallbladder hydrops B. Otitis media C. splenomegaly D. Nondeforming arthritis E. None of the above

309 C. Nelson 18th ed page 1037. other features of kawasaki disease are extreme irritability, aseptic meningitis, diarrhea, hepatitis, gallbladder hydrops, urethritis, otitis media, and arthritis. ( SIMILAR TO PREVIOUS BOARD EXAM CONCEPT/PRINCIPLE)

A 3-year-old girl presents with generalized edema shortly after recovery from an upper respiratory infection. Laboratory studies reveal marked albuminuria, as well as hypoalbuminemia and hyperlipidemia. Prior similar episodes responded to steroids. The most likely diagnosis is A. FSGS B. membranous GN C. lipoid nephrosis D. PSGN E. RPGN

31 C. Our Pediatrics exam featured numerous renal pathology questions as the stem case. It is therefore essential to nail the diagnosis at the start to keep you on track for the next questions. The combination of generalized edema, massive proteinuria, hypoalbuminemia, and hyperlipidemia constitutes the nephrotic syndrome, the prototype of which is minimal change disease (lipoid nephrosis). This disorder characteristically occurs in young children and demonstrates intracytoplasmic lipid in the proximal convoluted tubules, a paucity of glomerular abnormalities by light microscopy, and fusing (absence) of the epithelial foot processes by electron microscopy.

A 2-mo-old infant has a history of infrequent bowel movements and poor weight gain. Physical examination reveals a thin, poorly nourished child with abdominal distention. Rectal examination reveals normal tone, and the infant subsequently passes an explosive, foul-smelling stool.which of the following procedures is not useful in the diagnosis of this condition? A. rectal biopsy B. barium enema C. proctosigmoidoscopy D. rectal motility studies E. none of the above

310 C. the gold standard for the diagnosis of hirschprung disease is rectal biopsy. Colonoscopy will not yield any additional information since the classic presentation ( delayed passage of meconium) along with biopsy or barium enema will suffice.

a 3 month old child was brougt for consult due to staccato cough with mild respiratory distress. She is afebrile. Lung auscultation showed occasional bibasilar wheezing and fair entry. What is the most likely etiologic agent? A. Chlamydia trachomatis B. Mycoplasma pneumoniae C. Influenza virus D. streptococcus pneumoniae E. Listeria monocytogenes

312 A. stacatto cough is characteristic of chlamydia trachomatis infection.

Jaundice is most likely physiologic in a term infant in which of the following? A. serum bilirubin level <5mg/dL/24 hours in the first 2-4 days of life B. jaundice at 18 hours of life C. B2 greater than 1.5mg/dL D. jaundice at 12 days of life E. none of the above

314 A. parameters of physiologic jaundice: visible on 2nd - 3rd day, peaks on 4th day at 5-6mg/dl, decreasees on 5th -7th day at <2mg/dl.

The most common abnormality associated with UTI is: A. ureteropelvic junction obstruction B. urolithiasis C. vesicoureteral reflux D. posterior urethral reflux E. None of the above

315 C. Nelson 18th ed p 2228

the success rate for establishing good bile flow after Kasai procedure is much higher if it is done before in what age? A. 1 year B. 8mo C. 5mo D. 4mo E. 2mo

316 E. Nelson 18th ed p 1673

what is the single most predictive factor in acute lymphoblastic leukemia ? A. Age at onset B. Treatment C. Initial leukocyte count D. Chromosomal abnormalities E. None of the above

319 B. Nelson 18th ed chapter 487

Consistent with croup: A. It is a clinical diagnosis that doesn't require a neck x-ray. B. Drooling is present and the neck is hyperextended. C. It requires visualization of large cherry red swollen epiglottis on laryngoscopy. D. Classic x-ray finding is the thumb sign. E. The child may assume a tripod position.

338 A. The rest of the choices refer to acute epiglottitis. Nelson Textbook of Pediatrics 18th edition p.1763

The most common pediatric soft tissue sarcoma is: A. synovial sarcoma B. fibrosarcoma C. rhabdomyosarcoma D. malignant fibrous histiocytoma

339 C. Nelson Textbook of Pediatrics 18th edition p.2145

4-year-old boy develops weakness of proximal lower back and extremity muscles, manifested by lordosis, a waddling gait, and the need to push on his knees in order to stand. Examination reveals proximal muscle weakness and bilateral enlargement of the calves. His younger brother has begun to display similar findings, as has his older half-brother, who has the same mother. Serum CK is markedly elevated. Which of the following is characteristic of this disorder? A. Aberrant protein coded by a very small gene sequence on the Y chromosome B. Autosomal dominant mode of inheritance C. Mitochondrial inheritance D. Regression of findings in late adolescence and adult life E. Total absence or marked decrease of an important gene product

32 E. The clinical picture is that of Duchenne muscular dystrophy, the most common and most severe of the muscular dystrophies. This X-linked disorder is characterized by failure of synthesis of dystrophin, most often because of deletion of one or many exons in the DMD gene. Patients manifest with proximal muscle weakness, progressing to muscle necrosis. Serum CK is markedly increased. Compensatory hypertrophy is followed by pseudohypertrophy, in which necrotic muscle is replaced by fat and connective tissue. Most patients become wheelchair- bound and die of respiratory or cardiac failure in their late teenage years or in their early twenties.

a 9 y/o male patient brought for consult secondary to recurrent seizures. Upon secondary survey, you noted multiple melanotic macules around his trunk and red minute nodules on his nose and malar area. what is the most likely diagnosis? A. Von Hippel Lindau disease B. NF1 C. NF2 D. Tuberous sclerosis E. sturge weber syndrome

320 D. presence of ashleaf spots and adenoma sebaceum seen in this patient on the background recurrent seizures suggest tuberous sclerosis as the primary diagnosis.

Which paranasal sinuses are pneumatized at birth? A. maxillary B. frontal C. ethmoidal D. sphenoidal

321 C. SIMILAR TO PREVIOUS BOARD EXAM CONCEPT/PRINCIPLE. Nelson Textbook of Pediatrics 18th edition p.1749

Fetuses infected at that age of gestation may have maximal interruption with lumb development in congenital varicella syndrome? A. 6-12 weeks B. 16-20 weeks C. 20-24 weeks D. 24-28 weeks E. 28-32 weeks

322 A. Congenital varicella syndrome at 16-20 weeks: eye and brain development are affected. Nelson Textbook of Pediatrics 18th edition p.1368

The most frequently reported cardiac defect in Congenital Rubella Syndrome: A. Tetralogy of fallot B. right pulmonary artery stenosis C. left pulmonary artery stenosis D. patent ductus arteriosus E. pulmonic valve stenosis

323 D. Nelson Textbook of Pediatrics 18th edition p.1340

What is the most common cause of death in measles? A. encephalitis B. hemorrhagic skin eruption C. myocarditis D. acute otitis media E. pneumonia

324 E. Hemorrhagic skin eruption is rare; it is seen in hemorrhagic/black measles. Myocarditis is also a rare complication of measles. Acute otitis media is the most common complication of measles infection. Nelson Textbook of Pediatrics 18th edition p.1334

Ten years after a primary measles infection, a 13- year-old boy was diagnosed to have subacute sclerosing panencephalitis. He now exhibits repetitive myoclonic jerks and involuntary movements, but consciousness is maintained. He is now in this stage of SSPE: A. I B.II C.III D. IV E. V

325 B. SSPE stage I - subtle changes in school performance; stage II - massive myoclonus; stage III - choreoathetosis, rigidity, dystonia with deteriorating consciousness; stage IV - loss of cortical centers that support breathing, HR, BP. There is no stage V in SSPE. Nelson Textbook of Pediatrics 18th edition p.1335

Microorganism associated with bullous myringitis: A. S. aureus B. M. pneumoniae C. H. influenzae D. S. pneumoniae E. N. meningitidis

326 B. Nelson Textbook of Pediatrics 18th edition p.1279

What is/are the easiest and most reliable indicators of Hirschsprung disease? A. rectal manometry B. rectal suction biopsy C. barium enema D. A & B E. all of the above

327 D. But rectal suction biopsy is the procedure of choice. Nelson Textbook of Pediatrics 18th edition p.1566

Which is a true statement? A. In asthma, the patient exhibitis a polyphonic wheezing. B. Stridor is heard in intrathoracic airway obstruction. C. Most wheezing in infants is caused by anatomic abnormalities. D. Infants with acute bronchiolitis usually also exhibits diarrhea and/or vomiting. E. Bronchiolitis is more common in breastfed females.

328 A. Stridor is heard in extrathoracic airway obstruction. Most wheezing in infants is caused by inflammation. Infants with acute bronchiolitis usually do not exhibit diarrhea or vomiting. Bronchiolitis is more common in males, babies not breastfed, and those living in crowded conditions. Nelson Textbook of Pediatrics 18th edition pp.1774-1775

The following clinical manifestations of infective endocarditis are described accurately: A. Janeway lesions are painless, small, erythematous/hemorrhagic lesions on the palms and soles. B. Osler nodes are tender, pea-sized intradermal nodules in the pads of fingers and toes. C. Splinter hemorrhages are linear lesions beneath nails. D. A & B E. all of the above

329 E. Nelson Textbook of Pediatrics 18th edition p.1954

A 3-year-old boy, an inner city resident, has multiple bony abnormalities, including bowlegs and knock-knees, thickening of the skull with frontal bossing, knobby deformities of the costochondral junctions and, at the ends of the long bones, distortion of the rib cage with flaring over the diaphragm, and pigeon-breast deformity. A decrease in which of the following is characteristic of this condition? A. Bony osteoblastic activity B. Calcification of osteoid C. Release of parathyroid hormone D. Serum alkaline phosphatase activity E. Synthesis of osteoid

33 B. The child has rickets, which is caused by failure of action of calcitriol (1,25-dihydroxycholecalciferol), the active form of vitamin D. The bony abnormalities in rickets are caused by failure of osteoid matrix to calcify, leading to excess accumulation of osteoid, increased thickness of the epiphyseal growth plates, and other skeletal deformities. Many of the effects of calcitriol deficiency are mediated by increased release of parathyroid hormone (PTH). PTH stimulates bony osteoblastic activity, which is mirrored by an increase in serum alkaline phosphatase.

Why should milk consumption be limited in the treatment of a child with iron deficiency anemia? A. to increase the amount of iron rich foods consumed B. to decrease blood loss from intolerance to cow's milk proteins C. for optimal absorption of the oral iron D. A & B E. all of the above

330 D. SIMILAR TO PREVIOUS BOARD EXAM CONCEPT/PRINCIPLE. Nelson Textbook of Pediatrics 18th edition p.2016

A 10-year-old boy presents with a large abdominal mass. Computed tomography of the abdomen reveals enlarged retroperitoneal and mesenteric lymph nodes. Biopsy of one of the involved lymph nodes shows a starry-sky appearance, with prominent debris-containing macrophages. Which of the following statements about this disorder is correct? A. The sporadic form is most frequently associated with EBV. B. The disorder is considered to be a derivative of Hodgkin lymphoma, lymphocyte depletion subtype. C. The most common cytogenetic change is t(8;14), with increased expression of c-myc. D. The tumor cells are derivatives of T lymphocytes. E. The tumor most often has an indolent clinical course.

34 C. Expect many Pathology-type questions in Pediatrics. The typical cytogenetic change associated with Burkitt lymphoma is t(8;14) with increased expression of the c-myc gene. This disorder is an aggressive B-cell non-Hodgkin ymphoma most commonly affecting children. The endemic (African) form is characterized by the involvement of the maxilla or mandible, whereas the sporadic (Western) form usually involves the abdominal organs. Burkitt lymphoma is generally a rapidly growing neoplasm, and the endemic form has a frequent association with EBV.

The most common cause of suppurative arthritis in infants and children is: A. E. coli B. S. pneumoniae C. Candida D. S. aureus

340 D. Nelson Textbook of Pediatrics 18th edition p.2845

A newborn whose mother is both HBsAg and HBeAg (+) should be given Hepa B vaccine and HBIG simultaneously at the latest by how many hours after birth? A. 36 B. 24 C. 12 D. 6

341 C.

A 5 year old girl was brought to the clinic due to a 2 week history of fever, joint pains, and loss of appetite. On PE: enlarged cervical lymph nodes and hepatosplenomegaly. CBC showed Hgb 6 g/dL, Hct 18%, WBC 30x10^9/L, segmenters 5%, lymphocytes 95%, platelets 50x10^9/L, (+) blasts in the peripheral smear. The most likely diagnosis is: A. Neuroblastoma B. Non-Hodgkin’s lymphoma C. Acute leukaemia D. Rhabdomyosarcoma

342 C.

A 4 year old male had 1 month on and off cough, weight loss, and afternoon rise of temperature. His grandfather has apical infiltrates which are most likely tuberculous in origin. What is the initial work-up of this child? A. Mantoux test and chest X-ray B. Chest CT scan C. Bactec TB culture D. Sputum AFB smear

343 A. Sputum AFB is not usually a part of initial work-up for TB in young children who cannot yet follow instructions and give adequate samples for testing.

A 3 year old girl arrives at the ER with a one day history of fever and macular rash that evolved into petechiae and purpura. She was hypotensive with cold clammy extremities. What is the most likely diagnosis? A. ITP B. Meningococcemia C. Kawasaki disease D. Dengue hemorrhagic fever

347 B. The fulminant course of this disease, as well as the presence of multiple purpura, make meningococcemia the most likely diagnosis among the given choices.

A 5 year old girl presents with a 2 month history of progressive difficulty in ambulation, particularly in climbing up the stairs. After going on a field trip, she came home with an erythematous rash in the malar area and violaceous discolouration with swelling over the periorbital areas. On the day of consult, she was noted to have papules over the finger and elbow joints. What is the most likely diagnosis? A. SLE B. Juvenile idiopathic arthritis C. Juvenile dermatomyositis D. Systemic scleroderma

348 C. Difficulty in ambulating, particularly in movements which need proximal muscle strength, plus the presence of violaceous discolouration around the eyes (Heliotrope rash), and papules over the finger and elbow joints (Gottron rash) are characteristic of Dermatomyositis.

A 10 year old boy is noted to have hematuria and proteinuria on routine PE. He is asymptomatic, and examination findings are normal. Blood chemistry results were normal, but 24 hr urine specimen reveals 2g of protein. A renal biopsy is performed which reveals a mesangial proliferative glomerulonephritis with bright IgA deposition on immunofluorescence. Which of the following statements is true regarding this child’s form of glomerulonephritis? A. This disease is more common in females B. The primary treatment is blood pressure control C. Progressive kidney disease occurs in a majority of children D. C3 value is usually low

349 B. Diagnosis is IgA nephropathy

A 2-year-old boy presents with his third bone fracture within the past several months. There is no history or evidence of trauma. Several close family members have been similarly affected. The child is small for his age, and the sclerae are tinged a bluish color. Radiographs reveal generalized osteopenia and evidence of multiple fractures, both old and new. Which of the following is the usual mode of inheritance of this disorder? A. Autosomal dominant B. Autosomal recessive C. X-linked dominant D. X-linked recessive E. Mitochondrial

35 A. Osteogenesis imperfecta (or brittle bone disease) is an autosomal dominant disorder characterized by multiple fractures with minimal trauma. It is caused by mutations in either of the genes that code for type I collagen and is manifested by connective tissue abnormalities affecting the bones, teeth, skin, and eyes. The sclerae can appear blue due to translucency of the thin connective tissue overlying the choroid. The disorder occurs in several variants defined by the age of onset and the severity of the fractures. In the less obvious cases, the principal differential diagnostic consideration is child abuse.

A 5 year old girl presents with tea coloured urine, oliguria and body edema 2 weeks after being treated for streptococcal pharyngitis. Her C3 level is noted to be very low. When should this patient’s C3 level be repeated in order to confirm your suspected diagnosis? A. In 1 week B. In 2 weeks C. In 4 weeks D. In 8 weeks

350 D. C3 levels usually return to normal after 8 weeks in patients with post-streptococcal glomerulonephritis.

A 3 year old boy presents at the ER with a 3 day history of abdominal pain and difficulty walking. Abnormal findings include BP of 120/80, diffuse abdominal tenderness, purpuric rash of the legs and ankles, and swelling and tenderness over the ankles. The most likely diagnosis is: A. SLE B. Kawasaki disease C. Juvenile idiopathic arthritis D. Henoch Schonlein purpura

351. D.Findings which are characteristic of HSP are purpuric rash, especially over the lower extremities; abdominal pain and other GI symptoms; joint pains and swelling; and renal involvement.

All of the following statements about hemolytic- uremic syndrome are true except: A. HUS is the most common cause of acute renal failure in young children B. Verotoxin elaborated by E. coli O157:H7 initiated endothelial injury in HUS C. HUS always presents after an enteritis with diarrhoea D. The diagnosis of HUS requires microangiopathic haemolytic anemia, thrombocytopenia, and acute renal failure

352 C.

A 7 month old girl presents to the ER with gradual onset of fever, lethargy, and irritability. Her immunisations are up to date. Examination reveals a febrile infant who is crying inconsolably. A lumbar tap is performed with the following result: WBC 1500/mm3, 84% segmenters; glucose 12 mg/dL; protein 70 mg/dL. Gram stain is negative. The most likely etiologic agent for this infection is: A. An enterovirus B. Haemophilus influenzae type b C. Neisseria meningitides D. Group B streptococci

353 C. Based on the CSF findings, this is most likely a bacterial infection (predominant segmenters, elevated protein, decreased sugar). Group B strep is unlikely since the patient is already 7 months old. Hib is also unlikely because it was noted that her immunisation history is complete.

A 3 year old girl has a 2 week history of fever associated with headache, lethargy and vomiting. She has a history of perioral cyanosis and dyspnea with exertion ever since she was an infant. She suddenly has a 10 minute tonic-clonic seizure. The child is obtunded afterwards, with a temperature of 38.2C, PR 118/min, BP of 100/70. Perioral cyanosis is noted at rest. A harsh pan systolic murmur is heard along the left sternal border. Fundoscopy reveals papilledema. She is also noted to have right sided weakness. The most likely cause of her hemiparesis is: A. A brain tumor B. Neurocysticercosis C. Meningitis D. A brain abscess

354 D. Based on the patient’s history, she most likely has Tetralogy of Fallot, which predisposes her to having a brain abscess. Other findings supporting this aetiology is the presence of increased ICP (papilledema) and a focal neurologic deficit, which makes meningitis less likely.

A 6 month old infant develops protracted projectile vomiting and lethargy about 2 hours after ingesting a milk formula. The most likely diagnosis is: A. Generalized anaphylaxis B. Milk induced enterocolitis syndrome C. Allergic eosinophilic esophagitis D. Allergic eosinophilic gastroenteritis

356 B.

All of the following are prominent features of Wiskott-Aldrich syndrome except: A. Atopic dermatitis B. Thrombocytopenia C. Autosomal dominant inheritance D. Recurrent infections with encapsulated bacteria

359 C. It’s an X-linked recessive syndrome.

A fully immunized 2-year-old presents to the emergency room with several days of low-grade fever, barking cough, and noisy breathing. Over the past few hours he has developed a fever of 40°C (104°F) and looks toxic. He has inspiratory and expiratory stridor. The family has not noticed drooling, and he seems to be drinking without pain. Direct laryngoscopy reveals a normal epiglottis. The management of this disease process includes which of the following? A. Intubation and intravenous antibiotics B. Inhaled epinephrine and oral steroids C. Intravenous methylprednisolone D. Observation in a cool mist tent E. Oral antibiotics and outpatient follow-up

36 A. Bacterial tracheitis is an uncommon but severe and life-threatening sequela of viral laryngotracheobronchitis. The typical story is that presented in the case, with several days of viral upper respiratory symptoms, followed by an acute elevation of temperature and an increase in respiratory distress. Inspiratory stridor is typical in croup; the biphasic stridor and high fever in this patient should be a clue to consider alternative diagnoses. Children may also present acutely and without the initial viral symptoms. The differential must include epiglottitis; the lack of drooling and dysphagia (and the rarity of epiglottitis since the introduction of the Haemophilus influenza B vaccine) help make this a case of tracheitis. Management for tracheitis includes establishing an airway with endotracheal intubation and IV antibiotics. Special attention is focused on preservation of the airway, as even intubated children with tracheitis can have secretions thick and copious enough to occlude the airway. Oral antibiotics and outpatient follow-up for a patient with respiratory distress and toxic appearance are never appropriate. Inhaled epinephrine and oral steroids as well as observation in a cool mist tent suggest a diagnosis of croup, a disease that presents without high fever but with inspiratory stridor a few days after an upper respiratory infection. In the case presentation, the fever and toxic appearance differentiate this condition from viral croup. Intravenous methylprednisolone is a component of treatment for asthma, a diagnosis unlikely in a patient with high fever as outlined.

The most important extra medullary site of relapse in childhood ALL is: A. Kidney B. Lung C. Heart D. CNS

360 D. The CNS and the testes are the two most important sites of extra medullary relapse of ALL.

Patients with cystic fibrosis are susceptible to which of the following? A. Pancreatic insufficiency B. Recurrent pneumonia C. Rectal mucosal prolapse D. A and B E. All of the above

361 E. Nelsons 19th pg 1361

The period of communicability for measles is: A. 7 days before until 7 days after the onset of the rash B. 4 days before until 4 days after the onset of the rash C. 1-2 days before until 5 days after the onset of the rash D. 1-2 days before until 7 days after the onset of the rash E. Unknown

366 B. A - rubella; C - mumps (1-2 days before until 5 days after the onset of the swelling; D - varicella; E - roseola Topnotch

What is the strongest stimulus for the closure of the ductus arteriosus? A. Increase in O2 saturation of the systemic circulation B. Increase in the pressure of the systemic circulation C. Decrease in the O2 saturation of the pulmonary circulation D. Decrease in the pressure of the pulmonary circulation E. A and D

367 A. Increase in O2 saturation of the systemic circulation is the strongest stimulus for the constriction of the ductal smooth muscles which leads to the closure of the ductus. Topnotch

The pentalogy of cantrell consists of the following EXCEPT A. Omphalocele B. Sternal cleft C. Ectopia cordis D. Posterior diaphragmatic hernia E. VSD

368 D. Pentalogy of Cantrell consists of ectopia cordis, midline supraumbilical abdominal defect, deficiency of the anterior diaphragm, defect of the lower sternum, and an intra- cardiac defect (either a ventricular septal defect, tetralogy of Fallot or diverticulum of the left ventricle). Nelsons 19th pg 1599

A 2 hour old neonate was brought to the ER for cyanosis, tachypnea, grunting and retractions. On further history, the neonate was delivered by a hilot after a prolonged labor of more than a day. CXR was done which showed patchy infiltrate and coarse streaking of both lung fields. What is your diagnosis? A. Hyaline membrane disease B. Neonatal pneumonia C. Meconium aspiration syndrome D. Transient tachypnea of the newborn E. Persistent pulmonary hypertension of the newborn

369 C. The resulting small airway obstruction may produce respiratory distress within the 1st hours, with tachypnea, retractions, grunting, and cyanosis observed in severely affected infants. The typical chest roentgenogram is characterized by patchy infiltrates, coarse streaking of both lung fields, increased anteroposterior diameter, and flattening of the diaphragm.A normal chest roentgenogram in an infant with severe hypoxemia and no cardiac malformation suggests the diagnosis of pulmonary hypertension Nelsons 19th pg 591

A 15-year-old athlete with knee pain and severe point tenderness at the tibial tubercle most likely has A. Osgood-Schlatter disease B. Legg-Calve-Perthes disease C. Slipped capital femoral epiphysis (SCFE) D. Tibial torsion E. Charcot-Marie-Tooth Disease

37 A. Osgood-Schlatter disease is a very common problem most often seen in athletically active adolescents. This disorder is characterized by ossification in the distal patellar tendon at the point of its insertion onto the tibial apophysis. This disorder is thought to result from mechanical stress on the tendinous insertional area. X-ray views of the involved knee show a characteristic irregularity in the insertional area and often show separately discrete ossicles within the tendon itself. The disease will present with severe local pain and exquisite tenderness in the area of the tibial tubercle. In almost every case, symptoms do regress after skeletal maturity or the discontinuance of active athletic participation. In rare cases, persistive symptoms into adulthood can occur. Legg-Calve-Perthes disease, also known as coxa plana, is a condition of the pediatric hip characterized by a flattened, misshapen femoral head. The etiology of the problem is related to osteonecrosis of the proximal femoral epiphysis and is thought to result from vascular compromise. Legg-Calve-Perthes disease generally presents in children, usually males, between the ages of 4 and 8 years old. Presenting symptoms generally include groin or knee pain, decreased hip range of motion, and a limp. A slipped capital femoral epiphysis (SCFE) is an acquired disorder of the epiphysis thought to be associated with weakness in the perichondrial ring of the growth plate. Children within the ages of 10 to 16 years old are noted to have the displacement of the epiphysis on the femoral neck. In most cases, there is no identifiable trauma history. It is associated with African American heritage and obesity and is somewhat more common in boys than in girls. Tibial torsion is the most common cause of an intoeing gait. This is most frequently noted in 1- and 2-year-old children. This is often bilateral. Although occasionally intoeing can be marked, pediatric tibial torsion will completely resolve without treatment in almost every case.

An 14/M presented to you with headaches, visual disturbances, growth failure and delayed sexual maturation. A CT scan was done which revealed a large and heterogenous mass, with solid and cystic components, in the suprasellar region. What is your diagnosis? A. Pituitary adenoma B. Craniopharyngioma C. Germ cell tumor D. Ependymal tumor E. Astrocytoma

371 B. Children with craniopharyngioma often present with endocrinologic abnormalities such as growth failure and delayed sexual maturation. Visual changes can occur and may include decrease acuity or visual field deficits. These tumors are often quite large and heterogenous, displaying both solid and cystic components, and occur within the suprasellar region. They are minimally invasive, adhere to adjacent brain parenchyma, and engulf normal brain structures. Nelsons 19th pg 1752

The caudal end of the neural tube closes during: A. 23rd day B. 27th day C. 4th month D. 8th month E. None of the above

372 B. Normally, the rostral end of the neural tube closes on the 23rd day and the caudal neuropore closes by a process of secondary neurulation by the 27th day of development Nelsons 19th pg 1998

Perinatal asphyxia severe enough to result in acute neurologic injury if all of the following variables are present EXCEPT? A. Umbilical artery blood pH <7 B. 3 minute Apgar of 0-3 C. Seizures, coma or hypotonia D. Multiple organ involvement E. No exception

373 B. > 5 minute Apgar of 0-3

A 9/M presented with a 3 day history of symmetric multiple non-blanching purple discolorations in the lower extremities and buttocks. This was associated with postprandial abdominal pain, arthralgia and hematuria. What is your diagnosis? A. HSP B. Wegener granulomatosis C. Churg-Strauss syndrome D. Microscopic polyangiitis E. None of the above

375 A. The diagnosis of HSP is a clinical one and is often straightforward when the typical rash is present. Classification criteria for HSP: AMERICAN COLLEGE OF RHEUMATOLOGY CLASSIFICATION CRITERIA Two of the following criteria must be present: • Palpable purpura • Age at onset ≤20 yr • Bowel angina (postprandial abdominal pain, bloody diarrhea) • Biopsy demonstrating intramural granulocytes in small arterioles and/or venules EUROPEAN LEAGUE AGAINST RHEUMATISM/PEDIATRIC RHEUMATOLOGY EUROPEAN SOCIETY CRITERIA Palpable purpura (in absence of coagulopathy or thrombocytopenia) and one or more of the following criteria must be present: Dffiuse abdominal pain, Arthritis or Arthralgia, Biopsy of affected tissue demonstrating predominant immunoglobulin A deposits. Nelson 19th Ed. pg 870.

A 9 month old was brought to you for recurrent pneumonia and recurrent bacterial infections of the skin. On futher history, it was noted that the infant developed an infection of the cord stump due to its delayed separation? What is your diagnosis? A. SCID B. Chronic granulomatous disease C. Chediak-Higashi syndrome D. Leukocyte adhesion deficiency I E. Complement deficiency

376 D. Children with LAD-1 present in infancy with recurrent, indolent bacterial infections of the skin, mouth, respiratory tract, lower intestinal tract, and genital mucosa. They may have a history of delayed separation of the umbilical cord, usually with associated infection (omphalitis) of the cord stump. Nelsons 19th pg 741

A 16/M presents with acute onset scrotal pain and swelling. Upon further history, the patient also had nausea, vomiting and fever. On PE, the scrotum did not transilluminate, Prehn sign was positive and you were not able to elicit the cremasteric reflex. What is your diagnosis? A. Testicular torsion B. Epididymitis C. Varicocele D. Hydrocele E. None of the above

378 B. Prehn sign: relief when scrotum is lifted. Prehn sign positive in epididymitis and negative in torsion Topnotch

The following is/are TRUE regarding breastmilk and milk formula? A. Milk formula has higher digestibility and better absorption B. Breastmilk contains higher amounts of iron compared to milk formula C. Breastmilk is more allergenic than milk formula D. B and C E. All of the above

379 C. Breastmilk has higher digestibility and better absorption; Milk formula has a little higher iron than breastmilk Topnotch

Which of the following indicates a poor prognosis in a patient with neuroblastoma? A. Hyperdiploid DNA B. Absence of N-myc amplification C. Age 1 year D. Age >13 years E. Low serum ferritin levels

38 D. A number of biologic variables have been studied in children with neuroblastoma. An open biopsy is often required to provide sufficient tissue for analysis. The presence of hyper-diploid tumor DNA is associated with a favorable prognosis, whereas N-myc amplification is associated with a poor prognosis regardless of patient age. The Shimada classification describes tumors as having either favorable or unfavorable histologic features based on the degree of differentiation, the mitosis-karyorrhexis index, and the presence or absence of schwannian stroma. In general, children of any age with localized neuroblastoma and infants 1 year of age with advanced disease and favorable disease characteristics have a high likelihood of disease free survival. By contrast, older children with advanced disease have a significantly decreased chance for cure even with intensive therapy. For example, aggressive multiagent chemotherapy has resulted in a 2-year survival rate of approximately 20% in older children with stage IV disease. Neuroblastoma in the adolescent has a worse long-term prognosis regardless of stage or site and, in many cases, a more prolonged course. High serum ferritin levels portend a worse prognosis.

A 5-year-old boy diagnosed with poststreptococcal glomerulonephritis was admitted to the hospital several weeks ago. Over the last several weeks, his clinical state has not improved. Severe oliguria has developed, his serum creatinine has continued to rise, and his glomerular filtration rate has decreased by 50% since his admission to the hospital. Which of the following is the most likely diagnosis? A. Alport syndrome B. Membranoproliferative glomerulonephritis C. Membranous glomerulonephritis D. Rapidly progressive glomerulonephritis E. Renal papillary necrosis

39 D. The clinical description is that of RPGN, which is defined as the nephritic syndrome that progresses rapidly to fulminant renal failure within weeks or months. RPGN is classified into three categories: (1) antiglomerular basement membrane (anti- GBM) antibody disease, (2) immune complex deposit disease, and (3) pauciimmune disease (ANCApositive). Characteristic histologic findings in RPGN include crescents between the Bowman capsule and the glomerular tuft. In approximately 50% of cases of RPGN, the disease is of poststreptococcal etiology. It should be noted, however, that the most common outcome of poststreptococcal glomerulonephritis is complete recovery, and only a small minority of patients progress to RPGN.

A 6-year female was brought to your clinic due to symptoms of excessive urination with increased thirst, interfering with her appetite, weight gain and growth. Urination persisted despite fluid deprivation. Following administration of desmopressin, urine remained diluted with low specific gravity. What is the most likely diagnosis? A. Primary polydipsia B. Central diabetes insipidus C. Nephrogenic diabetes insipidus D. Addison disease E. Conn syndrome

397 C. SIMILAR TO PREVIOUS BOARD EXAM CONCEPT/PRINCIPLE.

Six days after a major bout of bloody diarrhea, a 5- year old boy developed crampy abdominal pain accompanied by irritability, pallor and multiple petecchial rashes throughout his trunk and extremities. There was a decrease in his urine output with note of hematuria. What is your most likely diagnosis? A. Lupus nephritis B. Hemolytic uremic syndrome C. Henoch-Schonlein purpura D. Thrombotic thrombocytopenic purpura E. Dengue hemorrhagic fever

399 B. SIMILAR TO PREVIOUS BOARD EXAM CONCEPT/PRINCIPLE.

A 4 yr old boy presents with his third episode of painful cervical lymphadenitis. Each was treated with incision and drainage and grew S. aureus. He also experiences recurrent skin infections. A year ago, he was hospitalized for osteomyelitis. The most important laboratory test is: A. PCR for ADA deficiency B. Nitroblue tetrazolium test C. MAC-I assay D. Neutrophil count E. Genetic chromosomal analysis

4 B. This is a case of chronic granulomatous disease (CGD) wherein patients are susceptible to catalase positive organisms like S. aureus. The nitroblue tetrazolium tests the neutrophils ability to generate superoxide anion and thus kill ingested bacteria. Source: Nelson's Textbook of Pediatrics 19th edition p746

Among the major criteria for Rheumatic fever, is the most specific? A. Arthritis B. Carditis C. Erythema marginatum D. Subcutaneous nodules E. Sydenham's chorea

412 E. Arthritis-most common, carditis-most severe, Subcutaneous nodules-least common, Sydenham's chorea-most specific

A newborn with tachypnea underwent a Babygram revealing Coarse streaking granular pattern of both lung fields, what would be your primary consideration? A. Respiratory Distress Syndrome B. Transient Tachypnea of the Newborn C. Bronchopulmonary Dysplasia D. Meconium Aspiration Syndrome E. Neonatal Pneumonia

416 D.

Signs of Cephalhematoma except: A. Subperiosteal Hemorrhage B. Does not cross the suture lines C. No discoloration of the scalp D. Does not cause jaundice E. No Exception

417 D. Cephalhematoma may cause jaundice.

Which of the following features is least consistent with juvenile rheumatoid arthritis? A. Onset before 16 years old B. Duration of symptoms for more than 6 weeks C. Most commonly oligoarthritic, affecting the knees and ankles D. Polyarthritic form is associated with uveitis in 20% E. Systemic Type is assosciated with characteristic quotidian Fever

400 D. Uveitis is associated with the oligoarthritic type of JRA.

Klienfelter's Syndrome: A. 47 XXY B. 45 XO C. 47 XXY/48 XXXY D. 47 XYY/47 XXY E. 47 XYY

401 C.

Among which of the following statements regarding breastmilk is/are true? A. Milk formula has lower Vitamin D levels than breastmilk B. Milk formula has lower Vitamin K levels than breastmilk C. Milk formula has higher Iron levels than breastmilk D. All of the above E. None of the above

402 C. Milk formulas have higher levels of Vitamin D,K and Iron than breastmilk.

A 3-year old patient presented with a palpable rash and arthritis on the knees after an apparent URTI few weeks prior to consult. What would be the next step for follow-up knowing your consideration? A. Platelet count after 1 month B. Arthrocentesis after 6 months C. Chest Xray after 6 months D. Urinalysis after 3 months E. C3 determination after 2 months

403 D. Regardless of the severity of symptoms, patients require urinalysis every 3 months for 1 year.

A 3-day old infant born to an indigent family, developed jaundice. The mother was unable to provide breast milk due to mastitis and occasionally gives rice-water with sugar as an alternative. Which of the following will be your next course of action? A. Sepsis neonatorum B. Physiologic jaundice C. Breastfeeding jaundice D. Breast milk jaundice E. None of the above

404 C. SIMILAR TO PREVIOUS BOARD EXAM CONCEPT/PRINCIPLE

A 2-year old suddenly develops maculopapular rashes suddenly after the disappearance of a 2-day fever. What is the probable cause of this disease? A. Rubella Virus B. Rubeola Virus C. Parvovirus B19 D. HHV-6 E. EBV

405 D. Roseola, caused by Human herpesvirus type 6

A 9-year old patient was brought to you with maculopapular rashes for 3 days. Among which of the following manifestations would you NOT suspect a Rubella virus infection? A. Posterior auricular lymphadenopathy B. Photophobia C. Forscheimer spots D. Low grade fever E. Polyarthritis

407 B. A patient with german measles does not develop photophobia.

An 18-year old patient presented with generalized lymphadenopathy, and hepatosplenomegaly, on diagnostic examinations atypical lymphocytes were seen with positive Paul-Bunnell antibodies. Which of the following Neoplastic diseases will the patient not be at risk of having? A. Nasopharyngeal CA B. Burkitt's Lymphoma C. Non-Hodkin's Lymphoma D. Lymphoproliferative Disease E. Leiomyosarcoma

408 C. The case is Infectious Mononucleosis. Hodkin's instead of Non-Hodkins

A 15-year old was brought to the ED due to dyspnea, multiple blisters, severe denudation of skin (45% TBSA) with intensive mucosal involvement. It appearad after she was prescribed Co-trimoxazole for an apparent UTI. What would be your primary consideration? A. Stevens Johnson Syndrome B. Toxic Epidermal Necrolysis C. Erythema Multiforme D. A and B E. All of the above

409 B. TBSA involvement of >40%, TEN may already be considered instead of SJS

After giving birth the ductus arteriosus closes due to which of the following mechanisms? A. Increase in systemic vascular resistance due to removal of the low resistance placenta B. Lack of blood return from the placenta C. Increased pressure in the Left Atrium D. Increased arterial Oxygen saturation E. None of the above

410 D. postnasal increase in oxygen saturation of the systemic circulation is the strongest stimulus for constriction of the ductal smooth musclesurrounding the ductus arteriosus.

A 4-year old male was brought to the ER after passing tea-colored urine. Patient was also noted to have periorbaital edema and with BP of 130/90. Urinalysis revealed 2-3 RBC casts. Mother insists that the child has no history of recent pharyngeal or oral infections. To support your hint that this is a case of cutaneous PSGN you would order for: A. C3 levels B. Streptozyme test C. ASO titer D. Anti-DNase B titer

421 D. *Favorite question. Remember that PSGN can follow either a throat (type 12) or cutaneous (type 49) infection by the nephrritogenic strain of S. pyogenes. Such test is the best single antibody that will document skin infection-related PSGN.

A child was admitted to your hospital due to cough for 8 days with associated restlessness, headache, anorexia, sore throat, hoarseness and eye redness. On physical exam, he was noted to have a swollen neck with an adherent, dense, grey pseudomembrane covering the posterior aspect of pharynx. If you are considering to eradicate possible nasopharyngeal carriage of the caregivers you will give them which drug? A. Erythromycin B. Ciprofloxacin C. Cotrimoxazole D. Ampicillin-Sulbactam

422 A. To differentiate diphtheria from strep throat, the former relatively presents afebrile and with the well- distinguished pseudomembrane in the pharynx. Diphtheria is notorious for its grave complications of toxic cardiomyopathy and toxic neuropathy (previous question as well!). Penicillin is the treatment of choice.

How is rubella different from measles? A. Presence of enanthem B. Low grade fever for up to 3 days C. Photophobia D. Development of dystonia and altered sensorium E. All of the above

423 B. The most characteristic sign of rubella is the presence of retroauricular lymphadenopathy (*buzz word). It can be differentiated from measles by having low grade fever for 1-3 days, polyarthritis, and absence of photophobia.

A 2-year old male infant with down syndrome is brought to your clinic for routine check-up. On auscultation you appreciate a continuous machinery-like murmur suggesting the presence of a shunt occuring throught the cycle. You are immediately convinced that this is abnormal because you know that the anatomic closure of the fetal shunt involved is at: A. 10 hours after birth B. 2 weeks after birth C. 10 months after birth D. 2 years after birth

424 B. This is a case of patent ductus arteriosus. Anatomic closure is normally completed at 2-3 weeks of age whereas functional closure is already achieved as early as 10-15 hours. PDA and other congenital heart diseases are common among Down syndrome patients.

These are benign cysts appearing on the hard palate of the newborn and are composed of accumulated epithelial cells: A. Ranula B. Epulis C. Mucous retention cysts D. Epstein pearls

425 D. A ranula is a mucocoele found on the floor of the mouth, a connective tissue swelling after rupture of a salivary gland duct as a result of local trauma. Epulis, on the other hand, simply refers to any benign mass situated on the gingiva. Both differ from epstein pearls by not being present in normal neonates and also by anatomic location.

A 5-year old male patient was seen at the ER due to one week history of fever with associated eye redness, swollen erythematous hands and soles, and nonvesicular truncal rash. On examination, cervical lymphadenopathy and strawberry tongue were observed. The single most important diagnostic test you must order for this patient is: A. ESR B. Acute phase reactants C. CT/MRI D. 2D echocardiography

426 D. The patient is suffering from Kawasaki disease. The diagnostic criteria for such: fever lasting for at least 5 days; bilateral bulbar conjunctival injection, non- purulent; Mucosal changes in the oropharynx (including strawberry tongue); peripheral extremity changes (including edema and erythema); rash; and cervical lymphadenopathy (>1.5 cm, usuallu unilateral); and illness should not be explained by other identifiable disease. There is no specific diagnostic test, however, 2D echo is essential to monitor the possible development of a coronary disease, the most serious complication of Kawasaki.

An infant was noted to have vomiting after every feeding. No other symptoms reported. Physical exam shows a restless tachycardic infant with a palpable firm, movable olive-shaped mass in the abdomen. Your initial impression of pyloric stenosis will be confirmed by: A. An abdominal UTZ showing pyloric length >14mm and thickness of >4 mm B. A barium study showing double tract sign corresponding to a narrowed channel C. An Xray exhibiting a double bubble sign D. An endoscopic biopsy revealing hypertonic and hypertrophic pyloric muscle

427 A. GI obstructive conditions and other anomalies (volvulus, duodenal atresia, Meckel, gastroschisis vs omphalocele) are high yield topics.

A 12-month old infant underwent an endorectal pull-through procedure to correct his Hirschprung. If you are to examine the biopsied segment you would say that there is: A. Excessive hypertrophy of the muscular and submucosal layers of the upper and lower rectum B. Lymphoid proliferation with notable enlargement of the submucosal layer C. Absence of ganglion cells in the wall D. Foci of muscular hypertrophy and sporadic villous masses with necrotic core

428 C. The recctosigmoid is affected in majority of the cases. There is arrest in neuroblast migration that leads to absence of Meissner and Auerbach plexuses. Such condition results in decreased motility in the affected segment; lack of propagation of the peristaltic waves going to the aganglionic part; and abnormal relaxation. All these explain its other common name congenital aganglionic megacolon.

This immunoglobulin does not fix complement. It is synthesized in the GI walls to bind with possible harmful elements in the lumen: A. IgM B. IgG C. IgD D. IgA

429 D. IgA, particularly the secretory type ( a dimer Ig). It is not only found in GI but also in the respiratory tract and in excretory glands. IgM is best in fixing the complement because it is pentameric, and hence the largest of all Igs. IgG is able to cross the placenta and is important in natural passive immunity. IgD on the other hand is said to play role in isotypic switching of Igs. IgE plays role both in hypersensitivity and parasitism.

Gaucher disease is deficiency of which enzyme A. Sphingomyelinase B. Hexosaminidase A C. Alpha-galactosidase D. Beta-glucuronidase E. Glucocerebrosidase

43 E. Inherited storage disease with deficiency of the enzyme glucocerebroside beta-glucosidase

A 4 year old child who just recently had impetigo, presents with gross hematuria and periorbital edema. The Blood Pressure is 100/70mmHg, Respiratory rate of 25cpm, heart rate of 80bpm, and temperature of 37.3deg C. There are RBC cast in urinalysis, CBC shows mild normochromic anemia, and levels of C3 is low. In this case, usually how long will hematuria persists? A. 6-12 months B. 6-12 weeks C. 6-12 days D. 2-4 years E. 3-5 months

464 A.

A 15-year old male sought consult in a local clinic complaining of recurrent headache. He described it as a band-like tightness sometimes throbbing occuring in the frontal region that radiates to his left parietal area. There was no nausea, vomiting, dizziness, anorexia, photophobia, change in alertness or intellect. He also reported that the condition seems to worsen during classes. At this point, the best possible impression is: A. Migraine without aura B. Hemiplegic aura C. Basilar-type migraine D. Tension headache

431 D. Migraine with aura aka Alice in wonderland syndrome is characterized by photopsia, blurring of vision and fortification spectra. Hemiplegic aura on the hand, has unilateral sensory or motor signs that may persist for days. Basilar-type migraine is due to vasoconstriction of basilar and posterior cerebral arteries that results in vertigo, tinnitus and diploma, with altered consciousness and seizures.

Laboratory findings in congenital adrenal hyperplasia (21-hydroxylase deficiency) will show LOW serum Na, Cl, and cortisol; and HIGH serum K, renin, and: A. Deoxycorticosterone B. 11-deoxycortisol C. 17-hydroxyprogesterone D. Pregnenelone

432 C. 21-hydroxylase deficiency also accounts for 90% of patients affected with CAH. Please review the adrenal steroidogenesis pathway, high yield.

A 2-year old female patient was diagnosed with iron deficiency anemia. Iron supplement therapy was immediately instituted. At what time after beginning of treatment do you expect hemoglobin to increase? A. 1 day B. 3 days C. 6 weeks D. 3 months

434 C. Ferrous sulfate is given at a dose 6 mg/kg/day (elemental iron). Subjective improvement is expected after 12-24 hours; preliminary BM response after 48 hours; increase in reticulocyte count after 72 hours; hemoglobin increase after 4-6 weeks; and repletion of iron stores after 3 months.

What is the most common cause of thrombocytopenic purpura in children? A. Idiopathic thrombocytopenic purpura B. Thrombotic thrombocytopenic purpura C. Microangiopathic hemolytic anemia D. Acute lymphoblastic leukemia

435 A. Differentiate ITP from TTP. ITP is an autoimmune destruction of platelets, and is usually precipitated by a viral infection. Bruising and petechial rash are the prominent signs. TTP, on the other hand, is more toxic-looking - similar to HUS. It has five cardinal symptoms: fever, hemolytic anemia, thrombocytopenia, renal dysfunction, and nervous sytem changes (mnemonic is FAT RN).

Which of the following is not true about brain tumors in children? A. Highest incidence in ages 1-10 years B. Commonly located in the infratentorial area C. Considered to be the most common solid tumor in pediatrics D. All of the above E. None of the above

436 E.

A 6-year old boy was brought to pedia ER because of acute onset of high fever and for appearing to be dyspneic. Mother reports that patient also has difficulty swallowing, drooling and muffled voice. Xray shows a thumbprint sign. Later on, the patient was intubated due to worsening symptoms. Being able to identify the most likely organism involved, the patient should be started with: A. Penicillin B. Trimethoprim-Sulfamethoxazole C. Ceftriaxone D. Racemic epinephrine

437 C. This is a case of acute epiglottitis, caused by H. influenzae. IV antibiotics 3rd generation cephalosporin or ampicillin-sulbactam are the primary drugs of choice.

A 3-year old malnourished male patient is brought to your health center due to difficulty of breathing. Initial assessment reveals T 39.0, RR 51 with episodes of severe coughing, and stable hemodynamics. There is focal dullness and faint vesicular breath sounds on left lower posterior chest. Given these quick initial findings, your strongest consideration is: A. Lobar pneumonia B. Bronchopneumonia C. Bronchiolitis D. Primary pulmonary tuberculosis

438 A.Fever, cough, tachypnea - triad of pneumonia. Dullness, faint or decreased breath sounds suggest consolidation.

What is the main purpose of giving BCG vaccine? A. To promote latency of TB infection B. To prevent extra-pulmonary TB C. To prevent primary TB infection D. All of the above E. None of the above

439. B..

Which serologic marker is found only in SLE? A. ANA B. Anti-Smith C. Anti-ds DNA D. ESR

440 B. ANA serves as an excellent screening parameter but it is not required in the diagnosis. It is nonspecific because it can also be significant in other rheumatic conditions. Anti-ds DNA is more specific to SLE than ANA and is used to assess the degree of disease activity. ESR is also non-specific and is a marker of acute inflammation.

A woman was noted to have a large volume of amniotic fluid at the time of her delivery of her child. At about 6-8 hours of age, the neonate begins regurgitating small amount of mucus and bile- stained fluid. Physical examination was normal. X- ray was requested and done revealing a double- bubble sign. The most commonly affected part of the gastrointestinal tract associated with the diagnosis is: A. Esophagus B. Stomach C. Pylorus D. Duodenum E. Large colon

441 D. Duodenal atresia is due to failure to recanalize the lumen of the duodenum. Obstruction is usually distal to the ampulla of Vater. Hallmarks iclude bilious vomiting without abdominal distention, polyhydramnios in 50% due to failure of absorption of amniotic fluid in the distal intestine. Double-bubble sign is a radiologic finding due to distended and gas- filled stomach and proximal duodenum.

A 5 year-old boy was brought by his mother at the out-patient department. He presented with a mild respiratory prodrome. The initial stage was characterized with erythematous facial flushing likened to a slapped cheek. The disease is most likely caused by: A. Rubella B. Rubeola C. HHV-6 D. Parvovirus B19 E. Varicella

442 D. Erythema infectiosum is caused by Parvovirus B19 characterized by low grade fever, headache, upper respiratory tract symptoms and a characteristic erythematous facial flushing or the slapped-cheek appearance which spreads rapidly to the trunk and proximal extremities as a diffuse macular erythema.

A 9 year-old boy has crampy abdominal pain and purpuric rashes on the back of his legs and buttocks as well as the extensor surfaces of his forearm. The most probable diagnosis is: A. Postrep GN B. Henoch-Schonlein Purpura C. IgA Nephropathy D. Kawasaki Disease E. Hemolytic Uremic Syndrome

443 B. Henoch-schonlein purpura is a mucocutaneous syndrome characterized by a rash which is palpable petechia or purpura that evolve from red to brown that last from 3-10 days. It is associated with arthritis, intermittent abdominal pain with renal involvement.

the trisomy most commonly associated with a heart defect in more than 90% of the cases is due to what chromosomal abnormality? A. 13 B. 18 C. 21 D. XO

569 B. Most common chromosomal anonaly assoc with cardiac defect: trisomy 18 Heart shapes: box shape: ebstein boot: tof egg: tga snowman: tapvr scimitar: papvr water bottle: pericardial tamponade

An infant was noted of intractable neonatal hypoglycemia associated with multiple congenital defects including macroglossia, visceromegaly, mild microcephaly, omphalocephaly, facial nevus flammeus and characteristic earlobe creases. The above findings pertains to the syndrome: A. Turner syndrome B. Klinefelter syndrome C. Beckwith-Wiedemann syndrome D. Down syndrome E. Edward's syndrome

444 C. Beckwith-Wiedemann syndrome is an overgrowth disorder usually present at birth characterized by an increased risk of childhood cancer and certain congenital features. Five common features used to define BWS are: macroglossia, macrosomia, midline abdominal wall defects (omphalocele/exomphalos, umbilical hernia, diastasis recti), ear creases or ear pits, and neonatal hypoglycemia. Other findings include nevus flammeus, prominent occiput, midface hypoplasia, hemihypertrophy, genitourinary anomalies, cardiac anomalies, musculoskeletal abnormalities, and hearing loss.

A 15 month-old male suddenly draws up his leg and scream in pain. It was repeated periodically throughout the night interspersed with periods of quiet sleep. He was brought to the Emergency Room after 12 hours and he looked pale. He vomited and passed out blood-streaked stools. PE revealed a palpable mass around the umbilicus. The most likely diagnosis is: A. Acute appendicitis B. Volvulus C. Meckel's diverticulum D. Intussusception E. Mesenteric adenitis

445 D. Intussusception occurs when a portion of the alimentary tract is telescoped into an adjacent segment. Clinical manifestations include severe paroxysmal colicky pain that recurs at frequent intervals with straining efforts, legs and knees are flexed with loud crying, about 60% of infants and children pass currant jelly stool. PE findings include palpation of slightly tender sausage-shaped mass in the RUQ which may increase in size and firmness during paroxysm of pain.

An 8 year-old girl has been having on-and-off fever for the past 10 days. On examination, there were conjunctivitis, strawberry tongue and periungual desquamation. The appropriate drug to be prescribed to the patient is: A. Steroids B. Aspirin C. Antibiotics D. Vitamin A E. Zinc sulfate

446 B. On the acute phase of Kawasaki disease, IV immunoglobulin results in rapid defervescence and resolution of clinical signs if illness. Aspirin is given during the convalescent stage, for prevention of acute coronary thrombosis.

The World Health Organization recommends that breastfeeding can be the infant's sole source of food up to about: A. 3 months of age B. 6 months of age C. 9 months of age D. 12 months of age E. 2 years of age

448 B. The World Health Organization recommends that breastfeeding can be the infant's sole source of food up to about 6 months of age.

A 6 month-old boy was brought at the OPD for check-up. On physical examination, you noted that the prepuce can be retacted but cannot be returned back. He has: A. Hydrocele B. Hypospadia C. Chordee D. Paraphimosis E. Epispadia

450 D. Paraphimosis is a condition that affects uncircumcised males and occurs when the retracted foreskin can no longer be pulled forward over the tip of the penis.

A neonate developed jaundice in the first 24 hours of life. These are differential diagnoses except: A. Physiologic jaundice B. ABO incompatibility C. Sepsis D. Hemorrhage E. None of the above

452 A. Neonatal jaundice is physiologic if it presents after the 24-48th hour of life. Pathologic jaundice presents in the 1st 24 hours of life. Among the causes include: sepsis, TORCHS, Rh and ABO incompatibility, hemorrhage

Radiographic examination to determine skeletal maturity or bone age is best evaluated by x-rays of the: A. Wrist B. Elbow C. Hip D. Knee joint E. Ankle

456 A. A bone age study helps doctors estimate the maturity of a child's skeletal system. It's usually done by taking a single X-ray of the wrist. A child's bones, such as those in the fingers and wrist, contain "growing zones" at both ends called growth plates.

A raised patch of epithelial debris on the conjunctiva of a child due to vitamin A deficiency is: A. Corneal scar B. Bitot's spots C. Kayser-fleischer rings D. Conjunctival xerosis E. Pterygium

457 B. Bitot's spots are the buildup of keratin debris located superficially in the conjunctiva, which are oval, triangular or irregular in shape. These spots are a sign of vitamin A deficiency and are associated with conjunctival xerosis. Kayserâ€"Fleischer rings are dark rings that appear to encircle the iris of the eye due to copper deposition in part of the cornea as a result of particular liver diseases.

A typical skin finding of infective endocarditis characterized by tender, pea-sized intradermal nodules in the pads of the fingers and toes is known as: A. Janeway lesions B. Subcutaneous nodules C. Osler's nodules D. Roth's spots E. Splinter hemorrhages

459 C. Osler's nodules are tender, pea-sized intradermal nodules in the pads of the fingers and toes found in Infective endocarditis. Janeway lesions are painless erythematous of hemorrhagic lesions on the palms and soles. Splinter hemorrhages are linear lesions beneath the nails. Roth's spots are retinal hemorrhages found in subacute bacterial endocarditis.

Gross Motor development: Sits without support A. 3 months B. 6 months C. 4 months D. 8 months E. None of the above

46 B.

Five years after a cutaneous viral infection, a 12 year-old boy had lethargy, altered level of consciousness, headache, nausea, vomiting and fever. This most likely post-infection is caused by a virus: A. Rubella B. Rubeola C. Herpes simplex D. Epstein Barr E. Cytomegalovirus

460 B. Subacute sclerosing panencephalitis is a chronic complication of measles with a delayed onset due to persistent infection with an altered virus that is harbored intracellularly in the CNS. It is caused by Rubeola from the family of Paramyxoviruses.

A 5 year old child presents with high grade fever, rhinorrhea, and conjunctivitis. The mother also noted rashes at the height of fever, which appears first on the face then spreads to the trunk and extremities. What is the diagnosis? A. Rubeola B. Rubella C. Roseola D. Varicella E. Fifth disease

461 A. this is rubeola, which presents with fever, coryza and conjunctivitis in which rashes appear at the height of the fever. In roseola, rashes appear when fever abates. In Rubella, the most characteristic is the presence of retroauricular, posterior cervical, and postoccipital lymphadenopathy.

What is the normal blood volume of a child? A. 100 ml/kg B. 90 ml/kg C. 80 ml/kg D. 125 ml/kg E. 130ml/kg

462 C. The normal blood volume of a child is 80 ml/kg, while for infant is 100 ml/kg.

Honey, a 6-year old girl, presents with erythematous, weeping, papules and vesicles, some coalescing to form scaly plaques distributed over the flexural areas. The mother said that the symptoms began in infancy and that her 8-month- old sibling, Dindin, also has similar symptoms. Which is the most appropriate treatment? A. Permethrin B. Antifungal cream C. Topical antibiotics D. Topical moisturizers and steroids

481 D. This is a case of atopic dermatitis which presents as an eczema in a population with a strong personal or family history of atopy. The management is maximal hydration with emollients, moisturizers and topical steroids.

Nes is a mentally retarded 15-year-old boy who has a long face, micropenis, large testes and large ears. Which of the following conditions does he most likely have? A. Fragile X syndrome B. Trisomy 21 C. Trisomy 13 D. Trisomy 18

482 A. The physical features of fragile X syndrome, long face, large ears, prominent jaw, large testes, hypotonia, repetitive speech, hand flapping etcetera, becomes more prominent after puberty.

The mother of Madeen, a 6-day old infant, has developed multiple vesicular lesions throughout her body. Which of the following is the correct management? A. Isolate the infant from the mother. B. Administer acyclovir to the infant. C. Administer varicella-zoster immunoglobulin to the infant. D. Continue regular well-baby care for the infant.

483 D. SIMILAR TO PREVIOUS BOARD EXAM CONCEPT/PRINCIPLE This is a case of varicella in a new postpartum mother. VZIG is only administered to the infant immediately if the mother had the onset of varicella within 5 days prior to delivery and 2 days after delivery. This is already a 6-day old infant so VZIG and isolation are not necessary. Acyclovir is given only to treat infants with lesions.

Loi, a newborn, has microcephaly, intracranial calcifications, hepatosplenomegaly, and marked hyperbilirubinemia and thrombocytopenia. This baby probably has: A. Congenital rubella B. Congenital CMV C. Congenital syphilis D. Congenital toxoplasmosis

484 B. SIMILAR TO PREVIOUS BOARD EXAM CONCEPT/PRINCIPLE. Microcephaly is for CMV, while hydrocephalus is for toxoplasmosis. Both of them presents with chorioretinitis and intracranial calcifications (although some references would label the calcifications of CMV as periventricular).

During a routine school physical examination, Jk, a 17 year-old girl, was noted to have a late apical systolic murmur, which is preceded by a click. The rest of the cardiac examination is normal. What kind of heart defect does she most likely have? A. None B. ASD C. VSD D. MVP

485 D. MVP presents with an apical murmur noted late in systole, which can be preceded by a click. It occurs with the billowing of the mitral valve leaflets into the atria at the end of systole. ASD presents with fixed split S2. SIMILAR TO PREVIOUS BOARD EXAM CONCEPT/PRINCIPLE

Sweet delivered to a term infant which was noted to be in respiratory distress shortly after birth. The infant has diminshed breath sounds on the left. A CXR was requested revealing a multiloculated mass in the left hemithorax. The tip of the NGT placed reveals that the stomach is below the diaphragm. Which is the most likely diagnosis? A. Congenital diaphragmatic hernia B. Congenital lobar emphysema C. Congenital cystic adenomatoid malformation D. Congenital bronchogenic cysts

487 C. Congenital cystic adenomatoid malformation (CCAM) arises from an embryonic disruption before 9 wks that causes improper development of bronchioles. Large lesions may compress the affected lung and cause pulmonary hypoplasia.

Joy, brought her 3-year old son in the middle of the night, who has developed noisy breathing on inspiration, chest retractions, and a barking cough. He has a mild upper respiratory tract infection for 2 days. Which would be the most helpful intervention? A. inhaled beta-agonist B. ceftriaxone C. racemic epinephrine D. amoxicillin

488 C. Laryngotracheobronchitis or viral croup presents with inspiratory stridor (extrathoracic symptom) as in this case. Agents causing croup are usually parainfluenza types 1 and 3. Racemic epinephrine can be used to alleviate respiratory symptoms. Epiglottitis presents with a more toxic-looking child and more acute presentation. The causative agent is H. influenzae, managed usually with ceftriaxone.

One-month-old Argie is admitted to the hospital for jaundice. Her total bilirubin is demonstrated to be 12 mg/dL with direct bilirubin of 4.5 mg/dl. Which among the following is a likely diagnosis? A. Gilbert syndrome B. ABO incompatibility C. Rh incompatibility D. Choledochal cyst

489 D. SIMILAR TO PREVIOUS BOARD EXAM CONCEPT/PRINCIPLE This is a case of obstructive jaundice (direct bilirubin >20% of total bilirubin). The rest of the other causes of jaundice listed typically lead to indirect hyperbilirubinemia.

5-month-old Maricel presented at the emergency room with irritability, repeated bouts of crying, and intermittent colicky abdominal pain. On physical examination, there is a sausage-shaped mass in the right upper quadrant. There is empty rectal vault on DRE, with blood per examining finger. What is the most likely diagnosis? A. Volvulus B. Intusussception C. Imperforate anus D. Malrotation

490 B. SIMILAR TO PREVIOUS BOARD EXAM CONCEPT/PRINCIPLE. The classic description would usually include currant-jelly stool (not mentioned during our exam) and a palpable sausage mass on the RUQ or RLQ.

Young Brian had numerous infected wounds 4 weeks ago. He presented at your clinic with gross hematuria but he is generally asymptomatic. You also find him to have hypertension. If this is post- streptococcal glomerulonephritis, when do you expect the microscopic hematuria to resolve? A. after 1 week B. after 6 months C. after a year D. never

491 C. Gross hematuria usually resolves within 3-6 months but microscopic hematuria may persist as long as 12- 18 months. C3 normalizes two months after the onset of illness.

Jarvie, a 6 year-old child, is noted to have 3+ protein on urinalysis. A 24-hr urine collection revealed a protein excretion of 3.7 g/24 hr. Elevated levels of cholesterol and triglycerides were also found. On your PE he has had unexpected weight gain and has scrotal edema. Which of the following drugs might have caused such presentation? A. penicillamine B. streptomycin C. paracetamol D. tetracycline

492 A. Drug-induced nephrotic syndrome has been described with the use of trimethadione, captopril, probenecid, lithium, procainamide, phenytoin, penicillamine and some NSAIDs and heavy metals (e.g gold).

Examination of the CSF of Zyra, a 10-year-old child, shows a classic picture of TB meningitis. She came in stuporous. What stage of TB meningitis is she in? A. Stage II B. Stage III C. Stage IV D. Stage V

493 B. There are only 3 stages of TB meningitis. Stage 1: non- specific signs, conscious, no hydrocephalus. Stage 2: confusion and or focal neurologic signs. Stage 3: stupor, delirium, hemiplegia.

Rhea, a 2-year-old child, presents with infantile spasms, hypopigmented lesions throughout her back (ash-leaf spots) and a single raised lesion over the sacrum (shagreen patch). Which is the most likely diagnosis? A. Sturge-Weber syndrome B. Neurofibromatosis C. McCune Albright Syndrome D. Tuberous sclerosis

494 D. SIMILAR TO PREVIOUS BOARD EXAM CONCEPT/PRINCIPLE. This case has the classic findings for tuberous sclerosis. Sturge-Weber presents with a portwine unilateral facial lesion, and blindness. NF presents classically with café au lait spots, Lisch nodules, and neurofibromas. Mc-Cune Albright presents with unilateral cafe au lait spots, fibrous dysplasia of bones and precocious puberty.

Ela presents with a 6-hour history of fever up to 39 C. Two hours prior to arrival at the ER, mother states that she noted a few purple spots scattered about the body on the patient especially on the buttocks and legs. During your PE the purple areas has spread rapidly and coalesced, and the patient is now obtunded. Which is the most likely diagnosis? A. Measles B. Rocky Mountain spotted fever C. Henoch-Schonlein Purpura D. Meningococcemia

495 D. Rocky mountain spotted fever is often a more indolent infection with rash developing days after the onset of fever. Henoch-schonlein purpura is actually a vasculitis and its progression is not as rapid nor toxic as described in the case. Measles present with coryza, conjunctivitis, and cough. Its rash is maculopapular.

Thirteen-year-old Mayda awakens with mild sorethroat, low-grade fever and a diffuse maculopapular rash. She developed arthralgia and marked swelling of the posterior cervical lymphnodes. Which is the most likely diagnosis? A. Rubella B. Rubeola C. Roseola D. Erythema multiforme

496 A. This is case is a classic description of Rubella. Rubeola is aka measles. Roseola is aka 6th disease or exanthem subitum.

Ivy, an 8-year-old child, was brought to your clinic for deep, aching pain over the lower extremities, most commonly noted in the evening and gone by the morning. The patient is otherwise asymptomatic and your PE is essentially normal. What is the best management? A. Reassure child and parents B. Get an xray C. Refer for bone marrow biopsy D. Give amoxicillin

497 A. "Growing pains" are commonly seen in the school-age child. They do not cause joint swelling, redness, heat, or systemic signs or symptoms. SIMILAR TO PREVIOUS BOARD EXAM CONCEPT/PRINCIPLE

While bathing her 2 year old son, Marg feels a mass in the abdomen. A thourough medical evaluation reveals aniridia, hypospadias, horseshoe kidney and hemihypertrophy. Which is the most likely diagnosis? A. neuroblastoma B. wilms tumor C. hepatoblastoma D. rhabdomyosarcoma

498 B. Usual presentation of Wilms (in decreasing frequency): abdominal mass, abdominal pain, hypertension, hematuria. The features of aniridia and renal anomaly makes the rest of the choices less likely.

Jerry is a mentally retarded 4-year-old who was noted to be hypotonic at birth and is noted to have tremendous appetite, obesity, hypogonadism and small hands and feet. The genetic mechanism responsible for this syndrome is due to: A. mitochondrial inheritance B. genomic imprinting C. mosaicism D. nondisjunction

499 B. The lack of part of paternal chromosome 15 causes Prader-Willi syndrome (described in the case) and the lack of part of maternal chromosome 15 causes Angelman syndrome.

What is the most likely serum concentration of Na+ and K+ in a 4-month-old boy with 21-hydroxylase deficiency? A. Na+ 115, K+ 7.5 B. Na+ 155, K+ 5.5 C. Na+ 144, K+ 3.0 D. Na+ 110, K+ 3.0

500 A. In 21-hydroxylase deficiency, the synthesis of both mineralocorticoids and cortisol is impaired. Aldosterone deficiency results to hyponatremia and hyperkalemia. Choice B is a finding in central and nephrogenic diabetes insipidus. Choice C is for patients with hyperaldosteronism.

APGAR score are routinely assessed at 1 and 5 minutes and every 5 minutes thereafter as long as resuscitation is continuing. The 5-minute APGAR score gives an idea of? A. what was going on during labor and delivery B. response to therapy and resuscitation C. what was going on during the last 9 months of pregnancy D. All of the above E. None of the above

502 B. 1-minute score gives an idea of what was going on during labor and delivery

A newborn infant shows an extraordinary division of the body from the forehead to the pubis into red and pale halves which is transient and harmless. What is the most likely diagnosis? A. Mongolian spots B. Acrocyanosis C. Ichthyosis congenita D. Mottling E. Harlequin color change

503 E. Nelson 19th ed., 533

What is the pathognomonic sign on plain abdominal radiograph of Neonatal Necrotizing Enterocolitis? A. Gas accummulation in the submucosa of the bowel wall B. Gas accumulation in the mucosa of the bowel wall C. Coiled spring sign D. Omega and Bird's beak sign E. Gas accumulation in the mucosa of the small intestines

504 A. pneumatosis intestinalis. Nelson 19th ed., 602

Which of the following is true of breast-milk jaundice? A. It means that the baby is not nursing well. B. Manangement includes continuation of the breastfeeding C. Occurs due to glucoronidase present in some breastmilk D. May be due to decreased milk intake E. All of the above

505 C. Significant elevation in unconjugated bilirubin (breast milk jaundice) develops after the 7th day of life; due to glucoronidase in milk; lasts 3w-3months, management: stop breastfeeding for 2 days.p607

What is the most common cardiac anomaly associated with congenital rubella syndrome? A. VSD B. ASD C. Coarctation of the aorta D. PDA E. TOF

506 D. PDA-78%, Right pulmonary artery stenosis-70%, L pulmonary artery stenosis-56%, Valvular pulmonic stenosis-40% p.1077

A premature newborn presented with intrauterine growth restriction, hepatosplenomegaly, jaundice, blueberry muffin-like rash, thrombocytopenia and purpura,and microcephaly and intracranial calcifications. What is the condition of the patient? A. Disseminated neuroblastoma B. Toxoplasmosis C. Congenital CMV infection D. Congenital rubella syndrome E. None of the above

508 C. Nelson 17th ed., 1116 Toxoplasmosis triad- chorioretinitis, hydrocephalus & cerebral calcifications

An infant brought to you with red eye due to mild corneal erosions from exophthalmos was also noted to have omphalocele as well as macroglossia. A probable cause of this is entertained: A. beckwith weidenmann syndrome B. WAGR C. Denys Drash syndrome D. Pituitary hyperfunction

570 A. most common cause of overgrowth in children: macroglossia exophthalmos omphalocele is Beckwith Weidenmann Syndrome

Nocturnal enuresis and encopresis refers to the occurrence of involuntary voiding at night and passage of feces into inappropriate places respectively, after what ages? A. 4 years and 5 years B. 5 years and 4 years C. 5 years and 6 years D. 6 years and 5 years E. None of the above

510 B. Nocturnal enuresis- refers to the occurrence of involuntary voiding at night after age 5 yr, the age when volitional control of micturition is expected. p. 1851 Encopresis- voluntary and involuntary passage of feces into inappropriate places at least once a month for 3 consecutive months once a chronologic or developmental age of 4 yr has been reached.p73

The major features of atopic dermatitis include the following, except: A. Pruritus B. Facial and extensor eczema in infants and children C. Flexural eczema in adolescents D. Personal or family history of atopic disease E. All of the above

511 E. also: Chronic or relapsing dermatitis Associated Features: Xerosis, cutaneous infections, nonspecific dermatitis of the hands and feet, Ichthyosis, palmar hyperlinearity, keratosis pilaris, nipple eczema, white dermatographism and delayed blanch response, anterior subscapular catarcts, keratoconus, elevated IgE, positive results of immediate-type allergy skin tests, early age at onset, Dennie lines, facial erythema or pallor, course influenced by environmental and/or emotional factors Nelson's 19th ed, p803

The following is true of Alport syndrome, except: A. It is a genetically heterogenous disease caused by mutations in the genes coding for type III collagen, a major component of the basement membranes. B. All patients with Alport syndrome have asymptomatic microscopic hematuria, which may be intermittent in girls and younger boys. C. The bilateral sensorineural hearing loss that occurs in most of the patient with the disease is never congenital. D. The presence of anterior lenticonus or the extrusion of the central portion of the lens into the anterior chamber is pathognomonic. E. None of the above

513 A. mutation in type IV collagen -Nelson's 19th ed., 1782

The classical clinical criteria of Kawasaki Disease include the following except: A. Fever persisting at least 7 days B. Acute changes in extremities manifesting as erythema of palms, soles; edema of hands and feet C. Bilateral bulbar conjunctival injection without exudate E. lip cracking, strawberry tongue

516 A. Fever persisting at least 5 days, subacute changes in the extremities manifesting asperiungal peeling of fingers and toes, polymorphous exanthem, cervical lymphadenopathy (>1.5 cm diameter) Nelson's 19th ed., 863

A 6 year old girl was brought to the ER because of episodes of loose bowel of 4 episodes per day amounting to 2 cups per bout which started 2 days ago. Physical exam reveals child to be irritable, eager to drink, tachycardic, weak pulse, with parched lips and decreased urine output. What is your admitting impression? A. Shigellosis B. Acute Gastroenteritis with moderate dehydration C. Acute Gastroenteritis with severe dehydration D. Ascariasis E. None of the above

518 C. patient has >2 signs and symptoms belonging to the severe dehydration category

Hematologic work up of a 15 year old female revealed microcytic hypochromic red blood cells with low reticulocyte count. Which of the following is excluded form the differential diagnosis? A. Iron deficiency anemia B. Thalassemia trait C. Lead poisoning D. Acute bleeding E. Sideroblastic anemia

519 D. Acute bleeding-Normocytic normochromic RBCs with low retic count

What is the most consistent clinical manifestation of pneumonia? A. Fever B. Cough C. Crackles D. Increased respiratory rate E. Tachycardia

520 D. Tachypnea is the most consistent clinical manifestation of pneumonia. Nelson's 19th ed., 1476

A previously healthy 10 year old boy is brought to the physician because of a rash over his arms and legs for 7 days. His blood pressure is elevated. On physical examination, a yellow, crusted, excoriated rash over the upper and lower extremities are seen. Urinalysis reveals 100 RBC/hpf and 5-10 WBC/hpf. Which of the following is the most likely diagnosis? A) Acute glomerulonephritis B) Henoch-Schonlein purpura C) Lupus nephritis D) Nephrolithiasis

521 A.

A 13 year old boy is brought to the physician because of a 2 week history of left hip pain with an associated limp. He has had no redness or swelling of the left leg or hip. There is no history of trauma or musculoskeletal problems. Vital signs are stable. He holds his left lower extremity in slight external rotation and hip flexion at rest. Internal rotation and abduction of the left hip are decreased. He walks with a limp and is unable to bear his full weight on the left. Which of the following is the most likely underlying cause of this patient’s condition? A) Acute inflammatory reaction of the synovial lining of the hip joint B) Avascular necrosis of the proximal femur C) Bacterial infection of the hip joint D) Disruption of the femoral head epiphyseal plate

522 D.

A 10 month old infant is brought to the emergency department because of labored breathing for 1 hour. She has had cough, coryza, and fever for 18 hours. Her temperature is 39 C, pulse is 120 bpm, respirations are 54 cpm. Pulse oximetry shows an oxygen saturation of 92%. Bilateral wheezes and basilar crackles are heard. What is the most likely diagnosis? A) Community acquired viral disease B) Contiguously spread bacterial infection C) Toxin mediated capillary leak D) Osmotically generated fluid shift

523 A.

A 13 year old boy is brought to the physician because of a 3 month history of left knee pain that is exacerbated by vigorous exercise. He also has had occasional pain in his right knee. There is no history of trauma. He is at the 50th percentile for height and weight. His temperature is 37 C. Examination shows mild swelling and tenderness of the left tibial tubercle; range of motion of the knees is full. Which of the following is the most appropriate next step in management? A) MRI of the affected knee B) Antibiotic therapy C) NSAIDs D) Knee immobilization

525 C.

A 17 year old girl comes to the physician because of fatigue, increased thirst, and increased urination over the past 2 weeks; she has had a 4.5 kg weight loss during this period despite an increased appetite. At the onset of her symptoms, she had a mild upper respiratory tract infection that resolved without treatment. Vital signs are within normal. Cranial nerves are intact. Muscle strength is 5/5, ad deep tendon reflexes are symmetric. Serum chemistries reveal Na 132 Cl 96 K 3.7 HCO3 26 and Crea 1.2. Which of the following is the most likely location of the primary disease process? A) Adrenal gland B) Central nervous system C) Pancreas D) Kidney

527 C.

digeorge syndrome: thymic hypoplasia in connection with CATCH22 has the following clinical features except: A. Cardiac anomaly B. Thymic hyperplasia C. Cleft palate D. Hypocalcemia

571 B. chromosome 22, cardiac, anomaly of face, thymic hypoplasia, cleft palate, hypocalcemia constitute CATCH22

A 2 month old boy is brought to the physician for a well-child examination. He was born at term following an uncomplicated pregnancy, labor, and delivery. He has been breast-feeding six to eight times daily since birth. He smile and lifts his head. He is at the 50th percentile for length, 75th percentile for weight, and 25th percentile for head circumference. Examination shows no abnormalities. His mother asks for nutritional recommendations. Which of the following is the most appropriate recommendation for the infant? A) Begin vitamin K supplementation B) Add rice cereal C) Add cow milk-based formula D) Begin vitamin D supplementation

529 D.

A 7 year old boy is brought to the emergency department because of facial edema and respiratory distress since eating dinner 2 hours ago. He has had coryza and cough during the past 2 days. His temperature is 37.5 C, pulse is 100 bpm, respirations are 40 cpm, and blood pressure is 100/70 mmHg. Breath sounds are unequal with decreased aeration and a prolonged expiratory phase. Which of the following is the most likely diagnosis? A) Allergen induced bronchospasm B) Community acquired viral disease C) Toxin-mediated capillary leak D) Chemical irritant pneumonitis

530 A.

A 5-month-old boy is brought for a follow-up examination. He was born at 37 weeks' gestation and has had persistent wheezing since shortly after birth despite treatment with nebulized and oral bronchodilators and oral corticosteroids. His diet consists of 32 ounces of iron-fortified cow's milk- based formula daily. He appears well nourished and happy. On examination, there is moderate relief of wheezing with extension of the neck. Which of the following is the most likely mechanism of this infant's wheezing? A ) Allergic reaction to cow's milk B ) Aspiration of a foreign body C ) Compression of the airway by a vascular ring D ) Concurrent upper respiratory tract infection

532 C.

A 16-year-old girl is brought to the physician because of episodes of palpitations over the past 6 months. The episodes occur when she runs or plays basketball. She is otherwise asymptomatic. Her blood pressure is 124/46 mm Hg, pulse is 78/min, and respirations are 18/min. She weighs 55 kg (121 lb) and is 180 cm (71 in) tall. Her arm span is 188 cm (74 in), and the upper segment to lower segment ratio is 0.85. Her fingers appear long and are hyperextensible. A grade 4/6, early diastolic murmur is heard along the upper and middle left sternal border with radiation to the apex. Peripheral pulses are bounding. Which of the following is the most likely cause of these findings? A) Aortic incompetence B) Aortic stenosis C) Mitral incompetence D) Mitral stenosis

533 A.

A 5-week-old boy is brought to the physician because of vomiting for 3 days. Switching from a cow's milk-based formula to a soy-based formula and one bottle of an electrolyte solution has not decreased his vomiting. His mother says that there is no yellow color to the vomitus, but it is forceful and occurs immediately after he has had 1 to 2 ounces of liquid. He appears to vomit more liquid than he drank. He has one mustard-colored seedy stool daily. Examination shows no abnormalities. Which of the following is the most likely explanation for his vomiting? A) Duodenal atresia B) Gastroesophageal reflux C) Hypertrophic pyloric stenosis D) Lactose intolerance

534 C.

A 3-year-old boy who is HIV positive is brought for a routine examination. His diet is appropriate for age. His medications include three antiretroviral drugs and trimethoprim-sulfamethoxazole for Pneumocystis carinii prophylaxis. Laboratory studies show: Hemoglobin 8.6 g/dL Mean corpuscular hemoglobin 38 pg/cell Mean corpuscular hemoglobin concentration 30% Hb/cell Mean corpuscular volume 101 μm3 Leukocyte count 5600/mm3 Segmented neutrophils 60% (many hypersegmented) Bands 3% Lymphocytes 37% Red cell distribution width 21% (N=10â€"16) Which of the following is most likely to have prevented this patient's anemia? A) Folic acid supplementation B) Iron supplementation C) Vitamin B12 (cyanocobalamin) supplementation D) Monthly intravenous immune globulin therapy

535 A.

A 10-year-old girl is brought to the emergency department because of diffuse, aching abdominal pain, nausea, and recurrent vomiting over the past 5 hours. She has an 8-year history of type 1 diabetes mellitus treated with 20 U of NPH and 6 U of regular insulin in the morning and 14 U of NPH and 5 U of regular insulin in the evening. She appears lethargic but is easily arousable. There is an obvious odor of ketones on her breath. Her blood pressure is 100/70 mm Hg, pulse is 95/min, and respirations are 20/min and deep. Serum studies show: Na+ 142 mEq/L K+ 5.3 mEq/L HCO3â€" 6 mEq/L Glucose 710 mg/dL Which of the following laboratory findings is most likely to be increased? A ) Arterial pH B ) Serum C-peptide level C ) Serum magnesium level D ) Serum osmolality

536 D.

A 5-year-old girl with ventricular septal defect is scheduled for tonsillectomy in 2 weeks. She has had an allergic reaction to penicillin in the past. Her temperature is 37 C (98.6 F). Examination shows no abnormalities. Which of the following is the most appropriate prophylaxis prior to tonsillectomy? A) Amoxicillin B) Ciprofloxacin C) Erythromycin D) No prophylaxis indicated

537 D.

A previously healthy 16-year-old boy is brought to the emergency department 20 minutes after an episode of left arm shaking that lasted approximately 3 minutes. Over the past 2 days, he has had fever and emotional lability. On arrival, his temperature is 38.9 C (102 F). He is somnolent and disoriented to person, place, and time. He responds poorly to pain. Neurologic examination shows no other abnormalities.Analysis of cerebrospinal fluid shows: Leukocyte count 120/mm3 Segmented neutrophils 20% Lymphocytes 80% Erythrocyte count 300/mm3 Glucose 60 mg/dL Protein 400 mg/dL Which of the following is the most likely cause of this patient's neurologic findings? A) Bacterial infection B) Viral infection C) Fungal infection D) Parasitic infection

538 B.

A newborn is in severe respiratory distress immediately following delivery. She was born at 35 weeks' gestation to a 35-year-old woman, gravida 2, para 1, aborta 1, who did not receive prenatal care. The newborn's pulse is 60/min, and respirations are irregular and labored. Examination shows pallor with perioral cyanosis, anasarca, hepatosplenomegaly, and scattered petechiae. Cord blood hemoglobin is 4 g/dL, and reticulocyte count is 18%. A direct antiglobulin (Coombs') test is positive. Which of the following sets of blood groups is most likely in the mother and her newborn? (Mother, Newborn) A) A, Rh-positive O, Rh-positive B) A, Rh-positive O, Rh-negative C) O, Rh-negative O, Rh-positive D) O, Rh-positive O, Rh-negative

539 C.

A 3 week old infant came in for non-bilous vomiting occurring immediately after feeding. This was followed by progressive loss of fluids and electrolytes and dehydration. Feeding was alright. Most likely you are dealing with A. Duodenal atresia B. Esophageal atresia C. Hypertrophic pyloric stenosis D. Duodenal stenosis

573 C.

A 16-year-old boy with neurofibromatosis is brought for a follow-up examination. His uncle also has neurofibromatosis. He has a 1-year history of headaches during which his parents say he appears pale. Six months ago, he underwent operative treatment for an optic nerve glioma. His blood pressure is 164/105 mm Hg, pulse is 102/min, and respirations are 14/min. The thyroid glands are not enlarged. No murmurs are heard, and radial pulses are equal. Abdominal examination shows no abnormalities. Which of the following is the most likely cause of this patient's high blood pressure? A) Catecholamine-producing tumor B) Carcinoma of the thyroid gland C) Essential hypertension D) Overproduction of aldosterone from an adrenal adenoma

540 A.

A 12/F is brought to the clinic due to fever and painful joints. Five days PTC, the parents noted that patient was febrile and has a sore and swollen left knee. A few days later, her left knee improved; but her left elbow started to become similarly sore and swollen. You note that the patient had a sore throat four weeks prior. What is the MOST likely diagnosis? A. Juvenile rheumatoid arthritis B. Septic arthritis C. Reactive arthropathy D. Viral arthropathy E. Rheumatic fever

541 E. SIMILAR TO PREVIOUS BOARD EXAM CONCEPT/PRINCIPLE. Note that the patient already satisfies 2 major criteria: fever + migratory polyarthritis. This plus a history of sore throat (likely streptococcal) raises suspicion for rheumatic fever.

At birth, an infant is noted to have a cardiac murmur. A loud, harsh blowing holosystolic murmur is heard best over the lower left sternal border. There is no clinical evidence of congestive heart failure. What is the most likely diagnosis? A. Atrial septal defect B. Ventricular septal defect C. Patent ductus arteriosus D. Coarctation of the aorta E. Tetralogy of Fallot

543 B. VSDs are the MOST COMMON of the CHDs. Small VSDs with trivial L->R shunts would present with a loud, harsh, blowing holosystolic murmur heard best over the L sternal border. ASD would present with a systolic ejection murmur produced by increased flow across the right ventricular outflow tract into the pulmonary artery. PDA presents with a continuous machinery like murmur.

You are part of a team sent to assess the nutritional status of school children in a far-flung locale. Which of the following anthropometric measurements would best screen for chronic malnutrition in the 2- 10 year old age group? A. Height for age B. Weight for height C. Weight for age D. Mid upper arm circumference E. BMI

544 A. Height for age - a measure of skeletal growth - reflects cumulative impact of events affecting nutritional status. Stunting is a reflection of chronic malnutrition. Weight for height on the other hand screens for wasting which is a measure of acute malnutrition. Weight for age has less clinical significance because it combines stature with current health conditions. MUAC is used in emergencies and in the field in lieu of weight for height.

A 3/F was rushed to the emergency room due to difficulty breathing. At the emergency room, patient was noted to have the following vital signs: RR 48, HR 104, and T 39.9C. On PE, there was note of chest indrawing. What is the most likely causative agent in this infection? A. Haemophilus influenzae B. Streptococcus pneumoniae C. Mycoplasma pneumoniae D. Chlamydia pneumoniae E. Chlamydia trachomatis

545 B. Streptococcus pneumoniae is the most common pathogen in children 3 weeks to 4 years of age.

A 45/F who has not had any prenatal check-ups delivers a 3,800 g live baby girl. The infant has decreased tone, upslanting palpebral fissures, epicanthal fold, redundant nuchal skin, fifth finger clinodactyly and brachydactyly and a single transverse palmar crease. A 2D Echo was performed by the attending physician to screen the child of congenital heart diseases; the most common of which in this population is: A. Atrial septal defect B. Ventricular septal defect C. Tetralogy of Fallot D. Coarctation of the aorta E. Endocardial cushion defect

547 E. Approximately 50% of Down's Syndrome infants have cardiac defects - most commonly an endocardial cushion defect (60%), VSD (32%), and TOF (6%).

A history of bloody stools, draining ears and atopic eczema in a 5-month old should prompt one to suspect: A. Ataxia-telangiectasia syndrome B. DiGeorge Syndrome C. Wiskott-Aldrich syndrome D. Severe Combined Immunodeficiency E. Combined Variable Immunodeficiency

548 C. Wiskott-Aldrich syndrome, an X-linked recessive syndrome, is characterized by atopic dermatitis, thrombocytopenic purpura and an undue susceptibility to infection.

A day old newborn who was under observation in the NICU for temperture instability had a suspected seizure. On examination, you note that the patient had wide set eyes, a small mandible and a cleft palate. You hear a murmur on auscultation. Chest radiograph reveals no apparent thymus. On seeing the results of serum chemistries, you determine that the seizure was probably due to severe hypocalcemia. You now suspect a syndrome caused by mutant genes in this chromosome: A. Chromosome 22 B. Chromosome 13 C. Chromosome 21 D. Chromosome 19 E. X chromosome

549 A. Patient has DiGeorge Syndrome. Remember the mnemonic: CATCH-22. It is characterized by: Cardiac abnormalities, Abnormal facies, Thymic aplasia, Cleft palate, Hypocalcemia and deletion in 22q11.

A newborn infant presents with generalized jaundice within the first 24 hours of life. Which of the following considerations is LEAST likely? A. Erythroblastosis fetalis B. Neonatal sepsis C. Congenital infections D. Physiologic jaundice E. Hemolysis

551 D. Jaundice at birth or within the first 24 hours of life is a cause for concern. Causes include: erythroblastosis fetalis, concealed hemorrhage, sepsis, congenital infections and hemolysis.

A 10/M came in to the community clinic due to ear pain accentuated by jaw motion. Prior to the pain, patient reported itching in his ear. PE is remarkable for severe tragal tenderness and ear pain with manipulaton of the pinna. You had to defer complete otoscopic examination due to severe canal tenderness. The most likely common etiologic agent in this condition is: A. Staphylococcus aureus B. Streptococcus pneumoniae C. Klebsiella pneumonier D. Enterobacter aerogenes E. Pseudomonas aeruginosa

552 E. External otitis is caused most commonly by P. aeruginosa. Otitis media is most commonly caused by S. pneumoniae.

A major criteria in the diagnosis of atopic dermatitis is A. chronic relapsing course B. elevated serum IgE C. early age of onset D. white dermatographism

574 A.

The use of short acting beta 2 agonists as sole treatment is limited to: A. Mild Interval Asthma B. Mild Persistent Asthma C. Mild Recurring Asthma D. Mild Intermittent Asthma

575 D. Mild Intermittent Asthma is treated with SABA PRN, whereas mild persistent asthma warrants a low-dose ICS.

For the patient above, you then advise: A. Hospitalization for administration of IV vancomycin B. Outpatient therapy with Amoxicillin 500mg BID x 7 days C. Ciprofloxacin eardrops applied to wick inserted into ear 3x/day for 2 days D. Hospitalization for administration of IV levofloxacin E. Daily cleansing and close follow up

553 C. Topical otic preparations containing neomycin (active against gram-positive organisms and some gram- negative organisms, notably Proteus spp.) with either colistin or polymyxin (active against gram-negative bacilli, notably Pseudomonas spp.) and corticosteroids are highly effective in treating most forms of acute external otitis. Newer preparations of eardrops (e.g., ofloxacin, ciprofloxacin) are preferable and do not contain potentially ototoxic antibiotics.

A patient who suffered kernicterus as a neonate would be at risk of developing which form of cerebral palsy? A. Spastic diplegic B. Spastic quadriplegic C. Hemiplegic D. Extrapyramidal

554 D. A similar question was asked in the Aug 2013 boards. Nelsons 19e p.2062

A 17/M comes in after having an episode of generalized tonic clonic seizure upon waking up. On probing, you learn that the patient has been having myoclonic jerks in the morning, often causing the patient to drop things. EEG in this patient demonstrates 4-5 cycle per second generalized spike and waves. Which anticonvulsant is MOST appropriate for him? A. Ethosuximide B. Carbamazepine C. Valproic acid D. Topiramate E. Vigabatrin

555 C. Patient has juvenile myoclonic epilepsy or Janz syndrome, the MOST COMMON generalized epilepsy in young adults. Patients usually present with myoclonic jerks in the morning, often causing patients to drop things. It is best treated with VALPROATE or LAMOTRIGINE

A term infant, delivered via cesarean section, is referred to you for tachypnea and expiratory grunting at the first hour of life. Otherwise, physical examination is unremarkable. You order a stat CXR which revealed prominent vascular markings and fluid in intralobar fissures. Which of the following is the MOST IMPORTANT predisposing factor in this condition? A. Birth by cesarean delivery B. Low birthweight C. Prolonged labor D. Administration of large amounts of IV fluids to the mother E. Birth to a diabetic mother

556 A. Patient has transient tachypnea of the newborn. Premature birth, precipitous birth, and operative birth without labor have all been associated with an increased risk of TTN.

A 2 month old boy was brought to the emergency room because after the mother discovered the boy no longer breathing in his bed. You start CPR at the ER; but the patient was pronounced dead after 30 minutes of resuscitation. After thorough review of the birth and medical history, you fail to identify a clear cause of death. Which of the following could have prevented these kinds of deaths? A. Use of a cardiac monitor B. Positioning the infant to sleep on his/her back C. Home monitoring with cameras and baby monitors D. Breastfeeding and administration of vitamins E. All of the above

557 B. The epidemiologic evi- dence that positioning the babies to sleep on their backs reduces the rate of SIDS deaths by more than 50% suggests that position, and not prematurity, has been the primary cause of SIDS. Avoid- ance of cigarette smoking exposure and of overheating the infant are also important in the prevention of SIDS.

A 17/F consulted due to recurrent episodes of having "iced tea" colored urine. He has been having these episodes for several years; notably occuring a few days after onset of upper respiratory tract infections. Patient has no other symptoms or remarkable physical findings. What is the most likely diagnosis? A. Post-streptococcal glomerulonephritis B. Nephrolithiasis C. Hemolytic-uremic syndrome D. Henoch Schonlein Purpura nephritis E. IgA nephropathy

558 E. IgA nephropathy or Berger nephropathy is the MOST COMMON chronic glomerular disease; commonly presenting as gross hematuria often occuring 1-2 days after onset of URTI or GI infection

A 24/F medical student comes in due to a 10-day history of malaise, low-grade fever, headache and nausea. On physical examination, enlarged posterior cervical lymph nodes are palpated, hypertrophied tonsils with exudates were seen, and the Traube's space was obliterated. CBC done revealed leukocytosis with lymphocytic predominance. Which of the following organisms is the most likely etiologic agent? A. Group A b-hemolytic Streptococcus B. Coxsackie virus C. Adenovirus D. Epstein-Barr virus E. Haemophilus influenzae

559 D. Patient has infectious mononucleosis.

The severe acute respiratory syndrome outbreak in the early 2000s was caused by what virus A. Rhinovirus B. Influenza virus C. Coronavirus D. Parainfluenza virus E. Respiratory syncitial virus

56 C. The SARS coronavirus, sometimes shortened to SARS-CoV, is the virus that causes severe acute respiratory syndrome (SARS).

A 2 year old afebrile male child was brought to you for consult due to 2 days of persistent diarrhea x 8 a day with vomiting episodes. Upon evaluation patient has no sunken eyeballs, awake and frequently cries with minimal handling. He drinks more than enough water but ends up vomiting. What is the estimated fluid deficit of this patient? A. 25 mL per kg B. 45 mL per kg C. 80 mL per kg D. 110 mL per kg

563 C. 80mL/kg; This patient presents with AGE with some dehydration because of increased thirst and irritability. (Consider the major clinical parameters: Sensorium, Thirst, and Skin pinch) Some Dehydration equates to 50-100 mL per kg of deficit (hence, we resuscitate using 75ml per kg to meet half way between the two ranges).

A 3 year old boy who just had his birthday presents to you with profuse watery diarrhea that was later diagnosed to be due to cholera. What is the antibiotic of choice? A. No antibiotic is needed B. Tetracycline even if the patient is only < 8years old C. Ceftriaxone IV drip D. Metronidazole PO dose

564 B. Although Tetracycline is generally not favorable for use among children, it may be so in cases of cholera by virtue of its short duration of therapy (3 days).

A 6 month old baby girl is expected to be able to do the following, except: A. Stand while holding the crib B. Sit with some support C. Grab large objects with both hands D. Social smile

565 A. the patient is not expected to be able to stand by 6 months of age.

Most common cause of arf in children: A. PSGN B. IgA Nephropathy C. Hemolytic Uremic Syndrome D. FSGS

566 C. HUS is the most common cause of ARF in children

A known case of congenital heart disease patient presents to you with an egg shaped chest Xray most likely has: A. TGA B. ASD C. Truncus Arteriosus D. Pericardial effusion

567 A.

A 3-week old infant initially normal at birth was noted to have episodes of vomiting accompanied by purposeless movements, rhythmic rocking, and athetosis. On physical examination, vital signs were normal and patient was hypertonic with hyperactive deep tendon reflexes. Newborn testing revealed elevated plasma phenylalanine levels >20 mg/dL. What is the initial management recommended for the patient? A. Restrict phenylalanine in the diet B. Oral administration of the cofactor tetrahydrobiopterin C. Sequential phenylalanine determination until levels decrease D. Repeat newborn screening E. Limit protein intake to amino acids

581 A. Deficiency of the enzyme phenylalanine hydroxylase or of its cofactor tetrahydrobiopterin causes accumulation of phenylalanine in body fluids and the central nervous system (CNS). The severity of hyperphenylalaninemia depends on the degree of enzyme deficiency and may vary from very high plasma concentrations (>20 mg/dL or >1,200 μmole/L, classic phenylketonuria [PKU]) to mildly elevated levels (2â€"6 mg/dL or 120â€"360 μmole/L).The affected infant is normal at birth. Mental retardation may develop gradually and may not be evident for the 1st few months. Vomiting, sometimes severe enough to be misdiagnosed as pyloric stenosis, may be an early symptom. Older untreated children become hyperactive, with purposeless movements, rhythmic rocking, and athetosis. The goal of therapy is to reduce phenylalanine in the body; formulas low in or free of this amino acid are available commercially. The diet should be started as soon as diagnosis is established. It is generally accepted that infants with persistent plasma levels of phenylalanine >6 mg/dL (360 μmole/L) should be treated with a phenylalanine- restricted diet similar to that for classic PKU.

An infant is born full term to a woman who had an uncomplicated pregnancy. Immediately after delivery, the infant was noted to be cyanotic, tachypneic, and with intercostals retractions. There was absent breath sounds on both lungfields. Bowel sounds were heard on the left hemithorax. The abdomen is scaphoid. The CXR shows a multicystic mass in the left chest with a shift of the mediastinum to the right. What is the most likely diagnosis? A. Congenital cystic adenomatoid malformation B. Diaphragmatic hernia C. Respiratory distress syndrome D. Transient tachypnea of the newborn E. Pulonary Hypoplasia

582 B. A diaphragmatic hernia is defined as a communication between the abdominal and thoracic cavities with or without abdominal contents in the thorax. Early respiratory distress within 6 hr of life is thought to be a poor prognostic sign. The clinical signs of respiratory distress are characterized by tachypnea, grunting, use of accessory muscles, and cyanosis. Children with CDH will also have a scaphoid abdomen and increased chest wall diameter. Bowel sounds may also be heard in the chest with decreased breath sounds bilaterally. The point of maximal cardiac impulse may be displaced away from the side of the hernia if mediastinal shift has occurred. A chest x-ray and nasal gastric tube is all that is usually required to confirm the diagnosis

Which of the following is the recommended vaccination schedule for neonates born mothers whose HBsAg status was positive? A. Hepatitis B immunoglobulin within the 12 hours after birth, Hepatitis B vaccine at 1 month, 3 months and 6 months old B. Hepatitis B Immunoglobulin at birth and Hepatitis B vaccine at birth, 1 month and 6 months old C. Hepatitis B immunoglobulin within the 24 hours after birth, Hepatitis B vaccine at birth, one and six months old D. Hepatitis B vaccine at birth, one and six months old E. Hepatitis B immunoglobulin within the 24 hours after birth and after 3 days, Hepatitis B vaccine at birth, one and six months old

583 B. To prevent perinatal transmission through improved maternal screening and immunoprophylaxis of infants born to HBsAg -positive mothers. Infants born to HBsAg -positive women should receive vaccine at birth, 1â€"2 mo, and 6 mo of age The 1st dose should be accompanied by administration of 0.5 mL of HBIG as soon after delivery as possible (within 12 hr) because the effectiveness decreases rapidly with increased time after birth. Postvaccination testing for HBsAg and anti-HBs should be done at 9â€"18 mo.

While at school a 7 year old boy is noted to have failing grades at school. His teacher noted him to be staring blankly for 15-20 seconds multiple times a day during class. There was no noted Consult done and EEG shows 3 per second generalized spike and wave discharges. What is the MOST likely diagnosis of this boy? A. Generalized tonic-clonicseizure B. Attention deficit disorder C. Complex partial seizures D. Absence seizures E. Mental retardation

584 D. Absence seizures are characterized by a sudden cessation of motor activity or speech with a blank facial expression and flickering of the eyelids. These seizures, which are uncommon before age 5 yr, are more prevalent in girls, are never associated with an aura, rarely persist longer than 30 sec, and are not associated with a postictal state. Children with Absence seizures may experience countless seizures daily, whereas complex partial seizures are usually less frequent. The EEG shows a typical 3/sec spike and generalized wave discharge.

A 2 year old male was noted to have a right inguinal bulge. On physical examination, the mass is smooth, firm, and more prominent on crying. Both testes were palpated in the scrotum. Transillumination was negative. This condition is due to? A. Weakness in the transverses abdominis muscle B. Failure of the testes to descend C. Patency of the procesus vaginalis D. Accumulation of serous fluid in the scrotum E. Enlarged and elongated pampiniform plexus

585 C. Most inguinal hernias in infants and children are congenital and result from a persistent patency of the processus vaginalis. The pertinent developmental anatomy of congenital indirect inguinal hernia relates to development of the gonads and descent of the testis through the internal ring and into the scrotum late in gestation.

A 15-month old boy was brought for consult due to cyanosis. Patient was noted to have bluish discoloration of the lips after vigorous crying. On physical examination, a systolic thrill is felt along the left sternal border in the 3rd and 4th parasternal spaces, there is a systolic murmur most intense at the left sternal border, and the 2nd heart sound is single. On chest x-ray, the heart is boot- shaped and hilar areas and lung fields are relatively clear. The management of this case will depend on? A. The patency of the ductus arteriosus B. The size of the ventricular septal defect C. The severity of the right ventricular outflow tract obstruction D. The presence of other congenital anomalies E. The degree of ventricular hypertrophy

586 C. Treatment of the tetralogy of Fallot depends on the severity of the right ventricular outflow tract obstruction. Infants with severe tetralogy require medical treatment and surgical intervention in the neonatal period. Therapy is aimed at providing an immediate increase in pulmonary blood flow to prevent the sequelae of severe hypoxia. Neonates with marked right ventricular outflow tract obstruction may deteriorate rapidly because, as the ductus arteriosus begins to close, pulmonary blood flow is further compromised. The intravenous administration of prostaglandin E1. Infants with less severe right ventricular outflow tract obstruction who are stable and awaiting surgical intervention require careful observation.

A 14-year old female was brought to the clinic due to pallor. This was associated with easy fatigability and frequent consumption of ice cubes. She had regular menses for 1 year. On physical examination, BP 100/60, PR 120, RR 22, temp 36.8, weight=40 kg, height=160 cm. There was pale palpebral conjunctivae and palmar pallor. CBC showed Hgb 90, Hct .28, WBC 5, platelet 250, 000. Serum ferritin and serum iron were low and total iron binding capacity was low. If you plan to start the patient on oral iron therapy, what is the recommended dose? A. 500 mg of elemental iron once a day B. 2-3 mg/kg of elemental iron in 3 divided doses daily C. 250 mg of elemental iron twice daily D. 4â€"6 mg/kg of elemental iron in 3 divided doses daily E. 1 g of elemental iron once a week

587 D. In iron deficiency anemia, the therapeutic dose should be calculated in terms of elemental iron; ferrous sulfate is 20% elemental iron by weight. A daily total dose of 4â€"6 mg/kg of elemental iron in 3 divided doses provides an optimal amount of iron for the stimulated bone marrow to use.

A 3 year old male was brought to your clinic due to right elbow swelling. Few hours prior to consult, patient fell and hit his right elbow. There was noted rapid swelling of the affected joint and violaceous discoloration of the overlying skin. On further probing, the mother claimed that the patient was noted to have easy bruisability. On physical examination, vital signs were normal. There was noted tenderness and swelling on the right elbow with limitation in the range of motion. If you are suspecting Hemophilia, what initial screening test will you request for the patient? A. Prothrombin time B. Activated partial thromboplastin time C. Bleeding time D. Thrombin time E. Clotting time

588 B. The laboratory screening test that is affected by a reduced level of factor VIII or factor IX is PTT. Results of the other screening tests of the hemostatic mechanism (platelet count, bleeding time, prothrombin time, and thrombin time) are normal. The specific assay for factors VIII and IX will confirm the diagnosis of hemophilia.

A 5 year old female was brought for consult due to a 2 month history of anorexia, weight loss, low grade fever and easy bruisabilty. On physical examination, patient had pale palpebral conjunctivae, generalized lymphadenopathy and splenomegaly. CBC showed anemia, thrombocytopenia, and leukocytosis with atypical lymphocytes. Bone marrow aspiration showed >25% of the bone marrow cells as a homogeneous population of lymphoblasts. What is the most likely diagnosis? A. Acute myelogenous leukemia B. Acute lymphoblastic leukemia C. Multiple myeloma D. Myelofibrosis E. Plasmacytoma

589 B. The initial presentation of ALL usually is nonspecific and relatively brief. Anorexia, fatigue, and irritability often are present, as is an intermittent, low-grade fever. On physical examination, findings of pallor, listlessness, purpuric and petechial skin lesions, or mucous membrane hemorrhage may reflect bone marrow failure. The proliferative nature of the disease may be manifested as lymphadenopathy, splenomegaly, or, less commonly, hepatomegaly. The diagnosis of ALL is strongly suggested by peripheral blood findings indicative of bone marrow failure. Anemia and thrombocytopenia are seen in most patients. ALL is diagnosed by a bone marrow evaluation that demonstrates >25% of the bone marrow cells as a homogeneous population of lymphoblasts.

A 12 year-old female was brought to your clinic room due to blurring of vision. Patient had a one month history of blurring of vision of both visual fields described as being cloudy accompanied by headaches. Patient was given paracetamol which afforded relief of the headache. On fundoscopy, there was noted papilledema. MRI was requested which showed a solid tumor with cystic structures containing fluid of intermediate density in the suprasellar area. What is the most likely diagnosis? A. Intracranial aneurysm B. Pineoblastoma C. Mennigioma D. Chiasmal glioma E. Craniopharygioma

590 E. Craniopharyngioma (WHO grade I) is a common tumor of childhood, accounting for 7â€"10% of all childhood tumors. These tumors are solid with cystic components and occur within the suprasellar region. They are minimally invasive, adhere to adjacent brain parenchyma, and engulf normal brain structures. MRI demonstrates the solid tumor with cystic structures containing fluid of intermediate density. CT may show calcifications associated with the solid and cystic wall components. There is significant morbidity (panhypopituitarism, growth failure, visual loss) associated with these tumors and their therapy owing to the anatomic location.

A 10 year old male was brought to the emergency room due to red-brown urine. Patient had a history of sore throat 2 weeks prior. On physical examination, patient was hypertensive, febrile, and with bipedal edema. Urinalysis showed ph 6.0, specific gravity 1.30, albumin 3+, sugar (-), RBC 40/hpf, WBC 0-2/hpf , RBC casts +. ASO titers were elevated. What are the expected biopsy findings that with differentiate it from other causes of acute glomerulonephritis? A. Crescents on light microscopy and absence of deposits on immunofluorescence and light microscopy B. Linear IgG, C3 on immunofluorescence, Positive anti-GBM antibody C. Diffuse mesangial IgA on immunofluorescence, mesangial deposits in electron microscopy, elevated seum IgA D. Low serum C3, Granular IgG, C3 on immunofluorescence, Subepithelial humps on electron microscopy E. Normal microscopy and Immunofluorescence with effacement of foot processes on electron microscopy

591 D. The clinical diagnosis of poststreptococcal glomerulonephritis is quite likely in a child presenting with acute nephritic syndrome, evidence of recent streptococcal infection, and a low C3 level. On renal biopsy, Granular IgG, C3 on immunofluorescence and subepithelial humps on electron microscopy will be seen.

On physical examination, the neonate was noted to have an absent Moro and Biceps reflex on the left arm. The left arm was also noted to be abducted, internally rotated, and the forearm was pronated. What birth injury does the patient most likely have? A. Erb-Duchenne paralysis B. Klumpke paralysis C. Horner syndrome D. Duchenne muscular dystrophy E. Myasthenia gravis

598 A. Injury to the brachial plexus may cause paralysis of the upper part of the arm with or without paralysis of the forearm or hand or, more commonly, paralysis of the entire arm. In Erb-Duchenne paralysis, the injury is limited to the 5th and 6th cervical nerves. The characteristic position consists of adduction and internal rotation of the arm with pronation of the forearm. Power to extend the forearm is retained, but the biceps reflex is absent; the Moro reflex is absent on the affected side.

Which of the following may cause a reactivation of pulmonary tuberculosis? A. Rubella B. Varicella C. Mumps D. Measles E. Herpes

599 D. Measles infection is known to suppress skin test responsiveness to purified tuberculin antigen. There may be an increased rate of activation of pulmonary tuberculoses in populations of individuals infected with Mycobacterium tuberculosis.

A 1 year old female was brought to the emergency room due to 1 week history of fever. On physical examination, PR 145, RR 32, temp 38.2. There was generalized maculopapular rash, erythema of the oral and pharyngeal mucosa, the tongue was swollen, lips were dry and cracked, and cervical lymphadenopathy. This disease entity is of clinical significance due to its predilection to? A. Renal arteries B. Aorta C. Coronary arteries D. Cerebral arteries E. Pulmonary arteries

592 C. Kawasaki disease (KD), formerly known as mucocutaneous lymph node syndrome and infantile polyarteritis nodosa, is an acute febrile vasculitis of childhood. Fever is characteristically high (104°F or higher), remittent, and unresponsive to antibiotics. The duration of fever without treatment is generally 1â€"2 wk, but it may persist for 3â€"4 wk. Prolonged fever is prognostic for the development of coronary artery disease. In addition to fever, the five characteristic features of Kawasaki disease are: bilateral bulbar conjunctival injection, usually without exudate; erythema of the oral and pharyngeal mucosa with strawberry tongue and dry, cracked lips, and without ulceration; edema and erythema of the hands and feet; rash of various forms (maculopapular, erythema multiforme, or scarlatiniform) with accentuation in the groin area; and nonsuppurative cervical lymphadenopathy, usually unilateral, with node size of ≥1.5 cm. Perineal desquamation is common in the acute phase. Periungual desquamation of the fingers and toes begins 1â€"3 wk after the onset of illness and may progress to involve the entire hand and foot. Cardiac involvement is the most important manifestation of Kawasaki disease. Myocarditis, manifested as tachycardia out of proportion to fever occurs in at least 50% of patients.

A 2 year old was admitted at the pediatric ICU due to burn injuries to the trunk and both lower extremities. On the 5th day at the ICU, patient was noted to have pink macules which later progressed to hemorrhagic nodules and eventually to ulcers with ecchymotic and gangrenous centers with eschar formation, surrounded by an intense red areola. If you are entertaining an infection caused by pseudomonas aeruginosa, what antibiotic has coverage for this organism? A. Ciprofloxacin B. Ceftriaxone C. Trimethoprim-Sulfamethoxazole D. Cloxacillin E. Doxycycline

593 A. The surfaces of burns or wounds are frequently populated by Pseudomonas and other gram-negative organisms. The characteristic skin lesions of Pseudomonas, ecthyma gangrenosum, whether caused by direct inoculation or metastatic secondary to septicemia, begin as pink macules and progress to hemorrhagic nodules and eventually to ulcers with ecchymotic and gangrenous centers with eschar formation, surrounded by an intense red areola. Antibiotics with Pseudomonas coverage are Ceftazidime, Ciprofloxacin, Tobramycin, Gentamicin, Aztreonam, and Amikacin.

A 17 year old female was rushed to the emergency room due to multiple eythematous papules on the trunk and upper extremities. Patient had was recently given Ciprofloxacin. On examination, there papules with an erythematous outer border, an inner pale ring, and a dusky purple to necrotic center noted on the trunk and upper extremities as well as on the oral cavity. What is the most likely diagnosis? A. Bullous pemphigoid B. Bullous drug eruption C. Allergic vasculitis D. Stevens-Johnson syndrome E. Pemphigus vulgaris

594 D. Cutaneous lesions in Stevens-Johnson syndrome generally consist initially of erythematous macules that rapidly and variably develop central necrosis to form vesicles, bullae, and areas of denudation on the face, trunk, and extremities. The skin lesions are typically more widespread than in EM and are accompanied by involvement of two or more mucosal surfaces, namely the eyes, oral cavity, upper airway or esophagus, gastrointestinal tract, or anogenital mucosa. A burning sensation, edema, and erythema of the lips and buccal mucosa are often the presenting signs, followed by development of bullae, ulceration, and hemorrhagic crusting.

A 24 year old primigravid gave birth to a live term baby boy. On physical examination, the child was noted to have imperforate anus. Patient was observed for 24 hours and there was no noted passage of meconium stained urine or the presence of meconium at the median raphe. At the 24th hour of life a, cross-table lateral x-ray was done and showed that the marker on the skin was 3.5 cm from the rectum. What is the definitive management for this type of impeforate anus? A. Perform colostomy immediately B. Simple dilatation with Hegar dilators C. Posterior sagittal anorectoplasty D. Swenson's pull-through procedure E. Kasai procedure

595 C. Children with a high lesion have a double-barrel colostomy performed. This effectively separates the fecal stream from the urinary tract. It also allows the performance of an augmented pressure colostogram before repair to identify the exact position of the distal rectum and the fistula. The definitive repair or posterior sagittal anorectoplasty (PSARP) is performed at about 1 yr of age.

An 11- month old infant was brought to you clinic due to fever irritability. On physical examination, PR 100, RR 25, temp 37.9. The nasal turbintaes were erythematous and congested without purulent discharge. On otoscopy, the tympanic membrane was bulging, amber-clored, and with air bubbles behind the tympanic membrane. If you are considering acute otitis media, what is the dose of amoxicillin that you will give to your patient? A. 10-15 mg/kg/day B. 30-40 mg/kg.day C. 40-60 mg/kg/day D. 60-80 mg/kg/day E. 80-100 mg/kg/day

596 E. Acute Otitis Media First-line: High-dose amoxicillin (80â€"100 mg/kg/day). Alt for penicillin allergy: Cefuroxime, cefdinir, cefprozil, azithromycin Persistent otitis media (after 3 days): Amoxicillin/clavulanic acid, cefuroxime, or ceftriaxone (IM/IV)

A 7 month old girl was brought to the emergency room due to high fever. Patient had 4 days history of fever (38C) followed by coryza, cough and maculopapular rash which started on the face and later spred to the trunk and extremities. What vitamin deficiency is associated with increased mortality from this disease entity? A. Vitamin K B. Vitamin A C. Vitamin D D. Vitamin B1 E. Vitamin B6

597 B. Vitamin A deficiency in children in developing countries has long been known to be associated with increased mortality from a variety of infectious diseases, including measles. Several randomized controlled trials of vitamin A therapy in the developing world and the United States have demonstrated reduced morbidity and mortality from measles. The American Academy of Pediatrics suggests vitamin A therapy for selected patients with measles.

A 4 year old boy was brought to ER due to uncontrolled bleeding. Upon history, you learned that few hours PTC, patient had 2 of his incisors extracted and bleeding was not controlled. Hematologic exams were normal aside from prolong bleeding time. What is your primary consideration? A. Von Willebrand Disease B. Hemophilia A C. Hemophilia B D. DIC E. None of the above

641 A. VWD is the most common inherited bleeding disorder. Usual clinical picture is a child who undergone a certain procedure and had uncontrollable bleeding. Another clue is the prolonged bleeding time with normal platelet count.

A 12 year old seeks consult for 1 day history of sneezing, clear rhinorrhea and nasal itching not associated with fever or any other systemic symptoms. PE reveals boggy, pale nasal edema with a clear discharge. The most likely diagnosis is? A. Foreign body B. Vasomotor rhinitis C. Allergic rhinitis D. Neutrophilic rhinitis E. Rhinitis medicamentosa

6 C. Allergic rhinitis is often seasonal and associated with allergic conjunctivitis. Eosinophils predominate in the nasal secretions. It is classified as: Seasonal (cyclical) or perennial (all year) Intermittent (symptoms occur <4 days per week or <4 consecutive weeks) or persistent (symptoms occur >4 days per week or >4 consecutive weeks) Mild-moderate (no impairment) or severe (with impairment of daily living/sleep) Source: Nelson's Textbook of Pediatrics 19th edition p775

59.) A client is diagnosed with an acute myocardial infarction and is receiving tissue plasminogen activator, alteplase (Activase, tPA). Which action is a priority nursing intervention? A. Monitor for renal failure. B. Monitor psychosocial status. C. Monitor for signs of bleeding. D. Have heparin sodium available.

60 B.

A 2 year old female was brought for consult due to a 2 day history of fever. Patient was also noted to have decreased appetite and was irritable. There was no accompanying cough, colds, or rashes. Mother gave paracetamol to the patient which provided temporary lysis of fever. On physical examination, PR 110, RR 24, temp 39.5C. There were noted multiple, small vesicles and ulcers that are surrounded by erythematous rings on the soft palate, uvula, and posterior pharyngeal wall. There were no noted skin lesions, and cardiac, pulmonary, and abdominal examinations were unremarkable. What is the most common etiologic agent that causes this type of infection? A. Coxsackie A virus B. Coxsackie B virus C. Herpes simplex virus D. Epstein-Barr virus E. Cytomegalovirus

600 A. Herpangina is characterized by sudden onset of fever, sore throat, dysphagia, and lesions in the posterior pharynx. Temperatures can range from normal to 41°C (106°F); fever tends to be greater in younger patients. Characteristic lesions, present on the anterior tonsillar pillars, soft palate, uvula, tonsils, posterior pharyngeal wall, and, occasionally, the posterior buccal surfaces, are discrete 1â€"2 mm vesicles and ulcers that enlarge over 2â€"3 days to 3â€"4 mm and are surrounded by erythematous rings that vary in size up to 10 mm. Typically about 5 lesions are present, with a range of 1 to >15. The remainder of the pharynx appears normal or minimally erythematous. Fever generally lasts 1â€"4 days, and resolution of symptoms occurs in 3â€"7 days. A variety of enteroviruses can cause Herpangina, although coxsackie A viruses are implicated most often.

What congenital anomaly is associated with growth and mental retardation, abnormal facial features like low set ears, hypotonia and an endocardial cushion defect? A. Velocardiofacial syndrome B. Down syndrome C. Turner syndrome D. VATER syndrome

601 B. Trisomy 21 (down syndrome) is characetrized by the ff: hypotonia, flat face, upward and slanted palpebral fissures and epicanthic folds, speckled irises (Brushfield sports);varying degrees of mental and growth retardation;dysplasia of the pelvis, cardiac malformations (endocardial cushion defect) and simian crease;short, broad hands, hypoplasia of middle phalanx of 5th finger, duodenal atresia, and high arched palate. (Nelson's 11th ed)

A 14-year old male is expected to have the following psychologic and mental characteristics, EXCEPT: A. More stable body image B. Intense peer group involvement C. Initiation of relationships and sexual activity D. Emergence of abstract thought (formal operations) E. No exception

603 A. 14-16 year old adolescents (middle adolescence) has the following characteristics: emergence of abstract thought (formal operations), may perceive future implications, but may not apply in decision-making, questioning more, Concern with attractiveness, Increasing introspection, Conflicts over control and independence, Struggle for acceptance of greater autonomy, Intense peer group involvement, Preoccupation with peer culture, Peers provide behavioral example, Testing ability to attract partner, Initiation of relationships and sexual activity, Questions of sexual orientation, Gauging skills and opportunities. (Nelson's 11th ed)

The following genetic conditions predispose a child to develop leukemia, EXCEPT: A. Down syndrome B. Fanconi syndrome C. Severe combined immune deficiency D. Klinefelter syndrome E. None of the above.

604 E. The following are genetic conditions that may predispose a child to develop leukemia: Down syndrome, Fanconi syndrome, Bloom syndrome, Diamond-Blackfan anemia, Schwachman syndrome, Klinefelter syndrome, Turner syndrome, Neurofibromatosis type 1, Ataxia-telangiectasia, Severe combined immune deficiency, Paroxysmal Nocturnal Hemoglobinuria, Li Fraumeni Syndrome (Nelsons 11th Edition)

The following conditions will give rise to a state of hyponatremia, EXCEPT: A. SIADH B. Acute Renal Failure C. Congenital Adrenal Hyperplasia D. Cystic Fibrosis of the Pancreas E. None of the above

605 C. Congenital adrenal hyperplasia has several forms. The most common form (21 b-hydroxylase deficiency) leads to increased mineralocorticoid synthesis, leading to sodium retention and hypertension.

A 7 year old male was noted to have fever of five days duration, with temperature ranging from 39 to 40 C . It is accompanied by frontal headache, joint pains and generalized malaise. Two days after fever has resolved, maculopapular erythematous rashes appear at the lower and upper extremities in glove and stocking pattern. The following describes the possible laboratory findings, EXCEPT: A. Neutropenia B. Low platelet count C. Hemoconcentration D. Prolonged PT and PTT E. Increased transaminase values

606 D. In dengue fever, pancytopenia may occur after the 3â€"4 days of illness. Neutropenia may persist or reappear during the latter stage of the disease and may continue into convalescence with white blood cell counts of <2,000/mm3. Platelets rarely fall below 100,000/mm3. Venous clotting, bleeding and prothrombin times, and plasma fibrinogen values are within normal ranges. The tourniquet test result may be positive. Mild acidosis, hemoconcentration, increased transaminase values, and hypoproteinemia may occur during some primary dengue virus infections. The electrocardiogram may show sinus bradycardia, ectopic ventricular foci, flattened T waves, and prolongation of the P-R interval.

Which of the following would indicate Tanner stage 3 sexual development in a female child? A. Acne B. Breast and papilla elevated as a small mound C. Darkly pigmented, slightly curly pubic hair D. Fine hair on upper lip E. Menstruation

643 C.

You went on a night duty as a ward resident-on- duty and you saw an interesting case of an infant with low-set ears whose laboratory examinations showed hypocalcemia, absence of a thymic shadow on chest roentgenogram, and persistent candidiasis. This is most probably a case of? A. an autoimmune disease B. Di George sundrome C. Idiopathic hypoparathyroidism D. Renal failure E. Severe Combined Immunodeficiency

645 B.

A 3-year old male was brought to the OPD due to oliguria, pallor, lethargy, edema and generalized weakness 2 weeks after an episode of bloody diarrhea. Laboratory exams revealed hemoglobin 3 is 6 mg/dl, platelet is 35,000/mm , creatinine is 2mg/dl, hematuria and proteinuria. Blood peripheral smear revealed segmented RBCs, Coomb's test is negative. The most likely diagnosis is: A. Hemolytic-Uremic Syndrome B. Idiopathic Thrombocytopenic Purpura C. Thrombotic Thrombocytopenic Purpura D. Neprotic-Nephritic Syndrome E. Acute Post-Streptococcal Glomerulonephritis

607 A. Hemolytic uremic syndrome (HUS) is characterized by the acute onset of microangiopathic hemolytic anemia, renal injury, and a low platelet count. The following are both present at some time during the illness: 1. anemia (acute onset) with microangiopathic changes (i.e., schistocytes, burr cells, or helmet cells) on peripheral blood smear; and 2. renal injury (acute onset) evidenced by either hematuria, proteinuria, or elevated creatinine level (i.e., ≥1.0 mg/dL in a child younger than 13 yr or ≥1.5 mg/dL in a person 13 yr or older or ≥50% increase over baseline).

A 15-year old female complained of pharyngitis, fever, muscle pain, body weakness, vomiting, watery diarrhea and headache. She was given supportive medications by her mother. After 3 days, she became dyspneic, lethargic and irritable. In the ER, she was found to be hypotensive and there was note of several purpuric rashes all over body.She then had a generalized seizure, lost consiousness and proceeded to coma. The most likely diagnosis is: A. Henoch-Schönlein purpura B. Toxic shock syndrome C. Meningococcemia D. Idiopathic thrombocytopenic purpura E. Subsclerosing panencephalitis

608 C. Acute meningococcemia initially may mimic viral illness with pharyngitis, fever, myalgias, weakness, vomiting, diarrhea, and/or headache. A maculopapular rash is evident in about 7% of cases, typically before more serious signs develop. Limb pain, myalgias, or refusal to walk occur in many cases and is the primary complaint in 7% of otherwise clinically unsuspected cases. Cold hands or feet and abnormal skin color are also early signs. In fulminant meningococcemia cases, the disease progresses rapidly over hours to septic shock characterized by prominent petechiae and purpura (purpura fulminans), hypotension, DIC, acidosis, adrenal hemorrhage, renal failure, myocardial failure, and coma

An 8-year old male had remittent fever, with Tmax at 40 C, accompanied by maculopapular rashes with accentuation on the groin. He was brought to a pediatrician and was diagnosed with measles. He was advised Vit.A supplementation and supportive treatment. Fever, however, persisted for 6 more days. On physical exam, the patient developed bilateral bulbar conjunctival injection without exudate, erythema of the oral and pharyngeal mucosa with strawberry tongue and dry, cracked lips, edema and erythema of the hands and feet, still with maculopapular rash and unilateral cervical lymphadenopathy. There was periungual desquamation of the fingers and toes. What is the most appropriate treatment for this patient? A. IV fluids with electrolyte replacement B. Aminoglycosides C. 3rd cephalosporin D. Intravenous immunoglobulin E. Supportive treatment

609 D. The patient has kawasaki disease. In addition to fever, the five characteristic features of Kawasaki disease are: bilateral bulbar conjunctival injection, usually without exudate; erythema of the oral and pharyngeal mucosa with strawberry tongue and dry, cracked lips, and without ulceration; edema and erythema of the hands and feet; rash of various forms (maculopapular, erythema multiforme, or scarlatiniform) with accentuation in the groin area; and nonsuppurative cervical lymphadenopathy, usually unilateral, with node size of ≥1.5 cm. Perineal desquamation is common in the acute phase. Periungual desquamation of the fingers and toes begins 1â€"3 wk after the onset of illness and may progress to involve the entire hand and foot. Treatment involves high dose aspirin and IVIG.

An 11 month old baby presents for evaluation of fever and rash. His mother noted a fever of 40 C two days ago. He appeared well and was eating and playing normally, so his mother was not alarmed. After the fever resolved, he developed a red rash on his trunk that progressed rapidly over the past 24 hours. What is the most likely diagnosis? A. Exanthem subitum B. Rubella C. Rubeola D. Erythema infectiosum E. Menigococcemia

61 A. Roseola infantum/exanthem subitum is characterized by the abrupt onset of high fever that usualy resovles after 72 hours coincident with the appearance of a faint pink-rose coloured morbiliform rash on the trunk.

A 1-year old old male comes in for a well-baby visit. He had BCG, 3 doses of DPT & OPV and 2 doses of hepatitis B vaccine. What can you give him for this particular visit? A. Measles and 3rd dose of hepatitis vaccines B. Measles vaccine C. 3rd dose of hepatitis B D. Varicella vaccine

610 A. The missed dose of hepatitis B may be given along with measles vaccine.

A preterm infant, 2-hr old, was observed to have shallow breathing at 65 breaths/minute, with grunting and cyanosis. CXR shows fine reticular granularity of the parenchyma and air bronchograms. Which of the following should have been done to prevent this condition? A. Administration of betamethasone to the mother 48 hr before delivery B. Administration of 1st dose of surfactant into the trachea of premature infant immediately after birth C. Determination of lecithin: sphingomyelin ratio in the amniotic fluid D. All of the above E. A and C only

612 D. Administration of betamethasone to women 48 hr before the delivery of fetuses between 24 and 34 wk of gestation significantly reduces the incidence, mortality, and morbidity of RDS. Administration of a 1st dose of surfactant into the trachea of symptomatic premature infants immediately after birth is also an effcetive preventive measure. In timing cesarean section or induction of labor, estimation of fetal head circumference by ultrasonography and determination of the lecithin concentration in amniotic fluid by the lecithin: sphingomyelin ratio decrease the likelihood of delivering a premature infant.

The WHO recommendation states that breastmilk should be the infant’s sole source of nutrition up to about: A. 2 months of age B. 6 months of age C. 12 months of age D. 2 years of age E. 3 years of age

613 B. After 6 months of age, complementary feeding should be started in order to achieve complete nutrition.

A newborn was delivered from a 32 year old G3P3 (3003). Within sixty seconds after delivery, he was crying loudly, coughing upon sunctioning, with pink body and blue extremities. There was some flexion of extremities and the heart rate was 120 beats/min. What is the APGAR score? A. 6 B. 7 C. 8 D. 9 E. Cannot be determined

614 C. APGAR scoring is as follows (0, 1, 2) Heart rate: Absent, Below 100, Over 100 Respiratory effort: Absent; Slow, irregular; Good, crying Muscle tone: Limp; Some flexion of extremities; Active motion Response to catheter in nostril (tested after oropharynx is clear): No response; Grimace; Cough or sneeze Color: Blue, pale; Body pink, extremities blue; Completely pink

A 4-year old boy was brought to your clinic because he was exposed to his grandmother who was coughing out blood for two months already. He has good weight gain and appetite, and was generally asymptomatic. A Mantoux tuberculin test. Which of the following should be the minimum size of induration for this child to be positive for TB infection? A. 5 mm B. 10 mm C. 15 mm D. 20 mm E. 25 mm

615 A. Definition of a positive tuberculin skin test for children varies according to risk. INDURATION ≥5 MM: 1.Children in close contact with known or suspected contagious people with tuberculosis disease. 2.Children suspected to have tuberculosis disease: Findings on chest radiograph consistent with active or previously tuberculosis disease and clinical evidence of tuberculosis disease. 3.Children receiving immunosuppressive therapy or with immunosuppressive conditions, including HIV infection. INDURATION ≥10 MM: Children at increased risk of disseminated tuberculosis disease: 1.Children younger than 4 yr of age. 2.Children with other medical conditions, including Hodgkin disease, lymphoma, diabetes mellitus, chronic renal failure, or malnutrition. 3. Children born in high-prevalence regions of the world. 4.Children frequently exposed to adults who are HIV infected, homeless, users of illicit drugs, residents of nursing homes, incarcerated or institutionalized, or migrant farm workers. 5.Children who travel to high-prevalence regions of the world. INDURATION ≥15 MM: Children 4 yr of age or older without any risk factors.

A 12-year old male with diagnosed with streptococcal pharyngitis and amoxicillin was started. One hour after drug intake, he complained of tingling sensation around his mouth, difficulty of swallowing and developed hoarse voice. At the ER, there was generalized urticaria, puffy eyelids, PR=130/min, RR=32/min, BP=70/30 and T= 37.2ËšC.The patient is experiencing this type of shock: A. Cardiogenic B. Hypovolemic C. Distributive D. Septic E. Obstructive

616 C. Anaphylactic shock is an example of distributive shock, which is caused by abnormalities of vasomotor tone. There will be loss of venous capcitance decreases preload, loss of arterial capcitance, leading to decreased afterload or systemic blood pressure.

A 9-month old male came to your clinic for scheduled follow up. He was born from a HBsAg positive mother, and already received HBIg and complete hepatitis B vaccination (3 doses). You obtained blood for immunologic studies. Result is negative for both HBsAg and anti-HBs. What should be your next step? A. Counsel parents and refer to pediatric gastroenterologist B. Give 4th dose of hepatitis B vaccine C. Start lamivudine therapy D. Start a 2nd complete hepa B vaccine series E. Repeat blood exam after 3 months

618 D. Infants born to HBsAg-positive women should receive vaccine at birth, 1â€"2 mo, and 6 mo of age, with the first dose accompanied with HBIG. Postvaccination testing for HBsAg and anti-HBs should be done at 9â€"18 mo. If the result is positive for anti-HBs, the child is immune to HBV. If the result is positive for HBsAg only, the parent should be counseled and the child evaluated by a pediatric gastroenterologist. If the result is negative for both HBsAg and anti-HBs, a 2nd complete hepatitis B vaccine series should be administered, followed by testing for anti-HBs to determine if subsequent doses are needed. Administration of four doses of vaccine is permissible when combination vaccines are used after the birth dose; this does not increase vaccine reactogenicity. (Nelson's 11th ed)

A 3-year old female presents with fever, difficulty breathing and barking cough. The symptoms are more apparent when the patient is lying down and about to sleep. On physical exam, there is apparent respiratory distress, tachypnea, alar flaring, o inspiratory stridor and fever at 38.3 C. The most likely diagnosis is: A. Laryngotracheobronchitis B. Epiglottitis C. Bacterial tracheitis D. Retropharyngeal abscess E. Acute bronchitis

619 A. Viruses most commonly cause croup, the most common form of acute upper respiratory obstruction. Most patients have an upper respiratory tract infection with some combination of rhinorrhea, pharyngitis, mild cough, and low-grade fever for 1â€"3 days before the signs and symptoms of upper airway obstruction become apparent. The child then develops the characteristic “barking†cough, hoarseness, and inspiratory stridor. The low-grade fever may persist, although temperatures may reach 39â€"40°C (102.2â€" 104°F); some children are afebrile. Symptoms are characteristically worse at night and often recur with decreasing intensity for several days and resolve completely within a wk.

An 11 month old baby presents for evaluation of fever and rash. His mother noted a fever of 40 C two days ago. He appeared well and was eating and playing normally, so his mother was not alarmed. After the fever resolved, he developed a red rash on his trunk that progressed rapidly over the past 24 hours. What do you call the ulcers at the uvulopalatoglossal junction characteristic of this disease? A. Koplik's spots B. Herpangina C. Dewdrop on a rose petal D. Forscheimer's spots E. Nagayama's spots

62 E. In asian countries, ulcers at the uvulopalatoglossal junction called Nagayama's spots are commonly reported in infants with roseola

Which of the following is NOT true in Nephrotic Syndrome? A. Elevated serum cholesterol B. 85% is due to minimal change disease C. Reduced sodium reabsorption by the kidney D. Elevated triglycerides E. None of the above

620 C. Because of renal protein loss and hypoalbuminemia, there is reduced intravascular volume. This stimulates the rennin â€" angiotensisn system, resulting in enhanced renal sodium reabsorption. Eventually, the sodium partially contributes to the edema. The four symptoms of nephrotic syndrome is proteinuria, hypoalbuminemia, hyperlipidemia, edema.

A male infant was born at 32 weeks AOG via cesarian section because of bleeding from placenta previa. Soon after birth, he developed respiratory distress requiring supplemental oxygen and mechanical ventilation. Chest x-ray shows decreased lung volumes and a diffuse ground glass pattern with air bronchograms. Which of the following is the most likely diagnosis? A. persistent pulmonary hypertension of the newborn B. deficient surfactant C. fluid retention in the lungs D. bronchopulmonary dysplasia E. congenital heart disease

621 B.

You are called to the newborn nursery to evaluate a 1 day-old female infant with unusual physical findings. On examination, you note that the neonate's hands are clenched with overlapping digits and her lower extremities are extended and crossed. You also note the presence of a rocker bottom feet and delicate, small facial features. Which of the following chromosomal abnormalities is the most likely cause of the patient's features? A. Trisomy 13 B. Trisomy 18 C. Trisomy 21 D. Deletion on chromosome 7 E. Absence of a region on paternally derived chromosome 15

622 B. Trisomy 18

Which of the following organism is the most frequent cause of neonatal meningitis? A. Group B strep B. E. coli C. Listeria monocytogenes D. H. influenza B E. Strep pneumoniae

646 A.

An 18 year old boy, college sophomore, presents with high fever, headache, cough, conjunctivitis and a diffuse macular rash over his trunk and face. He is unsure of his immunization status. You suspect measles infection. Which of the following is correct regarding this diagnosis? A. Koplik spots would likely be present upon examination B. Vitamin A may improve his outcome C. Mortality is most commonly caused by measles encephalitis D. Diagnosis is based on culture and direct fluorescent antigen testing E. Corticosteroids will decrease sympotoms and improve outcome

624 B. Vitamin A is proven as a beneficial supportive treatmennt in measles. Koplik spots are transient and by the time the rash is present it can be no longer appreciated. Bacterial pneumonia is the MC of moratality.

You are called to the nursery to evaluate a male newborn with cyanosis. On auscultation, you hear a single S2 but no murmur. Pulse oximetry shows an oxygen saturation of 72 % in room air. An electrocardiogram reveals left axis deviation and left ventricular hypertrophy. What is his likely diagnosis? A. Tetralogy of Fallot B. Transposition of great arteries C. Truncus arteriosus D. Total anomalous pulmonary venous connection E. Tricuspid atresia with intact ventricular septum

625 E. Tricuspid atresia is the only cause of cyanosis in the newborn period that manifests with left axis deviation and left ventricular hypertrophy on ECG.

A 5 week old infant has been brought to the clinic due to jaundice. He was born full term, weighing 8 pounds and 9 ounces and he had an uncomplicated delivery and neonatal course. Today, his weight is 9 pounds, 1 ounce, and his vital signs are normal. On examination, his liver is enlarged and 4 cm below the right costal margin. jaundice is present. Laboratory evaluation reveals a total bilirubin of 12.9 mg/dL with a direct component of 5.9 mg/dL. Which of the following is the next most appropriate next step? A. begin phototherapy to treat jaundice B. refer the patient for a liver transplant C. begin ursodeoxhycholic acid to enhance bile flow D. order an urgent abdominal ultrasound and radionucleotide imaging study of the liver E. reassure the parents that no treatment is required as he likely has neonatal hepatitis

627 D. this patient has cholestasis, or retention of bile in the liver, manifested by elevated conjugated (direct bilirubin).in addition, the findings of hepatomegaly and poor growth support the diagnosis. MC causes of cholestatsis include biliary atresia and choledochal cyst.

A previously well 8 month old infant has had a runny nose and has been sneezing and coughing for 2 days. 5 hours ago, his cough became worse. On P.E, he is in moderate respiratory distress with nasal flaring, hyperexpansion of the chest, and easily audible wheezing without rales. What is the most likely diagnosis? A. Bronchial asthma B. Bronchiolitis C. Viral croup D. Epiglottitis E. Diphtheria

628 B. bronchiolitis

A deficiency of this trace element is associated with skin ulcers, reduced immune response and hypogonadal dwarfism: A. Chromium B. Cobalt C. Iron D. Magnesium E. Zinc

629 E. Zinc

An 8 year old male was brought to the pediatrician by his mother because he developed low grade fevers several days ago, and now has red cheeks and a new rash on his body. What is the most likely diagnosis? A. Exanthem subitum B. Rubella C. Rubeola D. Erythema infectiosum E. Infectious mononucleosis

63 D. The initial stage of erythema infectiosum is an erythematous facial flushing often described as a slapped cheek appearance. The causative agent is Parvovirus B19

The highest death rate in the pediatric age group occurs in: A. First month of life B. Between 2 and 12 months C. Between 2 and 4 years of life D. Just before puberty E. During adolescence

631 A. Mortality is highest in the first year of life

Which of the following is most likely to occur as an isolated manifestation of acute rheumatic fever? A. Arthritis B. Carditis C. Chorea D. Erythema marginatum E. Fever

632 C. Chorea can stand alone in the diagnosis of Acute rheumatic fever

Which malignancy is uncommon in children under 5 years old? A. Retinoblastoma B. Neuroblastoma C. Osteosarcoma D. Leukemia E. Wilm's tumor

633 C. Osteosarcoma is usually seen in puberty of eardly adolescence period.

A 12 year old boy presents with severe arthritis of the hips and sacroiliac joints. Laboratory studies reveal that the patient is HLA-B27 positive. Which of the following is the most likely diagnosis? A. Early onset pauciarticular juvenile rheumatoid arthritis B. Late onset pauciarticular juvenile rheumatoid arthritis C. Rheumatoid factor-negative polyarticular JRA D. Rheumatoid factor-positive polyarticular JRA E. Systemic-onset JRA

636 B. Late onset pauciarticular JRA is male predominant and is almost presents in children older than 8 years old.

A 7 month old male infant is brought to your clinic with a 1 month long history of fussiness, intermittent vomiting, and nonbloody, foul- smelling stools. He has not gained weight since his 6 month health maintenance examination. His diet consists of formula and wheat cereal that was introduced at 6 months of age. Which of the following is the most likely diagnosis? A. lactase deficiency B. celiac disease C. crohn's disease D. gastroesophageal reflux disease E. cow's milk protein intolerance

637 B. Celiac disease may presnt with vomiting, bloating, foul-smelling stools, or failure to thrive.

When does physiologic anemia of TERM infants occur? A. 3-6 weeks of age B. 6-8 weeks of age C. 8-12 weeks of age D. 12-16 weeks of age E. At birth

638 C. 8-12 weeks: TERM. 6-8 weeks: PRETERM

An 8 year old male was brought to the pediatrician by his mother because he developed low grade fevers several days ago, and now has red cheeks and a new rash on his body. What is the primary target of the causative organism? A. Lymphoid cells B. Myeloid cells C. Erythroid cells D. Peyer's patches E. Skin langerhans cells

64 C. The primary target of B19 infection is the erythroid cell line, specifcially erythroid precursors near the pronormoblast stage. There is progressive depletion of erythroid precursors and a transient arrest of erythropoesis.

A 2 year old patient was doagnosed to have a hydrocephalus. He was operated and ventriculoperitoneal shunt was applied. After 1 year, infection of the shunt was noted. What is the most common causative organism for this kind of infection? A. Streptococcus pyogenes B. Staphylococcus epidermidis C. Staphylococcus aureus D. Pseudomonas aeruginosa E. E. coli

640 B.

Another 3 year old patient was brought tou you due to rashes. This time, it appeared during the peak of fever spikes and your PE showed unilateral posterior lymphadenopathy. What is your diagnosis? A. Roseola B. Rubeola C. Rubella D. Fifth disease E. Infectious mononucleosis

648 C. German Measles (rubella)

A 2 year old child presents to the ER with high fever, respiratory distress and brassy cough. CXR was done showing subglottic narrowing with ragged tracheal air column. Diagnosis? A. Croup B. Laryngomalacia C. Laryngitis D. Bronchitis E. Bacterial tracheitis

669 E. Bacterial tracheitis is a bacterial infection of the trachea and is capable of producing airway obstruction.One of the most common causes is Staphylococcus aureus and often follows a recent viral upper respiratory infection. It is the most serious in young children, possibly because of the relatively small size of the trachea that gets easily blocked by swelling. The most frequent sign is the rapid development of stridor.

Most common causative agent of external otitis? A. S. aureus B. S.pneumoniae C. Parainfluenza D. Pseudomonas E. H. influenza type b

670 D. Otitis externa (also known as "External otitis" and "Swimmer's ear") is an inflammation of the outer ear and ear canal. Most commonly caused by Pseudomonas aeruginosa.

True of murmurs in children? A. Absence of murmurs rules out congenital heart defects B. All diastolic murmurs are pathologic C. Physiologic murmurs have a grade of 3/6 or greater D. A and c only E. All of the above

671 B. Innocent murmurs in children are physiologic, usually grade 2/6 or less. Absence of murmurs does not rule out CHD. All diastolic murmurs are pathologic.

True of sources of Infective endocarditis and its causative agent, except? A. S. mutans : dental procedures B. Fungi : open heart surgery C. Staph. epidermidis : indwelling catheter D. S. aureus : previously normal heart E. None

672 A. Streptococcus viridans is the most common cause of infective endocarditis after dental procedures.

Patient presented with conjunctivitis, polymorphous exanthems, palpale lymph nodes > 1.5 cm diameter, strawberry tongue and swelling of hands, diagnosis? A. Scarlet fever B. SLE C. Kawasaki D. Henoch - Scholein Purpura E. Rheumatic fever

673 C. Kawasaki disease s an autoimmune disease in which the medium-sized blood vessels throughout the body become inflamed. It is largely seen in children under five years of age. It affects many organ systems, mainly those including the blood vessels, skin, mucous membranes, and lymph nodes; however its rare but most serious effect is on the heart where it can cause fatal coronary artery aneurysms in untreated children.

An 8 year old boy with a history of sore throat is seen with a palpable rash on the legs and buttocks associated with abdominal pain and diarrhea. On PE, spleen and liver were enlarged. Urinalysis showed microscopic hematuria. CBC showed thrombocytopenia. what is the cause of this patient's hematuria? A. Lupus nephritis B. Membranoproliferative glomerulonephritis C. Post - streptococcal glomerulonephritis D. Henoch Schonlein Purpura nephritis E. none

674 D. Henochâ€"Schönlein purpura is a disease of the skin and other organs that most commonly affects children. With kidney involvement, there may be a loss of small amounts of blood and protein in the urine, but this usually goes unnoticed; in a small proportion of cases, the kidney involvement proceeds to chronic kidney disease. HSP is often preceded by an infection, such as pharyngitis.

Treatment of pediatric patient with primary TB? A. 2 HRZE/ 4HR B. 2 HRZ/4HR C. 9 INH D. 2HRZES/1HRZ/5HR E. None of the above

676 B. Primary TB is treated with 2 months of INH, rifampicin and pyrazinamide plus additional 6 months of INH + rifampicin only. 9 months INH is for treatment of latent TB.

A 9 month old infant is brought to the ER because of development of rash at the trunk area. History showed a 3-day fever of about 39 degrees. The rashes appeared after the resolution of fever. Diagnosis? A. Rubella B. Rubeola C. Roseola D. Erythema infectiosum E. Varicella

677 C. Rubella - german measles, rubeola - measles, erythema infectiosum - 5th disease, varicella - chicken pox

Infection with EBV is associated with what type of malignancy? A. Hepatocellular carcinoma B. Osteosarcoma C. Squamous cell CA of the esophagus D. Burkitt's lymphoma E. Non - hodgkin's lymphoma

678 D. EBV is associated with particular forms of cancer, such as Hodgkin's lymphoma, Burkitt's lymphoma, nasopharyngeal carcinoma, and conditions associated with human immunodeficiency virus (HIV) such as hairy leukoplakia and central nervous system lymphomas.

Contraindication for gastric lavage? A. coma B. seizures C. Depressed gag reflex D. Changes in sensorium E. AOTA

679 E. Lavage is contraindicated when patients have a compromised, unprotected airway and in patients at risk of gastrointestinal hemorrhage or perforation. Relative contraindications include when the poisoning is due to a corrosive substance , hydrocarbons , or for poisons that have an effective antidote.

Best clinical indicator for measure or under or overweight infants A. Height B. Weight C. BMI D. Bone age E. Weight for height

680 C. BMI has been used by the WHO as the standard for recording obesity statistics since the early 1980s.

A 5 yo boy came to ED with complaints of intermittent abdominal pain associated with loose stools. He has recently recovered from a viral upper respiratory infection. Physical examination findings revealed maculopapular lesions on the legs and buttocks. CBC, coagulation studies, and electrolytes are normal. Urinalysis revealed RBC = 6-8/hpf, WBC = 2-4/hpf, (+) occasional cylindruria, (+) albuminuria. Which of the following is the most likely diagnosis to the case presented? A. Kawasaki Disease B. Guillain-Barré Syndrome C. Henoch-Schönlein Purpura D. Waardenburg Syndrome

681 C. Answer: C. Henoch-Schönlein Purpura (pp. 198, Pediatrics: USMLE Step 2 CK, 2008-2009 edition). SIMILAR TO PREVIOUS BOARD EXAM CONCEPT/PRINCIPLE

A 14 yo girl who has not achieved menarche presents to your clinic with her concerned mother. The mother is afraid that her daughter is not “normal.†On physical examination, the patient appears well-nourished and is in the 50th percentile for height and weight. Her breast examination shows the areolar diameter to be enlarged, but there is no separation of contours. Her pubic hair is increased in amount and is curled but is not course in texture. In order for you to classify the time, course and progress of the patient’s changes during puberty, you will categorize her to what Sexual Maturity Rating (SMR)? A. SMR I B. SMR II C. SMR III D. SMR IV

682 C. Answer: C. SMR III (pp. 285 and 286, Pediatrics: USMLE Step 2 CK, 2008-2009 edition).

The Kasai procedure is indicated for which of the following? A. Neonatal hepatitis B. Biliary atresia C. Metabolic liver disease D. Fulminant hepatic failure E. None of the above

7 B. Despite initial success of the Kasai operation, patients with biliary atresia eventually get liver transplantation. Source: Nelson's Textbook of Pediatrics 19th edition 1387

A child was brought to the Emergency Room due to severe abdominal pain, nausea with gradual distention. The PGI had a difficulty in inserting the Nasogastric tubing. Few hours after, patient ha d several episodes of vomiting. A plain abdominal x- ray demonstrates a huge air-filled distended bowel like the shape of an inverted U, with the convexity of the U facing the right upper abdominal quadrant. What is the most likely diagnosis? A. Volvulus B. Intussusceptions C. Pyloric Stenosis D. Meckel’s Diverticulum

685 A. Answer: A. Volvulus Notes: The radiographic finding of volvulus has been described as the kidney-bean shape, coffee-bean shape, bent inner tube shape, ace of spades or ‘Omega- loop sign’

A toddler was brought to your clinic for multiple bouts of watery, non-bloody diarrhea without vomiting. He is tachycardic, with weak pulses, reduced skin turgor and sunken eyes. He is lethargic but irritable to touch. He gulps vigorously the oral rehydrating solution offered to him. Upon your assessment based on IMCI, which of the following management plans is the most appropriate in this patient? A. Give fluid and food and follow-up in 5 days if not improving B. Continue feeding and follow-up in 5 days C. Refer urgently to a hospital D. Treat for 5 days Cotrimoxazole and follow-up after 2 days

686 B. Answer: B. Continue feeding and follow-up in 5 days Notes: The patient has AGE/AID with SOME dehydration. The appropriate management is choice B. Cotrimoxazole may be given if with blood in stool.

Upon physical examination of a newborn, an extraordinary division of the body from the forehead to the pubis into red and pale halves was noticed. This is most likely A. Caused by general circulatory instability B. Related to a transient fluctuation in skin temperature C. Due to anemia, asphyxia, shock or edema D. A transient and harmless condition

687 D. Answer: D. A transient and harmless condition Notes: • Mottling - general circulatory instability, related to transient fluctuation in skin temperature • Pallor â€" due to anemia, asphyxia, shock, or edema • Harlequin Color Change â€" a transient and harmless condition (extraordinary division of the body from the forehead to the pubis into red and pale halves)

Dr. B. Aguhan, a first year pediatric resident is called to the newborn nursery to evaluate a 2-day- old who has passed a bloody stool. The patient was born by NSVD to a 34 yo primigravida. The birth weight was 3.7 kg, and Apgar Score of 8,9. The mother had good prenatal care throughout the entire pregnancy. She denied using tobacco, alcohol, or drugs during her pregnancy. The patient has been sleeping, feeding, defecating, and urinating well since birth. Vital signs are stable, and the rest of the PE findings are within normal limits. Which of the following tests would best determine the cause of the bloody stool? A. Apt test on the blood in the stool B. Hemoglobin electrophoresis of the blood from the stool C. Barium enema D. Methylene blue stain in the stool

689 A. Answer: A. Apt test on the blood in the stool. Notes: The most common cause of blood in the stool of a newborn is swallowed maternal blood. The Apt test helps distinguish adult haemoglobin from fetal haemoglobin. It is easier and less expensive than electrophoresis. If adult Hb is detected, the blood swallowed is maternal blood. If fetal Hb is found, a search begins for the cause of the bleeding.

Which of the ff screening tests results is condiered to be positive for sepsis? A. Total WBC 3000/mm3; immature/total neutrophil ratio of 0.1 and gastric aspirate with neutrophils 3/hpf B. Micro-ESR 10 mm in 1 hr, total WBC of 4500/mm3 and gastric aspirate with neutrophil of 2/hpf C. CRP 3 mg/dl; immature/total neutrophil ratio of 0.5 and gastric aspirate with neutrophil 6/hpf D. Total WBC 6300/mm3; CRP 0.5 mg/dl immature/total neutrophil ratio of 0.2 E. All of the above

69 C. An immature/total neutrophil ratio of more than 0.2, WBC of less tha 5000, ANC of less than 1,500 and platelet of less than 100,000 is indicative of sepsis in the newborn.

Five days ago, a 10 yo child had an upper respiratory infection and was given symptomatic treatment and an antipyretic (i.e., Aspirin for fever). She had seemingly recovered but now presents with fever, protracted vomiting, and lethargy. Physical examination reveals mild hepatomegaly. Serum analysis found total bilirubin, serum transaminases, and serum ammonia to be increased. CSF opening pressure was 210 mmHg. CSF cell count & differential count and remaining parameters were normal. Which of the following is the most likely diagnosis? A. Hepatits A Virus (HAV) B. Drug-induced Hepatitis C. Gilbert Syndrome D. Reye Syndrome

690 D. Answer: D. Reye Syndrome Notes: Reye Syndrome is a secondary mitochondrial hepatopathy that, in genetically susceptible individuals, has a very high association with the ingestion of aspirin-containing medicines during influenza-like illness or varicella. It has a biphasic course. After the child seems to have recovered, the symptoms of vomiting, lethargy, and confusion appears and progress quickly. Liver enzymes and the serum ammonia level are elevated. Treatment requires early recognition and control of increased intracranial pressure.

During examination of a 2-month old infant, you note that the infant’s umbilical cord is still firmly attached. This delayed cord separation is suggestive of which of the following? A. Occult omphalocele B. Leukocyte Adhesion deficiency C. IgG subclass deficiency D. Umbilical granuloma E. Persistent urachus

691 B. Answer: B Delayed separation (after 1 month) characteristic of LAD type 1; px will have overwhelming bact.infection. Omphalocele- abdominal wall defect w/ protrusion of intestine Umbilical granuloma forms after cord is separated. Persistent urachus- urine coming out of umbilicus.

A 2-yr.old boy is brought to the E.R. w/ complaint of fever x 6 days and development of a limp. P/E reveals an erythematous exanthem over his body, ocular conjunctivitis, dry, cracked lips, red throat and cervical lymphadenopathy, (+) grade 2/6 systolic ejection murmur at LLSB. WBC Count & differential show predominance of neutrophils w/ increased platelets on smear. Your impression is A. Scarlet fever B. Rheumatic fever C. Kawasaki disease D. Juvenile rheumatoid arthritis E. Infectious mononucleosis

693 C. Answer: C Other S/Sx can be asso.w/ many conditions.Conjunctivitis is suggestive of Kawasaki.Fissured lips could occur due to dehydration from fever. Nuetrophilic predominance and inc.ESR is common to all but inc.plt within the constellation of these symptoms is seen only in Kawasaki.

A 16-year old male patient, heartbroken from a recent breakup tried to commit suicide by taking an unknown quantity of unknown material he found at home. Patient was then brought to the E.R within 30 mins. from ingestion. For which of the following household substances/medications should activated charcoal be given as part of emergency center treatment? A. Liquid sosa B. Ethylene glycol C. Zonrox D. Phenobarbital E. Lithium

694 D. Answer: D Compounds not adsorbed: alcohols, acids, ferrous sulphate, strong bases, cyanide, lithium & potassium. Activated charcoal dec.half-life & inc.clearance of drugs w/ enterohepatic circulation.

Which of the ff statements describes accurately meconium aspiration syndrome? A. 10-15% of births occur in premature babies B. Respiratory distress usually manifests within 24 hours after birth C. Chest x-ray is usually normal D. Routine intubation is not indicated for vigorous babies E. None of the above

70 D. Routine intubation to aspirate the lungs of vigorous infants born through meconium stained fluid is not effective in reducing MAS. 10-15% occurs in term or postterm babies. Respiratry distress occurs within the first hour after birth. A typcial CXR is char by patchy infiltrates and coarse streaking of both lugn fields.

A newly born baby girl os observed to have tremors and hyperirritability. Moter finally admitted taking an illicit drug. Which of the ff substances/drugs is commonly associated with the symptoms? A. Tobacco B. Alcohol C. Cocaine D. Heroin E. All of the above

71 D. Heroin is associated with tremors and hyperirritability of the infant

An 8 year old girl is brought to the ER with a fever of 38.5 C, stiff neck, photophobia and headache. Lumbar puncture was perfromed revealing the ff results: WBC 38- cells/mm3 with 40% PMN and 60% lymphocyte; normal protein and glucose and negative gram stain. Which of the ff is the most likely cause of her mengitis? A. Neisseria mengitidis B. Streptococcus pneumoniae C. Enterovirus D. Cryptococcus neoformans E. Mycobacterium tuberculosis

73 C. The CSF analysis is most consistent with aseptic mengitis, specifically viral menigitis. Enterovirus are the most common cause of viral menigitis. There is lymphocytic predominance and normal protein and sugar.

An 8 year old girl presents with sore throat, fever and a rough sandpaper like rash over her trunk and extremities. A throat culture is positive for group A Beta hemolytic streptococcus. Treatment of her infection with antibiotics will prevent which of the ff complications? A. Reactive arthritis B. Rheumatic fever C. Post strep glomerulonephritis D. Guillan Barre syndrome E. All of the above

74 B. The patient's clinical presentation is consistent wit scarlet fever, caused by the erythrogenic strain of GABHS. Althoigh there are multiple complications of GABHS as mentioned in the question, only rheumatic fever will be prevented by treatment of antibiotics.

A newborn male had cyanosis on birth. On auscultation, you hear a single S2 but no murmur. O2 sat is 72% in room air. An ECG reveals left axis deviation and left ventricular hypertrophy. What is the most likely diagnosis? A. Tetralogy of Fallot B. Transposition of the great arteries C. Truncus arteriosus D. Total anomalous pulmonary venous connection E. Tricuspid atresia with intact ventricular septum

75 E. Tricuspid atresia is the only cause of cyanosis in the newborn period that manifests with left axis deviation and left ventricular hypertrophy on ECG. Patients with TA without a ventricular septal defect have a single S2 as a result of the sual coexistence of pulmonary atresia and do not have a murmur.

A 3 year old boy presents with high grade fever, anorexia and drooling. He has been previously well. His immunixation records are up to date. On examination he appears very ill and prefers to lean forward on his hands and his neck hyperextended. His voice is muffled. Which of the ff is correct regarding his diagnosis? A. The patient likely has bacterial tracheitis and should be started on antistaphylococcal antibiotics. B. Racemic epinephrine should be immediately administered. C. The patient should have his airway visualized and intubated in a controlled environment D. A neck radiograph will show a steeple sign E. The throat should be examined to rule out a retropharyngeal abscess

77 C. All of the symptoms and PE findings of the patient points to a possible epiglottitis. One must avoid erxcessive stimulation, including examination of the pharynx with a tongue depressor bec this will induce respiratory distress. Evaluation of the airway and intubation is necessary.

A 5 month old infant in a daycare facility develops low grade fever, rhinorrhea and cough. A few days later, she is brought to the ER with tachypnea, chest retractions, diffuse expiratory wheezing and fine inspiratory rales bilaterally. Which of the ff is correct regarding his diagnosis? A. Chest xray will demonstrate decreased lung volumes with bilateral lobar consolidation B. Chlamydia trachomatis should be condered as a possible etiologic agent C. Supportive care is the most important management D. IV antibiotics should be started E. Ribavarin should be administered.

78 C. The patient's clinical features are conistent with bronchiolitis, a lower resp. tract infection. Still, supportive care is the most effective management and inhaled bornchodilators have some benefit as well. Chest x-ray will demonstrate hyperinflation. Evidence of ribavarin in the treatment if bronchiolitis is lacking and should only be given in severely ill infants.

A 9 month old girl is brought to the ER with a 1 day history of intermittent, inconsolable crying. She seems comfrotable in between the attacks. She has vomited twice and has had one bowel movement that the mother described as bloody. Based on the clinical presentation what is the most appropriate diagnostic procedure? A. Plain abdominal xray B. Plain whole abdomen CT scan C. Barium enema D. Surgical exploration E. Abdominal UTZ

79 C. The clinical picture is consistent with intussception. The most impt diagnostic test at this time is the contrast enema which demonstrates the involved segment. In addition, the pressure from the contrast enema will usually reduce the intussception.

An 8 month old manifests with fussiness and emesis and refuses to eat. 1 week ago, he was brought for consult and was diagnosed with URTI. On the morning of consult, his mother noted currant jelly like material on his diaper. On PE, a sausage shaped mass was palpable in the abdomen. There is likewise abdominal distention and rectal exam reveals blood in the stool. The most likely diagnosis is: A. Viral diarrhea B. Duodenal atresia C. Intussusception D. Hypertrophic pyloric stenosis E. Intestinal adenoma

8 C. Aside from that, intussusception may present with lethargy out of proportion to the intestinal signs and symptoms. Intussusception in children is usually due to lymphoid hyperplasia in the intestines. This becomes the lead point. The most common location is ileocolic. Source: Nelson's Textbook of Pediatrics 19th edition p1288

2. An 18 year old male came in for complaints of cough and watery nasal discharges 1 week PTA. Upon examination at the ER, T = 38.5C, RR = 65, dyspnea and difficulty to console the child. After 1 hour, pulse oximetry revealed 70% thus patient was intubated. Auscultation of chest revealed fine crackles on both lung fields with occasional wheezes. CBC taken was normal. CXR revealed hyperinflation and air trapping. Which of the following is the most likely diagnosis? a. bronchiolitis b. pneumonia c. acute bronchitis d. asthma e. NOTA

82 A. Nelson's Textbook of Pediatrics 19th edition p.1457

3. A 2 year old female comes in for complaints of low grade fever and watery nasal discharges 1 week PTA. Two days PTA, dry hacking cough was noted now becoming productive with yellowish sputum. Auscultation of the chest revealed coarse breath sounds. CXR and CBC were unremarkable. Which of the following is the most likely diagnosis? A. bronchiolitis B. pneumonia C. acute bronchitis D. asthma E. NOTA

83 C. Nelson's Textbook of Pediatrics 19th edition p.1460

4. A five year old male presents to the emergency room with sudden onset right sided hemiparesis. Two months ago, patient was diagnosed with iron deficiency anemia and ascariasis. Which of the following is the most likely diagnosis? A. Todd paralysis B. migraine C. meningitis (CNS infection) D. ischemic stroke E. NOTA

84 D. The acute onset of a focal neurologic deficit in a child is stroke until proven otherwise. THe msot common focal presentation is hemiparesis but acute visual, speech, sensory or balance deficits also occur. Three main etiologies are arteriopathic, cardiac, and hematological (sickle cell anemia and IDA) Nelson's Textbook of Pediatrics 19th edition p.2080

A five year old male presents to the emergency room with sudden onset right sided hemiparesis. Two months ago, patient was diagnosed with iron deficiency anemia and ascariasis. Which of the following laboratory tests in the above situation would confirm your impression? A. blood cultures B. MRI C. EEG D. lumbar tap E. CT scan

85 B. Acute ischemic stroke is a clinical and radiographic diagnosis. CT imaging can demonstrate larger mature AIS and exclude hemorrhage, however, MRI identifies early and small infarcts and is therefore required to exclude ischemic stroke. Nelson's Textbook of Pediatrics 19th edition p.2080

A five year old male presents to the emergency room with sudden onset right sided hemiparesis. Two months ago, patient was diagnosed with iron deficiency anemia and ascariasis. Which of the following would be the most appropriate initial management strategy for the patient above? A. call your friendly pediatric neurologist B. neuroprotective strategies C. antithrombotic strategies D. correction of iron deficiency anemia E. rehabilitation

86 A. Children with these presentations require urgent neuroimaging and consultation with a child neurologist as emergency interventions may be indicated. Nelson's Textbook of Pediatrics 19th edition p.2080

A five year old male presents to the emergency room with sudden onset right sided hemiparesis. Two months ago, patient was diagnosed with iron deficiency anemia and ascariasis. What is the definitive treatment in the above condition? A. broad spectrum antibiotics B. carbamazepine C. careful control of cerebral perfusion pressure D. antithrombosis E. NOTA

87 D. Emergency thrombolysis is not yet established for children. Antithrombotic strategies depend on the suspected cause but include anticoagulation with heparins or antiplatelet strategies such as aspirin. Nelson's Textbook of Pediatrics 19th edition p.2080

9. A 2 year old male, CF, presents to the clinic with fever and rash which began on the forehead and on upper neck spreading downward to extremities which subsequently faded leaving a fine desquamation. CF also has a brother who is 13 months old who has been exposed 4 days ago when his grandmother brought him along to visit him. Which of the following is the most appropriate management? A. give measles vaccine to CF's brother B. give measles immune globulin to CF's brother C. give both vaccine and immune globulin to CF's brother D. Give vitamin A 50,000 IU to CF E. give vitamin A 100,000 IU to CF's brother

89 B. Measles vaccine only effective in prevention or modification of measles if given within 72 hours of exposure. Immune globulin may be given up to 6 days after exposure. Vitamin A dosages is" 50,000 IU for less than 6 months, 100,000 6mo-1 yr, and 200,000 >= 1 year old Nelson's Textbook of Pediatrics 19th edition p.1073

10. A 25 year old G3P2 (2002) mother, KEC, was admitted to VCMC hospital due to hyperemesis gravidarum. Her 2 year old child started to have intensely pruritic erythematous macules that evolve through papules then vesicles distributed centripetally 2 days ago. KEC also has a 12 month old child at home and a 14 year old neighbor who got exposed. Which of the following is the most appropriate management? A. give varicella vaccine to the 12month old child B. give VariZIG to KEC C. give VariZIG to the 14 year old neighbor D. both A and B E. both B and C

90 D. Nelson's Textbook of Pediatrics 19th edition p.1110

11. When should you suspect the 14 year old neighbor to develop the same symptoms with the 2 year old child of KEC? A. 3 - 5 days B. 10 - 21 days C. 24 - 48 hours D. 7 - 10 days E. 3 - 4 weeks

91 B. Nelson's Textbook of Pediatrics 19th edition p.1105

A 25 year old G3P2 (2002) mother, KEC, was admitted to VCMC hospital due to hyperemesis gravidarum. Her 2 year old child started to have intensely pruritic erythematous macules that evolve through papules then vesicles distributed centripetally 2 days ago. KEC also has a 12 month old child at home and a 14 year old neighbor who got exposed. After 2 days, KEC was discharged. Nurse JoAnne who is also a neighbor of KEC came to visit her to check on her blood pressure at home but was also exposed to KEC's 2 year old child. Which of the following is not true regarding prevention of VZV transmission? A. person is contagious 24-48 hours before rash appears B. unvaccinated health care workers who are exposed should be furloughed for 8 - 21 days C. varicella vaccination can be given to KEC D. evidence of immunity may be through verification of history of herpes zoster disease by health care provider E. NOTA

92 C. Varicella vaccine is contraindicated during pregnancy Nelson's Textbook of Pediatrics 19th edition p.1109

14. RL, a 16 year old male, came in for recurrent tonsilitis 5x this year. Patient underwent bilateral tonsillectomy and during the 1st postoperative day, recurrent bleeding at operative site was noted. Laboratories requested were as follows: bleeding time = 3 minutes (2 - 8); PTT= 78 seconds (21 - 35); INR = 1.0; mixing studies = correction of PTT; CBC and thrombin time within normal limits. Which of the following is the most likely diagnosis? A. von willebrand's disease B. antiphospholipid antibody syndrome C. hemophilia D. DIC E. parahemophilia

94 C. In severe hemophilia, PTT value is usually 2 - 3 times the upper limit of normal. Results of other screening tests (platelet count, bleeding time, PT, thrombin time) are normal. Nelson's Textbook of Pediatrics 19th edition

A 7 year old male came in for high grade fever 6 days PTC associated with a polymorphous rash, diffuse injection of pharynx, cervical lymphadenopathy, and periungual peeling of toes. Which of the following is the most appropriate management for this patient? A. ASA B. IVIG C. dexamethasone D. both A and B E. all of the above

95 D. In the acute stage of Kawasaki disease, give IVIG and aspirin. Long term therapy for patients with coronary abnormalities include aspirin, clopidogrel, warfarin, LMWH. Nelson's Textbook of Pediatrics 19th edition

A 12 year old female came in for knee pain and swelling 3 months PTA. Patient cannot participate in physical education classes due to limited range of motion. One month PTA, patient now noted ankle and wrist pain and swelling associated with fever, generalized lymphadenopathy, and migratory salmon colored rash. Upon physical examination, pericardial friction rub was heard. ANA was negative while CBC showed anemia. Which of the following is the most likely diagnosis? A. systemic JIA B. sarcoidosis C. SLE D. rheumatoid arthritis E. acute rheumatic fever

96 A. It is characterized by arthritis, fever, and prominent visceral involvement including HSM, LAD, and serositis (pericarditis). The evanescent salmon colored lesions classic for systemic onset disease are linear or circular and most commonly distributed over trunk and proximal extremities. Nelson's Textbook of Pediatrics 19th edition

18. Which of the following congenital heart diseases presents with increased pulmonary blood flow? A. total anomalous pulmonary venous return B. hypoplastic left heart syndrome C. double outlet right ventricle D. both A and B E. both B and C

98 D. Also includes TGA, DORV without PS, TA, single ventricle if decreased pulmonary blood flow: TOF, TA, Ebstein anomaly Nelson'7s Textbook of Pediatrics 19th edition

19. A neonate 36 weeks by BS was admitted to the NICU for further evaluation due to an episode of apnea and cyanosis. Currently, patient is grunting with subcostal retractions. CXR revealed ground glass infiltrates on both lung fields. O2 sats remain 89%. Which of the following is not true? A. hyperoxia test can distinguish cyanotic congenital heart disease from pulmonary disease B. neonates with Congenital heart disease can increase PaO2 when given 100% oxygen C. if due to a CNS disorder, PaO2 will not normalize with 100% oxygen through artificial ventilation D. Both A and B. E. Both B and C

99 E. If PaO2 rises above 150mmHg during 100% O2, an intracardiac right to left shunt can usually be excluded. If with pulmonary disease, PaO2 generally increases significantly with 100% oxygen as ventilation - perfusion inequalities are overcome. If due to CNS disorder, PaO2 will normalize through artificial ventilation. Nelson's Textbook of Pediatrics 19th edition


Related study sets

Critical Care: Chapter 19: Endocrine Alterations

View Set

Women during the French Revolution

View Set

History of Graphic Design Exam 1

View Set

Module 2.10: Mutual Funds and Other Investments

View Set

Semester #1, Unit #2 Multiple Choice Study Guide- A.P. Biology

View Set

Anatomy and Physiology Unit 1 Practice quiz

View Set

Tax Quizzes and Post-Coverage Quizzes

View Set